<<

HISTORY March 23, 2018 SOCIO RELIGIOUS PROBLEMS IN DURING THE 19th CENTURY General Problems – Education, Gender inequality, Superstition

Specific Problems – Sati, condition of Widow, Tonsure, Child Marriage, Female Infanticide

Other problems– purdah , polygamy, Dowry System  Reformist movements – Responding to time, scientific temper of the modern era

 Revivalist movements – revival of old system

 Wahabi Movement

Causes Modern education

Revolutions in other countries (like France)

Pessimistic attitude towards activities of British and Christian Missionaries

Sanskritisation by Upper caste Impact Wave of reforms  Removal of Sati  Child infanticide banned  Widow remarriage act in 1856.  Focus on Education

National Orientation

Cultural consciousness

Woman Empowerment

 Marriageable age of girls was raised 1860. Negative effects Communal consciousness

Caste based identity

Community orientation rather than national orientation

Insecure feeling by large masses Limitations They concerned only small sections of society.

Worked for their respective community.

Failed to emphasize that colonial rule was inimical to their interest. Reforms regarding Women

 The conditions of women at the beginning of 19th century was miserable.

 They did not get equal treatment.

 They were denied Education.

 Female Infanticide, Child Marriages, Dowry System, Sati, Tonsure

 The movement for reforms regarding women addressed these issues. Q: What was sati and what does it means? The term sati literally means a 'pure and virtuous woman'. It was applied in case of a devoted wife who contemplated perpetual and uninterrupted conjugal union with her husband life after life and as proof thereof burnt herself with the dead body of her husband.  Indian reformers led by launched a frontal attack on the evil of sati.  With an eye, to the coming Charter debates in the British Parliament and anxious to get a renewal of its charter for another 20 years by presenting a creditable image of its activities in India, the Court of Directors encouraged Bentinck to enact legislation to suppress sati who banned it in1829. Raja Ram Mohan Roy : Father of the Indian Renaissance Learnt various languages as he thought religion can be studied only through the language its books were written in.

Used logical reasoning & rational thinking

1814:Set up Atmiya Sabha in Calcutta

Supported David Hare to set up Hindu college 1817 Founded Vedanta College at Calcutta in 1825 where he introduced Mechanics & Voltairs Philosophy

Wrote Gift to Monotheists (1809) Opposed Idol Worship

Translated to Bengali the Vedas & 5 Upanishads Demanded property inheritance rights for women. Political activist  Roy condemned oppressive practices of Bengali zamindars & demanded fixation of maximum rents.  He also demanded abolition of taxes on tax free lands.  He called for a reduction of export duties on goods abroad & abolition of the Company's trading rights.  He demanded judicial equality

 Roy was an internationalist

 Was influenced by the French Revolution.

 He stood for cooperation of thought and activity & brotherhood among nations.  He supported the revolutions of Naples & Spanish America & condemned the oppression of Ireland by absentee English landlordism & threatened emigration from the empire if the reform bill was not passed. Brahmo Sabha (1828)  Main theme “Nirgunasapna” (formless worship)

 Purpose was to purify  Promoted Monotheism – Oneness of God [opposites are polytheism – more than 1 God & Pantheism – Everything is God]

 The new society was to be based on the twin pillars of reason & the Vedas & Upanishads.

 Most of all it based itself on human reason which was to be the ultimate criterion for deciding what was worthwhile & what was useless in the past or present religious principles & practices.

 Followers of Samaj were Keshab Chandra Sen ,Debendra Nath Tagore, Ishwar Chandra Vidyasagar, Ashwani Kumar Datta, Derozians , Prasanna Kumar Tagore, Chandrashekhar Deb & Tarachand Chakravarty, 1st secretary of the Brahma Sabha. Continued  After Roy’s death, Debendranath Tagore headed Brahmo Sabha  Tagore repudiated the doctrine that the Vedic scriptures were infallible  In 1839 he, founded Tatvabodhini Sabha.  The Tatvabodhini Sabha & its organ the Tatvabodhini Patrika promoted a systematic study of India's past in the .

 The Samaj actively supported the movement for widow remarriage, abolition of polygamy, women's education & opposed superstitious practices & rituals  Samaj denied the need for a priestly class for interpreting religious writings

 Brahmo Samaj split, Keshub Chandra Sen , Anand Mohan Bose left & started “All India Brahmo Samaj”

 Under D Tagore, it was Adi Brahmo Samaj  AM Bose & Shiv Narayan Shastri started Saddharno Brahma Samaj 2012 Which of the following statements is/are correct regarding Brahmo Samaj?

1. It opposed idolatry.

2. It denied the need for a priestly class for interpreting the religious texts.

3. It popularized the doctrine that the Vedas are infallible.

Select the correct answer using the codes given below: a) 1 only b) 1 and 2 only c) 3 only d) 1, 2 and 3 Answer: B 2016 Consider the following :

1. Calcutta Unitarian Committee

2. Tabernacle of New Dispensation

3. Indian Reform Association

Keshab Chandra Sen is associated with the establishment of which of the above?

(a) 1 and 3 only (b) 2 and 3 only (c) 3 only (d) 1, 2 and 3 Answer b Henry Vivian Derozio (1809-1831) Teacher in Hindu college Calcutta 1826-31

Started the Young movement

Inspired by French revolution

Had radical thoughts of liberty & freedom

 Derozio was perhaps the 1st nationalist poet of modern India.

Due to his early demise the movement came to an end

SN Banerjee described the Derozians as "the pioneers of the modern civilization of Bengal, the conscript fathers of our race whose virtues will excite veneration & whose failings will be treated with gentlest consideration." Ishwar Chandra Vidyasagar (1820-91) Worked for the cause of emancipation of women. Promoted education among girls & set up institutions for them .  As a Government Inspector of Schools, he organised 35 girls' schools, many of which he ran at his own expense.  As Secretary to the Bethune School, he was one of the pioneers of higher education for women. All his life he campaigned against polygamy. Due to his efforts widow remarriage act was passed in 1856. In 1850, he protested against child-marriage. Principal of the Sanskrit College.  He opened the gates of the Sanskrit college to non-Brahmin students  To free Sanskrit studies from the harmful effects of self-imposed isolation, he introduced the study of Western thought in the Sanskrit College. He also helped found a college which is now named after him. He also devised a new Bengali primer & evolved a new prose style. Theosophical Society Started by Helena Blavatsky, Henry Steel Olcott, William Quan Judge in New York in 1875 After a few years Olcott & Blavatsky moved to India & established the International Headquarters at Adyar.

Theosophists advocated the revival & strengthening of the ancient religions of Hinduism, Zoroastrianism, & Buddhism. They recognized the doctrine of the transmigration of the Soul. They also preached the universal brotherhood of man.

As religious revivalists the Theosophists were not very successful.

Movement led by westerners who glorified & philosophical tradition. This helped Indians recover their self-confidence, even though it tended to give them a sense of false pride m their past greatness. (1847-1933) and Theosophical society  Annie Besant also joined the movement( came to India in 1893).

 Election of Annie Besant as its president after the death of Olcott in 1907.

 Worked for revival of Hinduism -its philosophy& modes of worship.

Founded the Central Hindu college at Banaras which was later developed by Madan Mohan Malaviya into the Banaras Hindu University.

Organized Home Rule Movement 2013 Annie Besant was

1. responsible for starting the Home Rule Movement

2. the founder of the Theosophical Society

3. Once the President of the

Select the correct statement/statements using the codes given below. a) 1 only b) 2 and 3 only c) 1 and 3 only d) 1, 2 and 3 Ans C Annie Besant joined the team much later, in 1889. Mahatma Jyoti Govindrao Phule Started the Dalit Movement Was against upper class Brahminical domination Satya Shodhak Samaj is a religion established by him in 1873.

This was started as a group whose main aim was to liberate the Shudra & Untouchable castes from exploitation and oppression.

Phule used the symbol of Rajah Bali as opposed to the, Brahmin's symbol of Rama.

He openly condemned the inequality in the religious books, orthodox nature of religion, exploitation of masses by the means of it, blind & misleading rituals, & hypocrisy in the prevalent religion.

Advocated the ideals of human well being in broader aspects, human happiness, unity, equality, & easy religious principles & rituals. Continued In 1851,Phule & his wife started a girls' school at Poona & soon many other schools came up. Active promoters of these schools were Jagannath Shankar Seth & Bhau Daji.

Through his writings & activities Phule always condemned caste hierarchy & the privileged status of priests in it.

Wrote Gulamgiri & Sarvajanic Satyadharma

Phule was also a pioneer of the widow remarriage movement

Vishnu Shastri Pundit founded the Widow Remarriage Association in the 1850's.

Another prominent worker in this field was Karsondas Mulji who started the Satya Prakash in Gujarati in 1852 to advocate widow remarriage. 2016 Satya Shodhak Samaj organized

(a) a movement for upliftment of tribals in Bihar

(b) a temple-entry movement in

(c) an anti-caste movement in

(d) a peasant movement in Punjab

Answer c Swami Dayanand Saraswati 1824-83 Mulshankar He had received education on Vedanta from a blind teacher named Swami Virajananda in Mathura.

Started Arya Samaj (1875) movement at Bombay & shifted headquarters to Lahore “Go Back to Vedas” was the slogan. Believed in infallibility of Vedas. Approved Vedas, rejected Puranas & anything related to western culture. He also favored the study of western sciences.  word was used for the 1st time by Swami Dayanand. Advocated equal rights for men & women. He was against idol worship, ritual, & priesthood

He opposed caste system. Inter caste marriage act 1872 Satyarth Prakash Continued  He attacked child marriage.

 Samaj fixed the minimum marriageable age at 25 years for boys & 16 years for girls.

 Swami once lamented the Hindu race as "the children of children".

 The work of the Swami after his death was carried forward by Lala Hansraj, Pandit Gurudutt, & Swami Shraddhanand, among others.

 Some of his followers later started a network of schools & colleges in the country to impart education on western lines.

 Lala Hansraj played a leading part in this effort.

 1902:Swami Shradhananda started the Gurukul near Hardwar to propagate the more traditional ideals of education. Continued This movement became anti Muslim & anti Christian movement.

Shuddhi movement began for reconversion of converted Hindus

Cow protection movement Ramakrishna Parmhansa (1834-1886)  Was a saintly person who sought religious salvation in the traditional ways of renunciation, meditation, & devotion(bhakti).

 In his search for religious truth or the realization of God he lived with mystics of other faiths, Muslims & Christians.

 He emphasized that there were any roads to God & salvation & that 'service of man was service of God, for man was the embodiment of God’.

 “So long as the millions live in hunger and ignorance, I hold everyman a traitor, who having been educated at their expense, pays not the least heed to them”. Disciple of Ramakrishna Paramhansa  Great humanist

Believed in Karma ( Action)

Proclaimed the essential oneness of all religions Vivekananda condemned the caste system and the current Hindu emphasis on rituals, ceremonies, and superstitions, & urged the people to imbibe the spirit of liberty, equality, & free-thinking.

He himself subscribed to Vedanta which he declared to be fully rational system.

Took part in World Religious Conference in US

Ramkrishna Mission started in 1897, Belur, Prarthana Samaj Religious reform

Prarthana Samaj was founded by Dr. Atmaram Pandurang in 1867 with an aim to make people believe in one God & worship only one God. Main reformers were the intellectuals who advocated reforms of the social system of Hindus.  Parallel to Brahmo Samaj

Although the adherents of Samaj were devoted theists, they didn’t regard the Vedas as divine or infallible.

They drew their nourishment from the Hindu scriptures & used the hymns of the old Marathi "poet-saints" like Namdev, Tukaram in their prayers.

Their ideas trace back to the devotional poems of the Vitthalas as part of the Vaishnava bhakti devotional movements of the 13th century in Maharashtra.

But, beyond religious concerns, the primary focus of the Samaj was social & cultural reform. Social reform Prarthana Samaj critically examined the relations between contemporary social & cultural systems & religious beliefs & gave priority to social reform as compared with the political changes already initiated by the British government.

Their reform movements led many projects of cultural change & social reform in , such as the improvement of the lot of women & depressed classes, an end to the caste system, abolition of child marriages & infanticide, educational opportunities for women, & remarriage of widows.

Its success was guided by Sir Ramakrishna Gopal Bhandarkar, a noted Sanskrit scholar, Narayan Chandavarkar, & Justice Mahadev Govinda Ranade. Mahadev Govinda Ranade(1842-1901)

Was the soul of The Indian National Social Conference.

Campaigned for abolition of caste, inter caste marriages, widow remarriage, education for women upliftment of low castes & Hindu Muslim unity.

Ranade emphasized that "the reformer must attempt to deal with the whole man & not to carry out reform on one side only".

Indian Social Conference-1887

Founded by M G Ranade & Raghunath Rao

Behramji Malabari was other important member

Met annually at the same time & venue as the INC

Can be called the social reform cell of the INC

Conference advocated inter caste marriages, opposed polygamy etc.

Launched pledge movement to inspire people to take a pledge against child marriage. 2012 During Indian freedom struggle, the National Social Conference was formed. What was the reason for its formation? a) Different social reforms groups or organisations of Bengal region united to form a single body to discuss the issues of larger interest and to prepare appropriate petitions/ representations to the government. b) Indian National Congress did not want to include social reforms in its deliberations and decided to form a separate body for such a purpose c) Behramji Malabari and M.G.Ranade decided to bring together all the social reform groups of the country order one organisation. d) None of the statements (a), (b) and (c) given above is correct on this context. Answer: C  An outstanding champion of new learning and social reform in Maharashtra was Gopal Hari Deshmukh, who became famous by the pen name of 'Lokahitawadi'.

 Advocated the reorganization of Indian society on rational principles & modern humanistic & secular values.

 He attacked Hindu orthodoxy & supported social & religious equality. He said, "If religion does not sanction social reform, then change religion."

Servants of India Society

founded the Servants of India Society in 1905.

 The aim of the society was to train national missionaries for the service of India; to promote, by all constitutional means, the, true interests of the ; and to prepare a cadre of selfless workers who were to devote their lives to the cause of the country in a religious spirit.

 After Gokhale's death (1915), Srinivasa Shastri took over as president Social Service League

 Another Gokhale follower Narayan Malhar Joshi founded it in Bombay with an aim to secure for the masses better & reasonable conditions of life & work

 They organized many schools, libraries, reading rooms, day nurseries & cooperative societies.

 Their activities also included police court agents' work, legal aid & advice to the poor & illiterate, excursions for slum dwellers, facilities for gymnasia & theatrical performances, sanitary work, medical relief & boys' clubs & scout corps.

 Joshi also founded the All India Trade Union (1920). (ca. 1854–1928)

Sree Narayana Guru Swami, was a Hindu saint, sadhu & social reformer

He was born into an Ezhava family, in an era when people from that community & other communities, faced much social injustice in the caste-ridden society.

Gurudevan, as he was known by his followers, led a reform movement in Kerala, rejected casteism & promoted new values of spiritual freedom & social equality.

S.N.D.P. Yogam

He stressed the need for the spiritual and social upliftment of the downtrodden by their own efforts through the establishment of temples and educational institutions.

In the process he denounced cultural convention of Chaturvarna. Justice Movement

of British India.

It was established in 1917 by T. M. Nair & P. Theagaraya Chetty as a result of a series of non-Brahmin conferences & meetings in the presidency.

Communal division between Brahmins & non-Brahmins began in the presidency during the late-19th & early-20th century, mainly due to caste prejudices & disproportionate Brahminical representation in government jobs.

They demanded separate representations for the lower castes in the legislature.

The 's foundation marked the culmination of several efforts to establish an organisation to represent the non-Brahmins in Madras. Self-Respect Movement

Movement with the aim of achieving a society where backward castes have equal human rights, & encouraging backward castes to have self-respect in the context of a caste-based society that considered them to be a lower end of the hierarchy.

It was founded in 1925 by E. V. Ramasamy (Periyar) in .

The movement was extremely influential not just in Tamil Nadu, but also overseas in countries with large Tamil populations, such as Malaysia & Singapore.

“We are fit to think of 'self-respect' only when the notion of 'superior' & 'inferior' caste is banished from our land“

 Self-respect marriage system Sarda Act Child Marriage Restraint Act 1929 popularly known as the Sarda Act after its sponsor Rai Sahib Harbilas Sarda to the British India Legislature in India was passed in 1929, fixed the age of marriage for girls at 18 years & boys at 21 years.

It came into effect in 1930 & it applied to all of British India, not just to Hindus.

It was a result of social reform movement in India.

The legislation was passed by the British Indian Government.

However, the Act remained a dead letter during the colonial period Sir Syed Ahmed Khan (1817-1898) Started the Aligarh movement Impressed by modern scientific thought & worked all his life to reconcile it with Islam. This he did, first of all, by declaring that the Quran alone was the authoritative work for Islam & all other Islamic writings were secondary.

According to him Muslim women should be given political & economic rights

Set up Mohammedan Anglo Oriental College in 1875 which became Aligarh Muslim University in 1920

Insisted on cooperation with Britishers & reforms among Muslims.

Persuaded Muslims to receive modern education.

Opposed purdah ,polygamy, easy divorce etc Continued  His loyal followers are collectively described as the Aligarh School.

 Chiragh Ali, the Urdu poet Altaf Husain Hali, Nazir Ahmad, & Maulana Shibli Nomani were some of the other distinguished leaders of the Aligarh School.

 Syed's progressive social ideas were propagated through his magazine Tahdhib-ul- Akhlaq Muhammad Iqbal (1876- 1938)  One of the greatest poets of modern India,  Influenced through his poetry the philosophical & religious outlook of the younger generation of Muslims as well as of Hindus.  Like Vivekananda, emphasized the need for constant change and ceaseless activity and condemned resignation, contemplation, & quiet 'contentment.  He urged the adoption of a dynamic out look that would help change the world  A humanist.  In fact he raised human action to the status of a prime virtue.  Man should not submit to nature or powers that be, he said, but should control this world through constant activity.  Nothing, was more sinful in his eyes than passive acceptance of things as they were.

 Condemning ritualism, asceticism, & other worldly attitude, he urged men to work for and achieve happiness in this world of the living. Religious Reform among the Parsis

 Religious reform begun in Bombay in the middle of the 19th century •  1851: Rehnumai Mazdayasan Sabha or· Religious Reform Association was started by Naoroji Furdonji, , S.S. Bengalee etc.

 Dadabhai was one of the founders of an association to reform the Zoroastrian religion & the Parsi Law Association which agitated for the grant of a legal status to women and for uniform laws of inheritance & marriage for the Parsis.

 The message of reform was spread by the newspaper Rast Goftar (Truth-Teller).

 It campaigned against the entrenched orthodoxy in the religious field

 Initiated the modernization of Parsi social customs regarding the education of women, marriage & the social position of women in general. HISTORY March 23, 2018 Revolutionary Terrorist Movement The Movement can be categorised in:-

 Pre-World War:-

 In the initial period, the movement was centred around religious symbols due to which it lacked mass appeal  These movements were directed towards various British Institutions but lacked proper planning

 Post World War:-

 The Movement was influenced from the Russian revolution of 1917, the movement became more organised and gained steam

 At this time, HSRA emerged. Most prominent leader of HSRA was . Pre-World War Militant Movement or Revolutionary Terrorist Movement  Emerged in first decade of 20th century in Bengal() and Maharashtra()

, Sandhya, Yuganthar were the groups formed in Bengal & Mithra Mela, Abhinav Bharat were formed in Maharashtra Bengal  By the 1870s, Calcutta’s student community was honeycombed with secret societies, but these were not active.

 Anushilan Samiti was the first revolutionary group organised in 1902 in Bengal.

 Established by Pramathanath Mitra it became one of the most organised revolutionary associations.

 It had two prominent if somewhat independent arms in East & West Bengal identified as Dhaka Anushilan Samiti centred in Dhaka & the group (centered at Calcutta) respectively. Jugantar  In April 1906, an inner circle within Anushilan (Barindra Kumar Ghosh, Bhupendranath Dutta) started the weekly Yugantar & conducted a few abortive ‘actions’.

 Barin Ghosh was the main leader.

 Rashbehari Bose & Sachin Sanyal had organised a secret society covering far-flung areas of Punjab, Delhi & United Provinces while some others like Hemachandra Kanungo went abroad for & political training.

 In 1907, an abortive attempt was made on the life of the very unpopular West Bengal Lt. Governor, Fuller, by the Yugantar group.

 In the 1920s, the Kolkata faction supported Gandhi in the Non-Cooperation Movement & many of the leaders held high posts in Congress.

Alipore Bomb Case  Also called the Manicktolla bomb conspiracy was the trial of a number of revolutionaries in Calcutta under charges of "Waging war against the Government" of the British between May 1908 and May 1909.

 The trial followed in the wake of the attempt on the life of Presidency Magistrate Douglas Kingsford in Muzaffarpur by and in 1908.

 Two ladies, instead, got killed. Prafulla Chaki shot himself dead while Khudiram Bose was tried and hanged.

 The whole gang was arrested including the Ghosh brothers, Aurobindo and Barindra, who were tried in this case.

 During the trial, Narendra Gosain, who had turned approver, was shot dead in jail.

Delhi-Lahore conspiracy case  Involving Rashbehari Bose along with Sachin Sanyal, the conspiracy culminated on the attempted assassination on 23 December 1912 when a home-made bomb was thrown on the when the ceremonial procession moved through Chandni Chowk . The Viceroy escaped with his injuries, along with Lady Hardinge.

 In the aftermath of the event, efforts were made to destroy the Bengali & Punjabi revolutionary underground, which came under intense pressure for sometime.

 Rash Behari successfully evaded capture for nearly 3 years, becoming actively involved in the Ghadar conspiracy before it was uncovered, & fleeing to Japan in 1916.

Maharashtra  The first of the revolutionary activities here was the organisation of the Ramosi Peasant Force by Vasudev Balwant Phadke in 1879, which aimed to rid the country of the British by instigating an armed revolt by disrupting communication lines. It hoped to raise funds for its activities through dacoities. It was suppressed prematurely.  During the 1890s, Tilak propagated a spirit of militant nationalism, including use of violence through Ganapati & Shivaji festivals & his journals Kesari & Maharatta.  His disciples—the Chapekar brothers, Damodar, Balkrishna & Vasudeo—murdered the Plague Commissioner of Poona, Rand, & one Lt. Ayerst in 1897.  Savarkar & his brother organised Mitra Mela, a secret society, in 1899 which merged with Abhinav Bharat in 1904.

 Nasik Conspiracy Case: In 1909, Anant Kanhare & Ganesh Savarkar shot dead collector Jackson of Nasik with the revolver sent by V.D. Savarkar. Punjab  Among those active here were Lala Lajpat Rai & Ajit Singh who organised the extremist Anjuman-i-Mohisban-i-Watan in Lahore with its journal, Bharat Mata.

 Extremism in the Punjab died down quickly after the Government struck in May 1907 with a ban on political meetings and the deportation of Lajpat Rai & Ajit Singh. After this, Ajit Singh & a few other associates , Lala Hardayal developed into full-scale revolutionary terrorists. Abroad  The need for shelter, the possibility of bringing out revolutionary literature that would be immune from the Press Acts and the quest for arms took Indian revolutionaries abroad.  had started in London in 1905 an Indian Home Rule Society—’’—as a centre for Indian students, a scholarship scheme to bring radical youth from India.  The revolutionaries such as Savarkar and Hardayal became the members of India House.  Madanlal Dhingra of this circle assassinated, the India office bureaucrat Curzon-Wyllie in 1909. Soon London became too dangerous for the revolutionaries, particularly after Savarkar had been extradited in 1910 and transported for life in the Nasik conspiracy case.

 New centres emerged on the continent- Paris and Geneva—from where Madam Bhikaji Cama, a Parsi revolutionary who had developed contacts with French socialists and who brought out Bande Mataram, & Ajit Singh operated.

 And after 1909 when Anglo-German relations deteriorated, Virendranath Chattopadhyaya chose Berlin as his base.

Revolutionary Activity in India during War  The revolutionary activity in this period was concentrated in Punjab & Bengal. The Bengal plans were part of a far-flung conspiracy organized by Rashbehari Bose & Sachin Sanyal in cooperation with returned Ghadrites in Punjab.

 Most Bengal groups were organized under Jatin Mukherji (Bagha Jatin) & planned disruption of railway lines, seizure of Fort William & landing of German arms.

 These plans were ruined due to poor coordination & Bagha died near Balasore in 1915.

 There was a temporary respite in revolutionary activity after the War because the release of prisoners held under the Defence of India Rules cooled down passions a bit; there was an atmosphere of conciliation after Montagu's August 1917 statement & the talk of constitutional reforms; & the coming of Gandhi on the scene with the programme of nonviolent non-cooperation promised new hope. Major Influences

1. Upsurge of working class trade unionism after the War; the revolutionaries wanted to harness the revolutionary potential of the new emergent class for nationalist revolution.

2. Russian Revolution & the success of the young Soviet state in consolidating itself.

3. Newly sprouting communist groups with their emphasis on Marxism, socialism & the proletariat.

4. Journals publishing memoirs and articles extolling the self-sacrifice of revolutionaries, such as Atmasakti, Sarathi & Bijou

5. Novels and books such as Bandi Jiwan by Sachin Sanyal & Maher Dabi by Sharatchandra Chatterjee (a Government ban only enhanced its popularity). Revolutionary Terrorism in Punjab-UP-Bihar  The revolutionary terrorist activity in this region was dominated by the Hindustan Republican Association/Army or HRA

 The HRA was founded in October 1924 in Kanpur by Ramprasad Bismil, Jogesh Chandra Chatterjee & Sachin Sanyal

 It was aimed to organise an armed revolution to overthrow the colonial government

 And establish in its place a Federal Republic of United States of India whose basic principle would be adult franchise

 Later renamed Hindustan Socialist Republican Association or HSRA Kakori Train Robbery (August 1925)  The most important “action” of the HRA was the Kakori train robbery.

 The men held up the 8-Down train at Kakori, an obscure village near Lucknow, & looted its official railway cash.

 Government crackdown after the Kakori robbery led to arrests of many

 Of whom 17 were jailed, four transported for life and four— Bismil, Ashfaqullah, Roshan Singh & Rajendra Lahiri—were hanged.

 Hence, Kakori proved to be a setback.

Hindustan Socialist Republican Association  Determined to overcome the Kakori setback, the younger revolutionaries, inspired by socialist ideas, set out to reorganize Hindustan Republic Association at a historic meeting in the ruins of Ferozshah Kotla in Delhi (September 1928).  H.R.A. was changed into H.S.R.A (Hindustan Socialist Republican Association)  The participants included Bhagat Singh, Sukhdev, Bhagwaticharan Vohra from Punjab & Bejoy Kumar Sinha, Shiv Verma & Jaidev Kapur from UP.  HSRA decided to work under a collective leadership & adopted socialism as its official goal.  Bhagat involved himself in this movement who was influenced by Marxist ideology & idolized Lenin  They wanted to abolish Zamindari System & used slogans like Inquilab Zindabad & Rang de Basanti  They targeted not only British but also Indian capitalists, zamindars, kings & communal forces.  They were influenced by communistic ideologies which had wide appeal among the educated youth. Saunders’ Murder (Lahore, December 1928)  Just when the HSRA revolutionaries had begun to move away from individual heroic action & terrorism, the, death of Sher-i-Punjab Lala Lajpat Rai due to lathi blows received during a lathi- charge on an anti- procession (October 1928) led them once again to take to individual assassination.

 Bhagat Singh, Azad & Rajguru shot dead Saunders, the police official responsible for the lathi charge in Lahore. Bomb in the Central Legislative Assembly (April 1929)  The HSRA leadership now decided to let the people know about its changed objectives & the need for a revolution by the masses.  Bhagat Singh & Batukeshwar Dutt were asked to throw a bomb in the Central Legislative Assembly on April 8, 1929 against the passage of the Public Safety Bill & Trade Disputes Bill aimed at curtailing civil liberties of citizens in general & workers in particular.  The bombs had been deliberately made harmless & were aimed at making ‘the deaf hear’.  The objective was to get arrested and to use the trial court as a forum for propaganda so that people would become familiar with their movement & ideology.  Bhagat Singh, Sukhdev and Rajguru were tried in the Lahore conspiracy case.  In jail, these revolutionaries protested against the horrible conditions through a fast, & demanded honourable & decent treatment as political prisoners.  Jatin Das became the first martyr on the 64th day of his fast.  Azad was involved in a bid to blow up Viceroy Irwin’s train near Delhi in December 1929.  Azad was killed in a police encounter in a park in Allahabad in February 1931.  Bhagat Singh, Sukhdev & Rajguru were hanged on March 23, 1931.

Revolutionary Terrorism in Bengal

 After Das’s death (1925), the Bengal Congress broke up into 2 factions

 One led by M. Sengupta (Anushilan group joined forces with him)  the other led by Subhash Bose (Yugantar group backed him)

 Actions of the re organized groups included an assassination attempt on the notorious Calcutta Police Commissioner, (another man named Day got killed) by Gopinath Saha in 1924. Gopinath Saha was hanged.

 Government, armed with a new ordinance, came down heavily on revolutionaries.

 Many including Subhash Bose were arrested.  Surya Sen had participated in the Non-Cooperation Movement & had become a teacher in the national school in Chittagong.

 He was imprisoned from 1926 to 1928 for revolutionary activity & afterwards continued working in the Congress.

 He was the secretary of the Chittagong District Congress Committee.

 He used to say "Humanism is a special virtue of a revolutionary.“

 He was a lover of poetry and an admirer of Tagore & Qazi Nazrul Islam.

Chittagong Armoury Raid (April 1930)  Sen decided to organise an armed rebellion along with his associates to show that it was possible to challenge the armed might of the mighty British.

 They had planned to occupy 2 main armouries in Chittagong to seize & supply arms to the revolutionaries

 They also aimed to destroy telephone & telegraph lines & to dislocate railway link of Chittagong with the rest of Bengal.

 Sen was arrested in February 1933 & hanged in January 1934.

Aspects of the New Phase of Terrorist Movement in Bengal  There was a large-scale participation of young women especially under Surya Sen.  These women provided shelter, carried messages and fought with guns in hand.  Prominent women revolutionaries in Bengal during this phase included , who died conducting a raid;  Kalpana Dutt who was arrested & tried along Surya Sen & given a life sentence;  Santi Ghosh & Suniti Chandheri, school girls of Comilla, who shot dead the district magistrate. (December 1931);  Bina Das who fired point blank at the Governor while receiving degree at convocation (February 1932).  There was an emphasis on group action aimed at organs of the colonial state, instead of individual action.  The objective was to set an example before the youth & to demoralise bureaucracy.  Some of the earlier Hindu religiosity was shed, & there were no more rituals like oath-taking, & this facilitated participation by Muslims.  Muslims such as Mir Ahmed, Fakir Ahmed Mian & Tunu Mian were in his group. 2001 Who among the following organised the famous ?

(a) Laxmi Sehgal

(b) Surya Sen

(c) Batukeshwar Datta

(d) J.M. Sengupta

Answer b Ideological Rethinking During their last days (late 1920s).  These revolutionaries had started moving away from individual heroic action and terrorism towards mass politics.

 Bismil, during his last days, appealed to the youth to give up pistols & revolvers, not to work in revolutionary conspiracies and instead work in an open movement.

 He urged the youth to strengthen Hindu-Muslim unity, unite all political groups under the leadership of the Congress.

 Bismil affirmed faith in communism and the principle that "every human being has equal rights over the products of nature".

 The famous statement of the revolutionary position is contained in the book The Philosophy of the Bomb written by Bhagwaticharan Vohra.

 Even before his arrest, Bhagat Singh had moved away from belief in terrorism & individual heroic action to Marxism & the belief that a popular broad-based movement alone could lead to a successful revolution. In other words, revolution could only be "by the masses, for the masses".  That is why Bhagat Singh helped establish the Punjab Naujawan Bharat Sabha (1926) as an open wing of revolutionaries to carry out political work among the youth, peasants & workers, and it was to open branches in villages.  Bhagat was fully secular—2 of the 6 rules drafted by him for the Sabha were that its members would have nothing to do with communal bodies & that they would propagate a general feeling of tolerance among people, considering religion to be a matter of personal belief.  Bhagat & Sukhdev organized the Lahore Students' Union for open, legal work among students.  Bhagat & his comrades also realised that a revolution meant organisation & development of a mass movement of the exploited & the suppressed sections by the revolutionary intelligentsia. He used to say, "real revolutionary armies are in villages and factories.“ What then was the need for individual heroic action?

 Firstly, because of the rapidity of change in thinking, effective acquisition of new ideology is a prolonged and historical process.

 Secondly, these young intellectuals faced the classic dilemma of how to mobilise people & recruit them.

 Here, they decided to opt for propaganda by deed, i.e., through individual heroic action & by using courts as a forum for revolutionary propaganda. HISTORY April 2, 2018  Tribal revolts were basically directed to preserve the identities of the tribal.

 About 70 such revolts took place between 1778 & 1947.

 Can be divided into 2 broad categories

 Movement of frontier tribe  Movement of non frontier tribe Causes Of Tribal Revolts

1) Extension of settled agriculture in to the tribal areas led to influx of non-tribals (dikkus) in the tribal areas.

2) Increasing demand for the wood from the early 19th century, 1st for the Royal Navy & then Railways, led to increasing control of government over the forests lands. 3) Colonial administration encouraged influx of Christian missionaries into the tribal areas, which were responsible for bringing about changes in the socio-economic & cultural aspects of the tribal life & the mainstream society.

4) also transformed tribal people’s relationship with the forest. They practiced shifting cultivation. But the colonial government usurped the forest lands & placed restrictions on access to forest products, land & village common lands. It refused to let cultivation shift to new areas.

5) Imposition of land revenue settlement, expansion of agriculture by non-tribals to tribal areas or over forest cover led to erosion of the tribal traditions of joint ownership & increased the socio- economic differentiation in the egalitarian structure of the tribal society. 2011 Which amongst the following provided a common factor for tribal insurrection in India in the 19th century ? (a) Introduction of a new system of land revenue and taxation of tribal products. (b) Influence of foreign religious missionaries in tribal areas. (c) Rise of a large number of money lenders, traders and revenue farmers as middlemen in tribal areas. (d) The complete disruption of the old agrarian order of the tribal com-munities.

The correct option is D.

Characteristics Of The Tribal Uprisings 1) Tribal uprisings were basically against the colonial administration’s effort to destroy their aboriginality, & their traditional thread of a protected social & economic life.

2) Ethnic ties were a basic feature of the tribal rebellions. The rebels saw themselves not as discreet class but as having a tribal identity. At this level the solidarity shown was of a very high order. Fellow tribals were never attacked unless they had collaborated with the enemy. 3) These uprisings were localized and isolated, & lacked any modern feeling of nationalism. The movements were mostly violent & frequent. 4) Often, religious & charismatic leaders – messiahs – emerged at this stage & promised divine intervention and an end to their suffering at the hands of the outsiders, & asked their fellow tribals to rise and rebels against foreign authority. Most of these leaders claimed to derive their authority from God.

5) The warfare between the tribal rebels & the British armed forces was totally unequal. On one side were drilled regiments armed with the latest & on other were men & women fighting in roving bands armed with primitive weapons, believing in the magical powers of their commanders.

These varied grievances reached their climax in the revolt of 1857. Santhal rebellion  On 30 June 1855, 2 Santhal rebel leaders, Sidhu & Kanhu Murmu, mobilized ten thousand Santhals & declared a rebellion against British colonists.

 The causes of the Uprising were primarily economic and it was directed against the money-lenders and their protectors, the British authorities.

 They proclaimed a government of their own in the area between Bhagalpur & Rajmahal.  Soon after the declaration the Santhals took to arms. The open rebellion caught the British Government in surprise. Munda Rebellion / Ulgulan uprising  Led by a great Munda leader Birsa Munda.

 The main rebellion took place in the region south of Ranchi in 1899-1900.

 The rebellion aimed to drive away the British & establish Munda Raj or Munda rule.

 Birsa Munda strongly protested against non-tribals occupying tribal lands .  Birsa Munda started the revolt in the Chotanagpur region . They attacked British officials missionaries and police stations.

 However the British captured Birsa Munda & suppressed the rebels. Chuar Uprisings

 Chuar Uprisings by the Chuar aboriginal tribesmen; against rise in demands & economic deprivation by the British.

 Leader of this revolt was Durjan Singh, a displaced Zamindar who along with his followers created havoc.

 Some historians want this revolt to be named as ‘Freedom Struggle of the Jangal Mahal’. Others

 The Kutch Rebellion, led by its chiefs, lasted in one form or another from 1816-32.

 Kol Uprising of 1831: The Kol tribesmen of Chota Nagpur led by Buddho Bhagat rebelled against the British for imposing on them outsiders as money-lenders & landlords.

 Thousands of Kols perished before British authority could be re imposed. Wahabi Movement The most serious and well-planned challenge to British supremacy in India from 1830's to 1860's (before 1857 rebellion)

Led by Syed Ahmed of Rae Bareilly

Influenced by the teachings of Abdul Wahab & Shah Waliullah

Actively supported the revolt of 1857

Gave the slogan of ‘Jihad’.

Declared India as ‘dar-ul-harb’ meaning land of infidels – to be converted to ‘dar-ul- Islam’ meaning land of peace. Sannyasi Rebellion  The Sannyasi Rebellion or Sannyasi Revolt , The Monks' Rebellion) were the activities of sannyasis & fakirs ( Hindu & Muslim ascetics, respectively) in Bengal, India in the late . It is also known as the Fakir-Sannyasi Rebellion which took place around Murshidabad & Jalpaiguri.

 Historians have not only debated what events constitute the rebellion, but have also varied on the significance of the rebellion in Indian history.

 While some refer to it as an early war for India's independence from foreign rule, since the right to collect tax had been given to the Company after Plassey , others categorize it as acts of violent banditry following the depopulation of the province, post the Bengal famine of 1770.  The Vellore mutiny on 10 July 1806 was the first instance of a large-scale & violent mutiny by Indian sepoys against the EIC, predating the 1857 revolt.

 The immediate causes of the mutiny revolved mainly around resentment felt towards changes in the dress code, introduced in November 1805.

 Hindus were prohibited from wearing religious marks on their foreheads & Muslims were required to shave their beards & trim their moustaches.  In addition Commander-in-Chief of the Madras Army, ordered the wearing of a round hat resembling that associated at the time with both Europeans in general & with Indian converts to Christianity. Kuka Movement  There has been a doubtful history about the initial days of Kuka Movement. There are two names associated with the start of this movement i.e. Baba Balak Singh & Bhagat Jawar (or Jawahar) Mal.  Balak Singh started preaching very early in his life and the objective of his preachings was to uphold the religious purity of Sikhism.  Some sources say that Balak Singh himself was a disciple of Bhagat Jawar Mal. Bhagat Jawar Mal was also known as Sian Sahib. Permanent settlement 1793  Bengal, Bihar, Orissa, Eastern UP & Northern Tamil Nadu  57 % of Agricultural land in British Provinces  Revenue amount was fixed at the beginning & remained the same permanently.  Lord Cornwallis , John Shore  Company ‘outsourced’ the revenue collection work to Zamindars  All the land belonged to the state and was thus at their disposal.  British designated zamindars (local tax collectors) as owners of the land in their district.  This system was adopted in several other forms such as Jagirdari, Inamdari, etc.  Dual Ownership: Zamindar was treated as owner in relation to peasant but in relation with state, state is considered as owner

Impact  Income to be collected at 1790’s base rate.  This controlled the fluctuations in collections faced by company from 1772  Company Peasant to pay rent to Zamindar.  Zamindar to pay tax to state  Transfer of Land became regular feature.  Absentee landlordism resulted.  Because they never had traditional bonds with villagers, only sent mediators to collect tax.  Exploitation increased. Led to many revolts. Hence British didn’t implement it in other parts of India.  Increase of Rural Indebtedness 2011 The tendency for increased litigation was visible after the introduction of the land settlement system of lord Cornwallis in 1793. The reason for this is normally traced to which of the following provisions ? (a) Making zamindar’s position stronger vis-à-vis the ryot. (b) Making an overlord of zamindars. (c) Making judicial system more efficient. (d) None of the (a), (b) and (c) above. The reasons given in the options don't seem to fit in the criteria of the reforms of Cornwallis so Option D is correct. Ryotwari settlement  1805  The ryotwari system is associated with the name of Sir Thomas Munro, who was appointed Governor of Madras in May 1820.  Madras, Bombay Assam & central province of India  38 % of Agricultural land in British Provinces

 To remove negatives of Permanent settlement’s Intermediaries & absence of Zamindars in Madras region  Ryotwari system had its origin in the Baramahal & Salem which were ceded to the company in 1792 & 1st introduced by Read & his assistants.

 State officials to collect tax directly from peasants.

 Fixed once every 20 yrs. 50%of produce in most of the times.

 But depending on assess, which didn’t take his investment costs. Features  government claimed the property rights to all the land, but allotted it to the cultivators on the condition that they pay taxes. In other words, It established a direct relation between the landholder and the government.

 Farmers could use, sell, mortgage, bequeath & lease the land as long as they paid their taxes. In other words Ryotwari system gave a proprietary rights upon the landholders.

 If they did not pay taxes, they were evicted

 taxes were only fixed in a temporary settlement for a period of 30 years & then revised.  government had retained the right to enhance land revenue whenever it wanted

 Provided measures for revenue relief during famines but they were seldom applied in real life situation. With reference to Ryotwari Settlement, consider the following statements:

1. The rent was paid directly by the peasants to the Government.

2. The Government gave Pattas to the Ryots.

3. The lands were surveyed and assessed before being taxed.

Which of the statements given above is/are correct? a) 1 only b) 1 and 2 only c) 1, 2 and 3 d) None Answer: C 2017 Who among the following was/were associated with the introduction of Ryotwari Settlement in India during the British rule?

1. Lord Cornwallis 2. Alexander Read 3. Thomas Munro

Select the correct answer using the code given below.

(a) 1 only

(b) 1 and 3 only (c) 2 and 3 only d) 1, 2 and 3 Answer c Mahalwari Settlement  The government of William Bentinck later made a thorough review of the scheme of 1822, by which the Mahalwari system was introduced.  Punjab region Gangetic valley north-west provinces, parts of central India  5 % of Agricultural land in British Provinces  Not as controversial as above two  The word Mahalwari is derived from the term Mahal, referring to a neighbourhood or quarter. Under this system the unit for revenue settlement is the village.  The village lands belong jointly to the village community technically called the body of co-shares. The body of co-shares is jointly responsible for the payment of land revenue, though individual responsibility is always there. If any co-sharer abandons his land, it is taken over by the village community as a whole. The community is the owner of village common land area, including the forestland, pastures etc.  Company ‘outsourced’ revenue collection work to Village community itself. – Technically village headman (Lambardar) was made responsible for tax collection  North West Provinces initially had Permanent settlement but transformed to Mahalwari system by Holt Mackenzie.(1822) In North India & Punjab, joint land rights on the village were common. So, British decided to utilize this utilize this traditional structure in a new form known as Mahalwari system. Features  unit of assessment was the village.  taxation was imposed on the village community since it had the rights over land.  The village community had to distribute these tax collection targets among the cultivators  Each individual farmer contributed his share in the revenue.  Everyone was thus liable for the others’ arrears.  Farmers had right to sell or mortgage their property.  The village community did not necessarily mean entire village population. It was a group of elders, notables of high castes.

 The lambardar, collected the amounts & gave to the British

 British periodically revised tax rates. Pagal Panthi, 1830s-40s  Reason: Zamindari Oppression

 Area: North Bengal, Hajong and Garo tribes.

 Leader: Karam Shah and his son Tipu

 Result: Initially British agreed to Pagal Panthi demand, made arrangement to protect the cultivators from Zamindar

 But later, launched massive military operation to suppress Pagal Panthis Indigo Revolt (1859-60)  In Bengal, the indigo planters, nearly all Europeans, exploited the local peasants by forcing them to grow indigo on their lands instead of the more paying crops like rice & without paying right price. The planters forced the peasants to take advance sums & enter into fraudulent contracts which were then used against the peasants.

 The anger of the peasants exploded in 1859 when, led by Digambar Biswas & Bishnu Biswas of Nadia, they decided not to grow indigo under duress & resisted the physical pressure of the planters & their lathiyals (retainers) backed by police & the courts. Continued  They also organised a counter force against the planters' attacks. Refused to grow Indigo. If police tried to intervene, they were attacked.  Gradually, they learned to use the legal machinery & initiated legal action supported by fund collection.

 The Government appointed an Indigo Commission to inquire into the problem of indigo cultivation. Based on its recommendations, the Government issued a notification in November 1860 that the ryots could not be compelled to grow indigo & that it would ensure that all disputes were settled by legal means.

 But, the planters were already closing down factories & indigo cultivation was virtually wiped out from Bengal by the end of 1860. Bengali Intelligentsia  The Bengali intelligentsia played a significant role by supporting the peasants' cause through newspaper campaigns, organisation of mass meetings, preparing memoranda on peasants' grievances & supporting them in legal battles.

 Harish Chandra Mukherji

 editor of Hindu patriot.

 published reports on indigo campaign, organized mass meetings etc.

 Din Bandhu Mitra

 wrote a play ‘Neel Darpan’ to portray the oppression of indigo farmers.

 Also got support from press, missionaries and Muslims. Pabna Agrarian Leagues  During the 1870s & 1880s, large parts of Eastern Bengal(Pabna is a jute growing district) witnessed agrarian unrest caused by oppressive practices of the zamindars.  Zamindars resorted to enhanced rents beyond legal limits through a variety of cesses (Abwab) & prevented tenants from acquiring occupancy rights under Act X of 1859.  To achieve their ends, the zamindars resorted to forcible evictions, seizure of cattle & crops and prolonged, costly litigation in courts where the poor peasant found himself at a disadvantage.  Leaders: Ishwar Chandra Roy, Shambhu Pal, Khoodi Mollah.  This league provided a sound platform to the peasants at a time when there was no kisan sabha or any political party to organize the peasants.  Hindu Muslim unity. Pabna Agrarian Leagues  The league organised a rent strike - the ryots refused to pay the enhanced rents challenging the zamindars in the courts. Funds were raised by ryots to fight the court cases. The struggles spread throughout Pabna & to other districts of East Bengal.  The main form of struggle was that of legal resistance; there was very little violence.  No zamindar or agent was killed / seriously injured. Very few houses looted, very few police stations attacked.  In 1885, the Bengal Tenancy Act was passed. But this act did not fully protect farmers from the zamindari oppression. Even non-cultivators were given occupancy right. It gave rise to a powerful jotedar groups. Later some of the Jotedars became as exploitative as the zamindars.  Again, a number of young Indian intellectuals supported the peasants‘ cause.  These included Bankim Chandra Chatterjee, R.C. Dutt & the Indian Association under SN Banerjee. Deccan Riots  Ryots of Deccan region of western India suffered heavy taxation under Ryotwari & had to pay land Revenue even during bad seasons  Here again the peasants found themselves trapped in a vicious network with the moneylender as the exploiter & the main beneficiary. These moneylenders were mostly outsiders— Marwaris or Gujaratis.  1860: American civil war led to a boom in demand of cotton export.  The conditions had worsened due to a crash in cotton prices after the end of the American civil war in 1864,the Government's decision to raise the land revenue by 50% in 1867, & a succession of bad harvests.  Also the farmers had taken loans from moneylenders, but now they couldn’t repay so moneylenders took away their land, cattle, jewellery & property.  In 1874, the growing tension between the moneylenders, & the peasants resulted in a social boycott movement organised by ryots against the "outsider" money lenders. Deccan Riots  This social boycott spread rapidly to the villages of Poona, , Sholapur & Satara.  Violence was used only when the moneylenders refused to hand over the documents. villagers led by traditional headmen (Patels)  The debt bonds and deeds were seized and publicly burnt.  The Government succeeded in repressing the movement by initially resorting to use of police force & arrest but later appointed a commission, passed Deccan Agriculturists Relief Act in 1879 & on the operation of Civil Procedure Code.  Now the peasants could not be arrested & sent to jail if they failed to pay their debts.  The modern nationalist intelligentsia of Maharashtra supported the peasants' cause.  Later got support from Poona Sarvajanik Sabha led by Justice Ranade Eka Movement 1920s  Towards the end of 1921, peasant discontent resurfaced in some northern districts of the United Provinces—Hardoi, Bahraich, Sitapur.

 The issues involved were: (i) high rents-50 % higher than the recorded rates; (ii) oppression of thikadars in charge of revenue collection; & (ii) practice of share-rents.  The meetings of the Eka or the Unity Movement involved a symbolic religious ritual in which the assembled peasants vowed that they would

 pay only the recorded rent but would pay it on time;  not leave when evicted;  refuse to do forced labor;  give no help to criminals;  a tip in  abide by panchayat decisions. Revolt  The were the Muslim tenants inhabiting the region where most of the landlords were Hindus.  Mappilas had expressed their resentment against the oppression of the landlords during the 19th century also.  Their grievances centered around lack of security of tenure, high rents, renewal fees & other oppressive exactions.  The Mappila tenants were particularly encouraged by the demand of the local Congress body for a government legislation regulating tenant landlord relations.  Soon, the Mappila movement merged with the ongoing Khilafat agitation.  The leaders of the Khilafat-Non-Cooperation Movement like Gandhi, Shaukat Ali & Azad addressed Mappila meetings.  After the arrest of national leaders, the leadership passed into the hands of local Mappila leaders.

Bardoli  Bardoli taluqa in had witnessed intense politicization after the coming of Gandhi on the national political scene.

 The movement sparked off in January 1926 when the authorities decided to increase the land revenue by 22 per cent.

 Congress leaders were quick to protest & a Bardoli Inquiry Committee was set up to go into the issue.

 The committee found the revenue hike to be unjustified.

 In February 1926, was called to lead the movement

 The women of Bardoli gave him the title of "Sardar". Sardar Patel persuaded the farmers:

 not to pay Revenue, required them to take oath in the name of their respective Hindu/Muslim gods.

 social boycott of anyone who paid revenue.

 Resist eviction and Jabti (Confiscation). houses and migrate to Baroda State

 social upliftment of Kaliparaj caste- who worked as landless laborers.

 Under Patel, the Bardoli peasants resolved to refuse payments of the revised assessment until the Government appointed an independent tribunal or accepted the current amount as full payment.

Bardoli Satyagraha  To organise the movement Patel set up 13 chhavanis or workers' camps in the taluqa.

Patrika was brought out to mobilise public opinion.

 An intelligence wing was set up to make sure all the tenants followed the movement's resolutions.

 Special emphasis was placed on the mobilisation of women.

 K.M. Munshi & Lalji Naranji resigned from the Bombay Legislative Council in support of the movement.

 By August 1928, massive tension had built up in the area.

 Bombay communists and railway workers also threatened strikes and boycotts. Bardoli Satyagraha  Gandhi reached Bardoli to stand by in case of any emergency. The Government was looking for a graceful withdrawal now.

 It set the condition that first the enhanced rent be, paid by all the occupants (not actually done).

 Government setup Maxwell-Broomfield commission.

 Commission went into the whole affair and found the revenue hike to be unjustified & recommended a rise of 6.03 per cent only.

 Returned confiscated land back to farmers.

All India Kisan Sabha, 1936  was set up in Lucknow

 It was initially known as All India Peasant Union

 1st President was – Swami & the 1st secretary was N.G. Ranga

 Lot of peasant organisations were found at provincial level, district level and micro level which came together and formed Kisan Sabha

Tebhaga, Bengal, 1946  In this region: Rich farmers (Jotedars) leased the farms to sharecroppers (Bargadar)  Flout Commission had recommended that Bargadar should get 2/3 of crop produce & jotedar (the landlord) should get 1/3rd of crop produce.  Tebhaga movement aimed to implement this recommendation through mass struggle.  Suharwardy’s Government introduced Bargardari Bill. But overall, Limited success:  Brutal police suppression.  Led by Bengal Provincial Kisan Sabha & communist groups  Lower stratum of tenants such as bargardars (share croppers), adhiars & poor peasants, tea plantation workers etc.  Against zamindars, rich farmers (Jotedars), moneylenders, traders, local bureaucrats  difference of opinion  tribal elements wanted more militant protest  poor and middle level farmer support declined  urban professional did not support (Because many of them had given their village land to Bargadars)  Riots started in Calcutta, demand for partition. 2013 The demand for the Tebhaga Peasant Movement in Bengal was for a) the reduction of the share of the landlords from one-half of the crop to one-third b) the grant of ownership of land to peasants as they were the actual cultivators of the land c) the uprooting of Zamindari system and the end of serfdom d) writing off all peasant debts

It was the Tebhaga struggle in Bengal that held the limelight. in 1946 the sharecroppers of Bengal began to assert that they would no longer pay a half share of their crops to Jotedars but only 1/3rd and that before division, the crop would be stored in their khamars (Godowns) and not that of the Jotedars. Answer A. HISTORY April 2, 2018 Press  In 1550 first press was established by Portuguese.  1780: James Augustus Hicky started the 1st newspaper weekly in India called Bengal Gazette .  This paper attacked both & Chief Justice  Bombay Courier & Bombay Gazette merged to form Bombay Herald in 1792.  In 1837 Syed-ul-Akbhar a weekly in Urdu was published.  1838: Dilli Akbhar was published.  1862: Indian Mirror was started . Initially the editor was Devendranath Tagore followed by Keshavchandra Sen & Narendranath Sen.  1861: Bombay Times, Bombay Standard, Bombay Courier & The Telegraph merged together to form Times of India. Its editor was Robert Knight.It was established by Carey, Ward & Marshman in 1818.Initially it was monthly but latter weekly.  1890: Statesman & Friend of India merged to become Statesman.  1878: Hindu was started from Madras by G.Subramanium Aiyar as a weekly. Later it was made triweekly in 1883 when Kusturiangar became its editor. In 1889 it was made a daily. Censorship of press Act 1799 by Lord Wellesley  Every newspaper should print the names of printer, editor & proprietor.

 Before printing any material it should be submitted to the secretary of Censorship.

 In 1802 the act was extended to all magazines pamphlets journals books etc.

 This Act was abolished by Hastings. Licensing regulation Act 1823 by John Adam( acting GG)

 Every publisher should get a license from the government, defaulters would be fined Rs 400 & the press would be ceased by the government.

 Magistrates were appointed to seal the press.

 Charles Metcalf abolished the Act.

 Due his liberal policy Metcalf is called the liberator of the Indian Press,

Vernacular Press Act IX 1878  Vernacular press criticized British rule.  Therefore British Government came down heavily on vernacular press.  Magistrates were authorized to ask any publisher of newspaper to give assurance of not publishing anything threatening peace & security.

 No exemption for any vernacular paper.  This law was not applicable to English Press.

 Passed by lord Lytton.

 It was repealed by Lord Ripon in 1882.  Som Prakash stopped by this act. Continued

 1883 Surendranath Banerjea became the first Indian journalist to be imprisoned.

 In an angry editorial in The Bengalee Banerjea had criticised a judge of for being insensitive to the religious sentiments of Bengalis in one of his judgements.

The Newspapers (Incitement of Offences) Act,1908  If any newspaper is found inciting the offences, crimes of murder & any act of violence, this act can put an end to the existing license of that newspaper.

 District Magistrate was empowered to confiscate the printing press where a newspaper containing an incitement to violence is printed.

 The police was also empowered to attach the printing press and issue warrants.

 The matter can be taken in appeal within 15 days.

Indian Press Act 1910  This act empowered the magistrate to require a deposit from the keepers of news printing presses & publishers of newspapers.

 The local government could even demand a security deposit of Rs.500 min to Rs.5k max.  It could be forfeited & it's registration cancelled owing to the printing of any objectionable material.  Fresh registration was allowed for a deposit of not less than Rs. 1k & not more than Rs.10k.  It was a huge money which would be generally beyond anybody’s affordability.  It was imposed due to seditious publication & enlarged to include writing against the Indian Princes, judges, executive officers & public servants.

Press committee 1921

 Recommended abolition of act of 1908 and 1910  Sapru committee

Indian Press Emergency Act 1931  The main feature of the act was that if any publication directly or indirectly admired the offence of any person real or fictitious the government could seize the press.

 The impact of the Gandhian Movement provoked the government to issue a fresh press ordinance in 1930 with the provision of Press Act 1910.

 By the provisions of this act the provincial Government were given sweeping powers to supress the press. In 1932 the provisions of this Act were further amplified in the form of the criminal Amendment Act. Press Regulating Act 1942

 Registration of journalists was made mandatory.

 Limitations were imposed on the messages regarding civil disturbances.

 Prohibition of news was imposed regarding acts of sabotage.

 Limitations on headlines and space given to news on disturbances.

 Government had the authority on arbitrary censorship.

Press inquiry committee 1947

 The committee abolished the emergency act of 1931.

In favor of censorship :-  Welleselly,  Lord Minto-II,  Lord Adams,  Lord Canning,  Lord Lytton,  Lord Elphinston,  Sir Munro

 In favor of freedom of press :-

 Lord Hastings,  Charles Metcalf,  Macaule,  Rippon Name of Paper/Journal Foundation Year Founder Facts Bengal Gazette 1780 James Augustus Hicky (Irishman) Indian Gazette 1787 Derozio associated with it st Madras courier 1784 1 paper from Madras Weekly Bombay Herald 1792 1st paper from Bombay Indian Herald 1795 R.Williams (Englishman) & Published by Humphreys st Digdarshan 1818 Marshman from Serampore. 1 Bengali monthly Calcutta Journal 1818 J.S Buckingham Bengal Gazette 1818 Harishchandra Ray 1st Bengali Newspaper Sambad Kaumadi 1821 Raja Ram Mohan Roy st Mirat-ul-Akhbar 1822 1 weekly journal in Persian Jam-i- Jahan Numah 1822 An English firm First paper in Urdu Banga-Dutta 1822 Weekly in 4 languages English, Bengali, Persian, Hindi Bombay Samachar 1822 East Indian 19th century Henry Vivian Derozio Bombay Times 1838 Foundation laid by Thomas Knight started by Times of India after 1861 Thomas Bennett Rast Goftar 1851 Dadabhai Naoroji Gujarati fortnightly Hindu Patriot 1853 Girishchandra Ghosh (later Harishchandra Mukerji became owner-cum-editor ) Somaprakasha 1858 Dwarkanath Vidyanath st Indian Mirror Early 1862 Dwarkanath Tagore 1 Indian Daily paper in English Bengalee 1862 Girishchandra Gosh Taken Over by S.N Banerjea in 1879 Pioneer 1865 Started from Allahabad st Madras Mail 1868 1 evening paper in India Amrita Bazar Patrika 1868 Sisir kumar & Motilal Ghosh In Bengali & later an English daily Bangadarshana 1873 Bankim Chandra Chatterji Indian Statesman 1875 Robert Knight Later, The Statesman The Hindu 1878 G.S Aiyer, Virarghavachari & Subba Rao Pandit Tribune 1881 Dayal Singh Majeetia Kesari & Maharatta 1881 Tilak, Chiplunkar, Agarkar before Tilak Agarkar & prof. Kelkar were the editors respectively Swadesh mitram G.S Aiyer Paridersik 1886 Yugantar 1906 Barindra Kumar Ghosh & Bhupendranath Dutta Sandhya 1906 Brahamabandhab Upadhya Kal 1906 Indian Sociologist London Shyamji Krishna varma Bande Mataram Paris Madam Bhikaji Cama Talavar Berlin Verendranath Chattopadhya Free Hindustan Vancouver Tarakanth Das Bombay Chroicle 1913 Pherozeshah Mehta, Editor B.G. Horniman (Englishman ) The Hindustan Times 1920 K.M. Panikkar as part of the Akali Dal Movement The Milap 1923 K.M Chand Leader Madan Mohan Malviya Kirti 1926 Santosh Singh Bahishkrit Bharat 1927 B.R Ambedkar Kudi Arasu 1910 E.V Ramaswamy Naicker Kranti 1927 S.S Mirajkar, K.N Joglekar, S.V Ghate Langal and Ganbani 1927 Gopu Chakravarti and Dharani Goswami National Herald 1938 Young India, Harijan Nation Gopalakrishna Gokhale Bengali Surendernath Banerjee Som Prakash Ishwar Chandra Vidyasagar Karmyogi Aurobindo Ghosh Zamindar, Lahore Zafar Ali Khan New India, commonweal Annie Besant Satpatra Series Gopal Hari Deshmukh Din Mitra Mukundrao Patil , Din Bandhu Bhaskar Rao Jadhav Darpan Bal Shastri Jambekar Prabhudha Bharat,Udbodhava Vivekananda HISTORY March 21, 2018 Growth of Indian nationalism was result of following:

1. Worldwide concept of nationalism & right of self determination initiated by French revolution

2. Indian Renaissance

3. Modernization initiated by British, western thoughts 4. Reaction to British colonial & imperialistic policies 5. Rise of Middle class intelligentsia 6. Unification of the country done under British rule (political, administrative, economic) hitherto cultural unity from Himalayas in north to Cape Comorin in south, from Assam in east to Khyber in west 7. Development of means of transport & communication 8. Press Reactionary policies of Govt.  1876 Lord Lytton reduced age of ICS exam from 21 to 19 years

 1877 grand Delhi Durbar when country was suffering from famine

 1878 Vernacular press act

 1878 Indian Arms act provoked opposition in the country

 1883-84 Ripon & Ilbert bill controversy, he wanted to abolish judicial disqualification on racial basis through this bill & give the Indian members of covenanted civil service same powers as their European colleagues, but Ripon was forced to modify bill under severe opposition from European community which further increased hate among Indian for foreigners 2013 The Ilbert Bill controversy was related to the a) imposition restrictions the Indians of certain to carry arms by Indians b) imposition of restrictions on newspapers and magazines published in Indian languages c) removal of disqualifications imposed on the Indian magistrates with regard to the trial of the Europeans d) removal of a duty on imported cotton cloth

Growth Of Modern Political Ideas & Political Associations

 Raja Ram Mohun Roy was the pioneer

 But the task to organise associations was left to his supporters Pre-Congress Associations  Bangabhasha Prakasika Sabha was formed in 1836 by associates of RM Roy.

 1838: The Zamindari Association, more popularly known as the 'Landholders‘ Society', was founded to safeguard the interests of the landlords.

 1839: British India Society founded by Mr. Adams in london

 The Bengal British India Society was founded in 1843.

 1851: Both the Landholders' Society & the Bengal British India Society merged into the British Indian Association as they had not been faring well.

 1866 Dadabhai Naoroji in London set up East India Association to influence English men there for Indian welfare Continued  1875 Sisir Kumar Ghosh started Indian league to instill nationalism among people

 1876 Indian Association superseded Indian league by Surendernath Banerjee & Anand Mohan Bose to unify people on common political ground. Wanted support of not only middle class but also the masses so kept their subscription to Rupees 5 instead of Rupees 50 kept by British Indian Association

 1867 Ranade’s Poona Sarvajanik Sabha

 1885 Badruddin Tyabji, Pherojshah Mehta, K.T.Telang formed Association

 1884 Subramanyam Aiyer, Anand Charlu founded Madras Mahajan Sabha 2017 Consider the following pairs:

1. Radhakanta Deb — First President of the British Indian Association

2. Gazulu Lakshminarasu Chetty — Founder of the Madras Mahajana Sabha

3. Surendranath Banerjee — Founder of the Indian Association

Which of the above pairs is/are correctly matched?

(a) 1 only (b) 1 and 3 only (c) 2 and 3 only (d) 1, 2 and 3 INC  A.O Hume retired ICS officer organized 1st session of INC in Dec 1885 in Bombay  Prelude to it SN Banerjee & AM Bose(president) had conducted All-India National conference in December 1883 organized an National Conference and given a call for another one in December 1885. SN Banerjea, could not for that reason attend the founding session of the Congress in 1885).  1st session of INC was attended by 72 delegates presided over by W C Bannerjee , INC annual meetings took place in December every year after it.  1st organized expression of Indian nationalism on an all-India scale.

 But why was it founded by these 72 & why at that time?

 Lord Dufferin considered INC to be “safety Valve” but earlier congress leaders used Hume as “lightening conductor”  In his Young India published in 1916, Lala Lajpat Rai

 More than a quarter century later, R. Palme Dutt’s authoritative work India Today made the myth of the safety-valve a staple of left-wing opinion

 1939: M.S. Golwalkar, had also found the theory handy in attacking the Congress for its secularism and, therefore, anti-nationalism in his pamphlet We

 The liberal C.F. Andrews & Girija Mukherji fully accepted the theory in their work, The Rise and Growth of the Congress in India published in 1938. They were happy with it because it had helped avoid ‘useless bloodshed. Moderate Phase Of Congress (1885 – 1905) Demands  Expansion of the Legislative Council

 Inclusion of Indians in the Executive Council

 Reduction of taxes

 Checking Drain of Wealth (1st mentioned by Dadabhai in “Poverty & Un British Rule in India”)

 Reduction on expenditure on Army & utilizing the same amount for the public

 Indianisation of Civil Services Exam  To conduct the exam in India also  Increase in the maximum age limit  Exam syllabus common to all Objectives  Create a consciousness & national spirit & then educate & unite people on common political questions

 Persuade British Government & British public opinion to introduce reforms in India.

 For this purpose, a British committee of the Congress was established in London in 1899 which had India as its organ. Dadabhai spent a portion of his life & income campaigning for India's case abroad.

 1890: It was decided to hold a session of the Congress in London in 1892, but owing to the British elections of 1891the proposal was postponed & never revived later.

 Moderate leaders believed that political connections with Britain were in India's interest at that stage of history & that the time was not ripe for a direct challenge to the British rule. Reasons for not demanding Swaraj (Independence) or (Complete Independence)

 Leaders of INC during this phase had faith in the British system

 The movement was in its early stages & was a restricted in nature

 It was not a mass movement Methodology or Style of Functioning  Peaceful assembly in the last 3 days of the year

 Election of President by consensus and voting

 Discussion of Important National & local issues

 Passing and adoption of resolution through consensus & voting

 Resolution presented to the government in form of petitions or request

 The style of writing these petitions was humble & soft

 This was an approved & Legal method as prescribed by the government Achievements  These 20 years resulted in creation of an all India Political Base

 Support for the Party increased from 72 in 1885 to few lakh in 1905

 Politicization of Local issues as National Issues gave a feeling of Nationalism

 Various issues were discussed & the government was put under pressure resulting in reforms like 1892 Indian Council Act that allowed Indians to enter Legislative Assembly

 It succeeded in creating a wide national awakening, in arousing among the people the feeling that they belonged to one common nation-the Indian nation  It trained people in the art of political work, popularized among them the ideas of democracy & nationalism, propagated among them a modern outlook & exposed before them the evil results of British rule. Continued  Most of all, it made people recognize the economic content & character of British -that Britain was making India a supplier of raw materials, a market for British manufacture, & a field for investment of British capital.

 Exploitative nature of colonial power was revealed , “Drain theory of wealth” was brought in public

 It established the political truth that India must be ruled in the interests of the Indians. It made the issue of nationalism a dominant one in Indian life.

 While its weaknesses were to be removed by the succeeding generations, its achievements were to serve as a base for a more vigorous national movement in later years.  In 1890, Kadambiny Ganguly the 1st woman graduate of Calcutta University addressed the Congress session, which symbolised the commitment of the freedom struggle to give the women of India their due status in national life. 2012 Consider the following statements :

The most effective contribution made by Dada Bhai Naoroji to the cause of Indian National Movement was that he

1. exposed the economic exploitation of India by the British 2. interpreted the ancient Indian texts and restored the self-confidence of Indians 3. stressed the need for eradication of all the social evils before anything else

Which of the statements given above is/are correct? (a) 1 only (b) 2 and 3 only (c) 1 and 3 only (d) 1, 2 and 3 Indian Councils Act 1861

 It made a beginning of representative institutions by associating Indians with the law-making process.

 It thus provided that the viceroy should nominate some Indians as non-official members of his expanded council.

 In 1862, Lord Canning, nominated 3 Indians to his legislative council—the Raja of Benaras, the Maharaja of Patiala & Sir Dinkar Rao. Constitutional Reforms and Propaganda in Legislature  Legislative councils in India had no real official power till 1920.  Yet, work done in them by the nationalists helped the growth of the national movement.  Imperial Legislative Council constituted by the Indian Councils Act (1861) was an impotent body designed to disguise official measures as having been passed by a representative body.  Indian members were few in number—30 years from 1862-92 only 45 Indians were nominated to it most of them being wealthy, landed & with loyalist interests.  Only a handful of political figures and intellectuals such as Ahmed Khan, Kristodas Pal, V.N. Mandlik, K.L. Nulkar & Rashbehari Ghosh were nominated.  From 1885 to 1892, the nationalist demands for constitutional reforms were 1. expansion of councils—i.e., greater participation of Indians in councils, 2. reform of councils—i.e., more powers to councils, especially greater control over finances.  The early nationalists ’long-term objective was of a democratic self-government.  Their demands for constitutional reforms were conceded in 1892 Indian Councils Act 1892  Number of additional members in Imperial Legislative Councils & the Provincial Legislative Councils was raised.  In Imperial Legislative Council, now the governor-general could have 10 to 16 non officials (instead of 6 to 10 previously).  Some of these additional (nonofficial) members could be indirectly elected both in the Central & provincial legislative councils. Thus an element of election was introduced for the 1st time.  The word “election” was, however, not used in the act. It was described as nomination made on the recommendation of certain bodies like in Central Legislative Council by viceroy on the recommendation of the provincial legislative councils & Bengal Chamber of Commerce, & that of the Provincial legislative councils by Governors on the recommendation of the district boards, municipalities, universities, trade associations, zamindars etc.  It increased the functions of legislative councils  Budget could be discussed.  Questions could be asked. Limitations  The officials retained their majority in the council, thus leaving ineffective the non- official voice.

 The 'reformed' Imperial Legislative Council met, during its tenure till 1909, on an average for only 13 days in a year, & the number of unofficial Indian members present was only 5 out of 24.

 The budget could not be voted upon, nor could any amendments be made to it.

 Supplementaries could not be asked, nor could answers be discussed. Reaction  These reforms were severely criticised at Congress sessions, where the nationalists made no secret of their dissatisfaction with them.

 Now, they demanded (i) a majority of elected Indians, & (ii) control over the budget i.e., the 'power to vote upon & amend the budget.

 They gave the slogan—"No taxation without representation'.

 Gradually, the scope of constitutional demands was widened & Dadabhai (1904), Gokhale (1905) & Tilak (1906) demanded self government like the self-governing of Canada & Australia.

 Also, leaders like Pherozshah Mehta & Gokhale put government policies & proposals to severe criticism. Pherozeshah Mehta  Born 1845 in Bombay

 Pherozeshah Mehta came under Dadabhai Naoroji’s influence while studying law in London during the 1860s.

 He was one of the founders of the INC.

 From about the middle of the 1890s till his death in 1915 he was a dominant figure in the Congress & was often accused of exercising autocratic authority over it.

 He was a powerful debater & his speeches were marked by boldness, lucidity, incisiveness, a ready wit & quick repartee, & a certain literary quality. Gopal Krishna Gokhale  1901: Mehta retired from Imperial Legislative Council due to bad health.  He got elected in his place 35-year-old Gokhale, who had already made his mark as the Secretary of the Poona Sarvajanik Sabha & the editor of the Sudharak.  In 1897, as a witness in London before the Royal Commission on Expenditure in India, Gokhale had outshone veterans like SN Banerjea, Wacha, G. Subramaniya Iyer & Dadabhai Naoroji.  His very 1st budget speech on 26 March 1902 established him as the greatest parliamentarian that India has produced.  An outstanding intellectual who had been trained in Indian economics by Ranade & G.V. Joshi.  He was no orator. He did not use strong & forceful language as Tilak, Dadabhai & R.C. Dutt did. Nor did he take recourse, as Mehta did, to humour, irony & courteous, sarcasm. As a speaker he was gentle, reasonable, courteous, non- flamboyant & lucid. He relied primarily upon detailed knowledge & the careful data. Consequently, while his speeches did not entertain or hurt, they gradually took hold of the listeners’ or readers’ attention by their sheer intellectual power. Achievements  He showed how land revenue & the salt tax had been going up even in times of drought and famine.

 He asked for the reduction of these 2 taxes & for raising the minimum income liable to income tax to Rs. 1k so that the lower middle classes would not be harassed.

 He condemned the large expenditure on the army & territorial expansion beyond Indian frontiers & demanded greater expenditure on education & industry instead.  The management of Indian finances, he said, revealed that Indian interests were invariably subordinated to foreign interests.

 He linked the poor state of Indian finances & the poverty of the people with the colonial status of the Indian economy & polity.

 And he did all this by citing at length from the Government’s own blue books.’  Gokhale was to be repaid in plenty by the love & recognition of his own people.

 Proud of his legislative achievement they were to confer him the title of ‘the leader of the opposition’.

 Gandhiji was to declare him his political guru.

 And Tilak, his lifelong political opponent, said at his funeral: ‘This diamond of India, this jewel of Maharashtra, this prince of workers, is taking eternal rest on the funeral ground. Look at him & try to emulate him.” The new Councils attracted some of the most prominent nationalist leaders.

 SN Banerjea, Kalicharan Banerjee, A M Bose, Lal Mohan Ghosh, W.C. Bonnerji & Rash Behari Ghosh from Bengal,

 Ananda Charlu, C. Sankaran Nair & Vijayaraghavachariar from Madras

 Madan Mohan Malaviya, Ayodhyanath & Bishambar Nath from U.P.

 B.G. Tilak, Pherozeshah Mehta, R.M. Sayani, Chimanlal Setalvad, N.G. Chandravarkar & G.K. Gokhale from Bombay

 G.M. Chitnavis from Central Provinces were some of the members of the Provincial or Central Legislative Councils from 1893 to 1909. Moderate Leaders  Dadabhai Naoroji, Pherozshah Mehta, Dinshaw E. Wacha, W.C Bonnerjee, S.N Banerjee believed in liberalism & moderate methods in comparison to the neo- nationalist(extremists like Lal, Bal, Pal) of 19th century

 Other prominent leaders included

 Mahadeo Govind Ranade,  Sisir Kumar Ghosh,  Motilal Ghosh,  Madan Mohan Malaviya,  G. Subramaniya Aiyar,  C. Vijayaraghavachariar

2011 What was the purpose with which sir William Wedderburn and W.S. Caine had set up the Indian parliamentary committee in 1893 ?

(a)To agitate for Indian political reforms in the house of commons.

(b)To campaign for the entry of Indians into the imperial judiciary.

(c)To facilitate a discussion on India’s independence in the British parliament.

(d)To agitate for the entry of eminent Indians into the British parliament.

Criticism  Confined to urban centers – Bombay, Madras & Calcutta which left rural part untouched  Class & Caste Composition of the party focused only on certain sections of society

 Most leaders were from the Upper Caste (Brahmins or Kshatriya)  Class wise it was dominated by people from upper class like Zamindars, Traders, Lawyers, Doctors, Capitalist etc

 Peasant class or working class people were not present in the party

 From 1893 Nationalism was linked to symbols of Hinduism which alienated the Muslims & people from other religion  INC failed to address the grievances of the minority

 Number of women members in the party was very low  They lacked faith in the common people, did no work among them & consequently failed to acquire any roots among them.  Even their propaganda did not reach them.  Nor did they organize any all- India campaigns & when, during 1905-07, such an all-India campaign did come up in the form of the Swadeshi & Boycott Movement, they were not its leader & (though Bengal Moderates did play an active role).  Their politics were based on the belief that they would be able to persuade the rulers to introduce economic & political reforms but their practical achievements in this respect were meagre.  Instead of respecting them for their moderation, the British treated them with contempt, sneered at their politics & met popular agitations with repression.  Their basic failure, however, was that of not keeping pace with events.

 They could not see that their own achievements had made their Politics obsolete.  They failed to meet the demands of the new stage of the national movement. Visible proof of this was their failure to attract the younger generation. Attitude of the government  The British Indian Government was hostile to the Congress from the beginning despite the latter's moderate methods & emphasis on loyalty to the British Crown.  The official attitude stiffened further after 1887 when the Government failed to persuade the Congress to confine itself to social questions while the Congress was becoming increasingly critical of the colonial rule.

 Now, the Government resorted to open condemnation of the Congress, calling the nationalists "seditious brahmins", "disloyal babus“ & ‘violent villains.’

 Congressmen as ‘disappointed candidates for office & discontented lawyers who represent no one but themselves.’  In 1888, Dufferin, the Viceroy, attacked the Congress in a public speech & ridiculed it as representing only the elite ‘a microscopic minority.”  Dufferin called, the Congress "a of sedition".  George Hamilton, Secretary of State for India, accused the Congress leaders of possessing ‘seditious & double sided character.’ Attitude of the government  Later, the Government adopted a 'divide & rule‘ policy towards the Congress.

 The officials encouraged reactionary elements like Syed Ahmed khan & Raja Shiv Prasad Singh of Benaras to organize the United Patriotic Association to counter, Congress propaganda.

 The Government also tried to divide the nationalists on the basis of religion & through a policy of 'carrot & stick', pitted the Moderates against the Extremists. But it failed to check the rising tide of nationalism.

 This hostility did not abate when the Moderates, who then controlled the Congress, began to distance themselves from the rising militant nationalist tendencies of certain sections of the Congress which became apparent when the government unleashed a repressive policy against the Indian Press in 1897.

British appeared more eager to attack & finish the Congress. Why was this so?

 1st because however moderate & loyal in their political perception the Moderates were, they were still nationalists and propagators of anti-colonialist politics & ideas.

 As Curzon put it in 1905: ‘Gokhale either does not see where he is going, or if he does see it, then he is dishonest or his pretensions. You Cannot awaken & appeal to the spirit of nationality in India & at the same time, profess loyal acceptance of British rule.’

 George Hamilton, the Secretary of State, had complained to Dadabhai in 1900: ‘You announce yourself as a sincere supporter of British rule; you vehemently denounce the condition, & consequences which are inseparable from the maintenance of that rule.” Important sessions of INC  1885 Gokuldas Tejpal sanskrit pathsala Bombay W.C.Bonnerjee (Indian Christian)

 1886 Calcutta Dadabhai Naoroji (Parsee) 3 times presided , grand old man of India, father of Indian economic nationalism

 1887 Madras Badrudin Tyabji (1st muslim)

 1888 Allahabad George Yule (1st European)

 1889 Bombay William Wedderburn (2nd European)

 1891 Nagpur P. Anand Charlu (1st Hindu)

 1893 Lahore Dadabhai Naoroji

Some of the other great presidents of the Congress during this early phase were

 Pheroz Shah Mehta

 Surendranath Banerjee

 Romesh Chandra Dutt

 Ananda Mohan Bose

Important sessions of INC  1905 , , Gopal Krishna Gokhale, difference between moderates & neo nationalists on issue of swadeshi & boycott

 1906 Calcutta Dada Bhai , 1st time Vande Matram was sung on INC platform

 1907 Surat , Rash Bihari Ghosh

 1908 Madras, Rash Bihari Ghosh same session reconvened, neo nationalists were expelled from INC , 1908-1915 moderates dominance

 1916 Lucknow Ambika Charan Majumdar Lucknow pact between INC & Muslim League

Important sessions of INC

 1917 Calcutta Annie Besant, 1st woman president ,4th European

 1918 Delhi Madan Mohan Malviya

 1919 Amritsar Motilal Nehru

 1922 Gaya ,C.R Das, Congress Khilafat ,

 1924 Mahatma Gandhi

 1925 Kanpur 2nd woman president, 1st Indian woman, 1st lady governor of independent India Important sessions of INC 1929 Lahore, Jawaharlal Nehru

1938 Haripura, Subhash Bose

1939 Tripuri, Subhash Bose  Pattabhi Sitaramiya 

1940 Ramgarh, Maulana longest serving president (1940-45), No annual session held

1946 Merrut , Acharya J.B.Kriplani, president at the eve of Independence, launched Kisan Majdur Praja Party

1948 Jaipur Pattabhi Sitaramiya , 1st session after Independence 2015 Consider the following statements: 1) The first woman President of the Indian National Congress was Sarojini Naidu.

2) The first Muslim President of the Indian National Congress was Badruddin Tyabji.

Which of the statements given above is/are correct? a) 1 only b) 2 only c) Both 1 nor 2 d) Neither 1 nor 2 HISTORY March 23, 2018 Civil Disobedience Movement  It means to disobey a law with great civility and humility

 It was Gandhi’s movement. Causes of the Movement  The British passed a law forbidding Indians to manufacture salt which could only be done by English or European manufacturers

 There were large scale arrests during the anti-Simon Commission movement.

 1929-33 was the period of Great Depression which affected all capitalist countries including India.

 It led to widespread unemployment, poverty and unrest.

 Gandhi presented 11 point program to the government. 11 point program 1) Total prohibition (of alcohol). 2) Reduction of ratio to 1s. 4d. (rupee revaluation). 3) Reduction of land revenue to at least 50 % and making it subject to Legislative control. 4) Abolition of the Salt Tax. 5) Reduction of Military expenditure at least by 50 % to begin with. 6) Reduction of salaries of the highest grade services by 1/2 or less, so as to suit the reduced revenue. 7) Protective tariff on foreign cloth. 8) Passage of the Coastal Traffic Reservation Bill.

9) Discharge of all political prisoners, save those condemned for murder or attempt to murder ... 10) Abolition of the CID (Criminal Investigation Department)

11) To issue licenses to use fire-arms for self-defense, subject to popular control. Nature  Gandhi started Dandi March with only 78 people, from Gandhi’s Sabarmati Ashram on 12 March, 1930, but was joined by thousands by the time he reached Dandi.

 The entire distance was covered on foot passing through villages, towns. This enabled Gandhi to connect to the masses with his unique pad-yatra.

 Gandhian politics was not power-centric but focussed more on mobilising people for the cause of nation.  Swadeshi goods became popular again and foreign goods were boycotted  The peasant movement became more radical in regions where their union was strong like eastern UP & Bihar.  peasants stopped paying Chowkidari Tax to Zamindars.  In North West Frontier Province, Khan Abdul Gaffar Khan  C Rajagopalachari  K Kellapan 2015 Who of the following organized a march on the Tanjore coast to break the Salt Law in April 1930? (a) V.O. Chidambaram Pillai

(b) C. Rajagopalachari

(c) K. Kamaraj

(d) Annie Besant

Answer – (B) Features  It was an all India mass movement.

 It was more popular among the peasants & in rural areas.

 The unity among Hindus & Muslims was missing. This was due to Jinnah’s demands & differences between Congress & Muslim League.

 The kind of resilience shown by Gandhian Satyagrahis was very unique.

 Dharasana salt works

 Sarojini Naidu, Imam sahab , Manilal Gandhi

 Webb Miller

Round Table Conference  Round Table conference signifies that all the participants are equal.

 It is symbolic that every group will be treated equally

 Participants of these conferences were – All political parties of India, Representatives of India Princely states

 It was held in Buckingham Palace in London

 Purpose of this meeting according to British was that they wanted to understand demands & grievances of Indian Leadership from different social backgrounds

 Nationalist interpreted it as a mere eyewash First Round Table Conference  November 1930-January 1931

 1st ever conference arranged between the British & Indians as equals

 It was held during the civil disobedience movement

 Congress did not participate in the conference

 Most business leaders‘ boycotted it

 Muslim League, Hindu Mahasabha, the Liberals & princes attended it.

 Virtually every delegate reiterated that a constitutional discussion to which the Congress was not a party was meaningless.  Also, at the conference, the British PM hinted at an olive branch to the Congress & expressed the hope that the Congress would attend the next RTC. Gandhi-Irwin Pact – 5th March, 1931  It had been almost a year since the movement began which forced the British to initiate a dialogue with Gandhi  On January 25, 1931 Gandhi & all other members of the CWC were released unconditionally.

 1st time that an Indian leader was invited by the British for a dialogue at almost equal footing  Gandhi agreed to suspend the Civil Disobedience & government agreed to release all political prisoners

 The CWC authorized Gandhi to initiate discussions with the viceroy.  As a result of these discussions, a pact was signed between the viceroy, representing the British Indian Government, & Gandhi, representing the Indian people, in Delhi.

Irwin on behalf of the Government agreed on  immediate release of all political prisoners not convicted of violence;  remission of all fines not yet collected;  return of all lands not yet sold to third parties;  lenient treatment to those government servants who had resigned;  right to peaceful & non-aggressive picketing;  withdrawal of emergency ordinances.

 The salt law was amended & people living in coastal areas were allowed to manufacture salt for personal consumption.

 Peaceful agitation against liquor was allowed & confiscated property was returned

 The viceroy, however, turned down 2 of Gandhi's demands—

 public inquiry into police excesses  commutation of Bhagat Singh & his comrades' death sentence to life sentence. Gandhi on behalf of the Congress agreed—

(i) to suspend the civil disobedience movement,

(ii) Gandhi agreed to participate in II Round Table Conference on the constitutional question. Evaluation of civil disobedience movement Was Gandhi-Irwin Pact a Retreat? Gandhi's decision to suspend the movement as agreed under the Pact was, not a retreat, because:

(i) mass movements are necessarily short-lived;

(ii) capacity of the masses to make sacrifices, unlike that of the activists, is limited;

(iii) there were signs of exhaustion after September 1930, especially among shopkeepers and merchants, who had participated so enthusiastically. Continued youth.

Peasants of Gujarat But vast masses of people were jubilant that the Government had to regard their movement as significant & treat their leader as an equal, & sign a pact with him.

The political prisoners when released from jails were given a hero's welcome. Compared to Non-Cooperation Movement 1. The stated objective this time was complete independence & not just a vaguely- worded swaraj.

2. The methods involved violation of law from the very beginning & not just non- cooperation with foreign rule.

3. There was a decline in forms of protests involving the intelligentsia, such as lawyers giving up practice, students giving up government schools to join national schools & colleges. 4. Muslim participation was nowhere near the Non Cooperation level. 5. No major labor upsurge coincided with the movement. 6. But massive participation of peasants & business groups compensated for decline of other features. 7. The number of those imprisoned was about 3 times more this time. 8. The Congress was organisationally stronger. Karachi Congress Session(March 29) In March 1931, a special session of the Congress was held to endorse the Pact.

6 days before the session Bhagat Singh, Sukhdev & Rajguru had been executed.

Throughout Gandhi's route to Karachi, he was greeted with black flag demonstrations by the Punjab Naujawan Bharat Sabha, in protest against his failure to secure commutation of the death sentence for Bhagat & his comrades.

Congress Resolutions

While disapproving of & dissociating itself from political violence, the Congress admired the "bravery" & "sacrifice" of the 3 martyrs. The Delhi Pact was endorsed.

The goal of purna swaraj was reiterated. 2 resolutions were adopted—one on Fundamental Rights & the other on National Economic Programme. The resolution on Fundamental Rights guaranteed—

* free speech and free press * right to form associations * right to assemble * universal adult franchise * equal legal rights irrespective of caste, creed and sex * neutrality of state in religious matters * free and compulsory primary education * protection to culture, language, script of minorities and linguistic groups The resolution on National Economic Programme included—  substantial reduction in rent and revenue  exemption from rent for uneconomic holdings  relief from agricultural indebtedness  control of usury  better conditions of work including a living wage, limited hours of work & protection of women workers  right to workers and peasants to form unions

 state ownership & control of key industries, mines & means of transport.

 This was the 1st time the Congress spelt out what swaraj would mean for the masses—"in order to end exploitation of masses, political freedom must include economic freedom of starving millions.“  The Karachi Resolution was to remain, in essence, the basic political and economic programme of the Congress in after years. Second Round Table Conference  December 1931

 Congress participated in this conference as it was a part of pact

 Gandhi represented Congress in this conference

 This conference turned aggressive between Gandhi & rival nationalist group leaders. Not much was expected from the conference because of the following reasons. 1. The Right Wing in Britain led by Churchill strongly objected to the British Government negotiating with the Congress on an equal basis. They, instead, demanded a strong government in India. 2. Overwhelming majority of RTC delegates were conservative, loyalist, reactionary & communal, men who had been used by the colonial government to assert that the Congress did not represent all Indians & to neutralise Gandhi & his efforts.

3. The session soon got deadlocked on the question of the minorities. Separate electorates were being demanded by the Muslims, depressed classes, Christians & Anglo-Indians. All these came together in a "Minorities' Pact". 4. Princes were also not as enthusiastic about a federation Second Round Table Conference The session ended with MacDonald's announcement of:

(i) 2 Muslim majority provinces—NWFP & Sindh;

(ii) the setting up of Indian Consultative Committee;

(iii) three expert committees—finance, franchise and states;

(iv) the prospect of a unilateral British Communal Award if Indians failed to agree.

The Government failed to concede the basic Indian demand of freedom.

Gandhi returned to India on December 28, 1931. On December 29, the CWC decided to resume the civil disobedience movement. Changed Government Attitude  The higher British officials had drawn their own lessons from the Delhi Pact which had raised the political prestige of the Congress & the political morale of the people & had undermined British prestige. They were now determined to reverse this trend. There were 3 main considerations in British policy:

 Gandhi wont be permitted to build up the tempo for a mass movement again.

 Goodwill of the Congress was not required, but the confidence of those who supported the British against the Congress—government functionaries, loyalists, etc.—was very essential.

 The national movement would not be allowed to consolidate itself in rural areas.

 After CWC had decided to resume civil disobedience the new Viceroy Willingdon refused a meeting with Gandhi on December 31. On January 4, 1932, Gandhi was arrested. Popular Response  People responded with anger.  In the first 4 months alone, about 80,000 satyagrahis, mostly urban & rural poor, were jailed.  Other forms of protest, included picketing of shops selling liquor & foreign cloth, illegal gatherings, non-violent demonstrations, celebrations of national days, symbolic hoisting of national flag, non-payment of chowkidari tax, salt satyagraha, forest law violations & installation of a secret radio transmitter near Bombay.

 This phase of the civil disobedience movement coincided with upsurges in 2 princely states—Kashmir & Alwar.

 But this phase of the movement could not be sustained for long because  (i) Gandhi & other leaders had no time to build up the tempo; &  (ii) the masses were not prepared.

 Finally in-April 1934, Gandhi decided to withdraw the movement. MacDonald Award or Communal Award  British PM Ramsay Macdonald announced in 1932 that depressed class is a minority & will get separate electorate.

 This was a long demand of B.R. Ambedkar as the depressed classes had no chance of entering the assembly as election was based on limited franchise.

 The reasons for this opposition was:-

 British government was dividing Indians

 This would lead to great damage to community

Congress Stand

 Congress leadership was not happy.

 Though opposed to separate electorates, the Congress was not in favor of changing the Communal Award without the consent of the minorities.

 Thus, while strongly disagreeing with the Communal Award, the Congress decided neither to accept it nor to reject it.

 The effort to separate the depressed classes from the rest of the Hindus by treating them as separate political entities was vehemently opposed by all the nationalists. Gandhi's Response  Gandhi was in deep shock & opposed separate electorate.  He saw the Communal Award as an attack on Indian unity and nationalism.

 He thought it was harmful to both Hinduism and to the depressed classes since it provided no answer to the socially degraded position of the depressed classes.

 He said that what was required was not protection of the so called interests of the depressed classes but eradication of untouchability

 Gandhi demanded that the depressed classes be elected through joint & if possible a wider electorate through universal franchise, while expressing no objection to the demand for a larger number of reserved seats. Poona Pact 1932  Gandhi was lodged in the Yerwada Jail. He went of fast unto death in opposition of Macdonald Award on September 20, 1932  Due to Gandhi’s fast unto death, there was huge pressure on Ambedkar  Now leaders of various persuasions, including Ambedkar, MC Rajah & Malaviya got together to hammer out a compromise  Ambedkar agreed to meet Gandhi for negotiations  After the meeting pact was signed, and its features were:-  No Separate electorate for depressed classes  Instead of separate electorate reservation of seats would be given  But the seats reserved for the depressed classes were increased from 71 to 147 in provincial legislatures & 18 per cent of the total in the central legislature.  Candidates would be from depressed class but voters from all communities will vote for electing one of the candidates  Pact was accepted by the Government as an amendment to Communal Award. Gandhi's Harijan Campaign Determined to undo the divisive intentions of the Government's divide and rule policy, Gandhi gave up all his other preoccupations & launched a whirlwind campaign against untouchability—1st from jail & after his release in August 1933 from outside. In jail, he had set up the All India Anti Untouchability League in September 1932 and had started the weekly Harijan in January 1933. After his release, he shifted to the Satyagraha Ashram in Wardha as he had vowed in 1930 not to return to Sabarmati Ashram unless swaraj was won.

Starting from Wardha, he conducted a Harijan tour of the country in the period from November 1933 to July 1934, covering 20,000 km, collecting money for his newly set up Harijan Sevak Sangh, & propagating removal of untouchability in all its forms. He urged political workers to go to villages & work for social, economic, political & cultural upliftment of the Harijan.

He undertook 2 fasts— on May 8 & August 16, 1934—to convince his followers of the seriousness of his effort & the importance of the issue. Gandhi's Harijan Campaign

Throughout his campaign, Gandhi was attacked by orthodox & reactionary elements.

These elements disrupted his meetings, held black flag demonstrations against him & accused him of attacking Hinduism.

They also offered support to the Government against the Congress & the Civil Disobedience Movement.

The Government obliged them by defeating the Temple Entry Bill in August 1934.

Orthodox Hindu opinion in Bengal was against the acceptance of permanent caste Hindu minority status by the Poona Pact. Gandhi's Thoughts on Caste  He put forward a damning indictment of Hindu society for the kind of oppression practiced on Harijans.

 He called for the root & branch eradication of untouchability symbolised by his plea to throw open temples to the untouchables.

 He stressed the need for caste Hindus to do "penance" for untold miseries inflicted on Harijans. For this reason he was not hostile to his critics such as Ambedkar

 He said, "Hinduism dies if untouchability lives, untouchability has to die if Hinduism is to live.“

 His entire campaign was based on principles of humanism & reason He said that the shastras do not sanction untouchability & if they did, they should be ignored as it was against human dignity. Impact of the Campaign

Gandhi repeatedly described the campaign as not a political movement but as being primarily meant to purify Hinduism & Hindu society.

Gradually, the campaign carried the message of nationalism to Harijans who also happened to be the agricultural laborers in most parts of the country, leading to their increasing participation in the national & peasant movements. Third Round Table Conference

 It was held in 1932

 Congress boycotted this conference

 Muslim League, Hindu Mahasabha & Princely states participated

 There were no significant outcomes of this conference Spread of Marxism and Socialist Ideas These ideas inspired many socialist & communist groups to come into existence & resulted in the rise of a left wing within Congress, represented by J Nehru & Bose.

These young nationalists, inspired by the Soviet Revolution & dissatisfied with Gandhian ideas & political programme, began advocating radical solutions for economic, political & social ills of the country. These younger nationalists—

 were critical both of Swarajists and No-changers, advocated a more consistent anti- imperialist line in the form of a slogan for purna swarajya.

 though still vague, of international currents, stressed the need to combine nationalism and anti imperialism with social justice &

 simultaneously raised the question of internal class oppression by capitalists & landlords.  Among the communist groups, the Communist Party of India was formed in 1920 in Tashkent (Uzbekistan) by M.N. Roy, Abani Mukherji & others after the 2nd Congress of Commintern.  M.N. Roy was also the 1st to be elected to the leadership of Commintern.  In 1924, many communists—S.A. Dange, Muzaffar Ahmed, Shaukat Usmani, Nalini Gupta—were jailed in the Kanpur Bolshevik conspiracy case.  1925:Indian Communist Conference at Kanpur formalized foundation of the CPI.

 In 1929, the Government crackdown on communists resulted in the arrest & trial of 31 leading communists, trade unionists & left-wing leaders who were tried at Meerut in the famous Meerut conspiracy case.  Workers' & peasants' parties were organised all over the country & they propagated Marxist & communist ideas.  All these communist groups & workers' & peasants' parties remained an integral part of the national movement. Left Groups In India – 1934

 Congress was formed at Bombay

 Acharya Narendra Dev was the President & Loknayak was the secretary

 The movement originated from 1917 October Revolution. Origin of the Movement  Congress had to satisfy every section of society

 Gandhi evolved philosophy of Trusteeship which came which was not idealistic & was very difficult to implement.

 Trusteeship is a socio-economic philosophy that was propounded by Mahatma Gandhi . It provides a means by which the wealthy people would be the trustees of trusts that looked after the welfare of the people in general.

 All this lead to Congress v/s Communist verbal fights. Communist labelled Congress as a party of Bourgeoisie (middle) class

 Congress termed Communists as “Russian satellite”

 As congress had mass support, communist party weakened. Communist Party could not become influential as it suffered from two sides.

These are:-

 From Society  Peasants did not connect  The Ideas of the party were very radical  Communism failed to penetrate rural majority  Members against revolution formed Socialist group in 1934

 From Government  Americans, French supported capitalism  Government suppressed communist movement  CPI was banned & members were arrested. Meerut Trial & Kanpur Trial were the two famous trials. Purpose of Movement  This movement was started as Congress didn’t do much about peasants’ & workers’ plight.

 Number of worker & peasant organizations emerged but didn’t join Congress movement

 Real issues of peasants were not addressed thus this job was left to communist & socialist.

Government Of India Act 1935  Simon Commission

 Issue of paper by Government of England

 Failure of Round table Conference

 Pressure from Nationalists

 Demands made by Provisions  Autonomy to the provinces

 End of Dyarchy from provinces but introduced at the central government level

 All – India Federation would be formed  Thus increasing the franchise from 7 million to 35 million people  Federal court was set up in 1936 (Called Supreme court of India later on)  To settle dispute between Union Government &Provincial Government  To settle disputes between two provinces

 Burma was separated from British India  Orissa was separated from Bihar with Cuttack as capital  Sind was separated from Bombay with Karachi as the capital  The number of provinces were increased to 11 by giving NWFP the status of fully- fledged province & creating 2 new provinces, Orissa & Sindh. Significance  This act became the basis of free India’s Constitution & had great impact on the modern Indian federal structure.

 Modern Federal Court Structure was taken from this act.

 This act gave more powers to our legislators Criticism

 It gave too much power to the governors of provinces

 All India Federation clause could not be implemented

 Nehru : "a machine with strong brakes but no engine.“

 Jinnah : "thoroughly rotten, fundamentally bad and totally unacceptable."

2012 The distribution of powers between the Centre and the States in the Indian Constitution is based on the scheme provide in the a) Morley-Minto reforms 1909 b) Montagu Chelmsford act 1919 c) Govt. of India act, 1935 d) Indian Independence Act 1947

Answer: C Congress Manifesto for Elections  Total rejection of the 1935 Act

 Promised release of prisoners

 Removal of disabilities on the basis of gender & caste

 Radical transformation of the agrarian system

 Substantial reduction of rent and revenue

 Scaling down of rural debts, cheap credit

 Right to form trade unions & to strike.

Congress' Performance  Provincial elections were held in British India in 1936-37 as mandated by the 1935 Act.

 Elections were held in 11 provinces - Madras, Central Provinces, Bihar, Orissa, United Provinces, Bombay Presidency, Assam, NWFP, Bengal, Punjab & Sindh.

 Congress emerged in power in 8 of the provinces - the 3 exceptions being Bengal, Punjab & Sindh.

 Gandhi did not attend a single election meeting. League’s Performance  All-India Muslim League failed to form the government in any province.

 League was the second-ranked party.

 After the election Jinnah offered to form coalitions with the Congress.

 League insisted that the Congress should not nominate any Muslims to the ministries, as it claimed to be the exclusive representative of Indian Muslims.

 This was not acceptable to the Congress, & it declined the League's offer. 28 Months Of Congress Rule In Provinces Civil Liberties The Congress ministries did much to ease curbs on civil liberties:  Laws giving emergency powers were repealed.  Ban on illegal organizations, such as the Hindustan Seva Dal & youth Leagues, & on certain books & journals was lifted.

 Press restrictions were lifted & newspapers were taken out of black lists.  Confiscated arms & arms licenses were restored.  Police powers were curbed & CID stopped shadowing politicians.

 Political prisoners, & revolutionaries were released, & deportation & internment orders were revoked.

Blemishes  Gandhi urged Congressmen to prove that the Congress could rule with least assistance from the police & the Army.

 But there were certain blemishes in the performance of the Congress ministries.

 Yusuf Maherally, a socialist, was arrested by the Madras Government for inflammatory speeches & later released.

 S.S. Batliwala, a socialist, was arrested by the Madras Government for seditious speech & given 6 months' sentence.

 K.M. Munshi, the Bombay Home Minister, used CID against communists & leftists. Agrarian Reforms  There were certain basic constraints before the Congress ministries could undertake a complete overhaul of the agrarian structure by completely abolishing zamindari.  These constraints were

 The ministries did not have adequate powers.  Inadequate financial resources as a lion's share was appropriated by the GoI.

 There was constraint of time since the logic of Congress politics was confrontation & not cooperation with colonialism.  The agrarian structure was too complex & complicated. Agrarian Reforms

 In spite of these constraints, the ministries managed to legislate a number of laws relating to land reforms, debt relief, forest grazing fee, arrears of rent, land tenures, etc.

 But most of these benefits went to statutory & occupancy tenants while sub-tenants did not gain much. Social Welfare Reforms  Prohibition imposed in certain areas.

 Measures for welfare of Harijans taken—temple entry, use of public facilities, scholarships, an increase in their numbers in government service & police, etc.

 Attention given to primary, technical & higher education, to public health & sanitation.

 Encouragement given to khadi through subsidies & other measures.

 Prison reforms undertaken.

 Encouragement given to indigenous enterprises.

 Efforts taken to develop planning through National Planning Committee set up under Bose in 1938. Extra-Parliamentary Mass Activity of Congress  Launching of mass literacy campaigns

 Setting up of Congress police stations & panchayats

 Congress Grievance Committees presenting mass petitions to Government

 States peoples' movements. Attitude Towards Labor  The basic approach was to advance workers' interests while promoting industrial peace. This was sought to be achieved by reducing strikes as far as possible & by advocating compulsory arbitration prior to striking.  Goodwill was sought to be created between labor and capital with mediation of ministries, while at the same time efforts were made to improve workers‘ condition and secure wage increases for them. The ministries treated militant trade union protests as law & order problems, & acted as mediators as far as possible.  This approach was largely successful but not so in Bombay. Also, leftist critics were not satisfied by this approach.  Generally, the ministries took recourse to Section 144 & arrested the leaders. Congress Position Before War  Congress' hostility to Fascism, Nazism, militarism & imperialism had been much more consistent than the British record.

 But the Indian offer to cooperate in the war effort had 2 basic conditions:

 After the war, a constituent assembly should be convened to determine political structure of a free India.

 Immediately some form of a genuinely responsible government should be established at the center.

 The offer was rejected by Lilingthow.

 Congress argued that these conditions were necessary to win public opinion for war. CWC Meeting at Wardha (September 10-14, 1939) Different opinions were voiced on the question of Indian support to British war efforts—  Gandhi advocated an unconditional support to the Allied as he made a clear distinction between the democratic states of West Europe & the totalitarian Nazis.  Bose & the socialists argued that war was an imperialist one since both sides were fighting for gaining or defending colonial territories. Therefore, the question of supporting either of the sides did not arise. Instead, advantage should be taken of the situation to wrest freedom by immediately starting a civil disobedience movement.  Nehru made a sharp distinction between democracy & Fascism. He believed that justice was on the side of Britain & France but was also convinced that Britain & France were imperialist powers, & that the war was the result of the inner contradictions of capitalism maturing since the end of World War I. He, therefore, advocated no Indian participation till India itself was free. However, at the same time, no advantage was to be taken of Britain's difficulty by starting an immediate struggle. CWC resolution The CWC resolution condemned Fascist aggression. It said that

 India could not be party to a war being fought ostensibly for democratic freedom, while that freedom was being denied to India;

 if Britain was fighting for democracy & freedom, it should prove it by ending imperialism in its colonies & establishing full democracy in India;

 the Government should declare its war aims soon and, also, as to how the principles, of democracy were to be applied to India.

 Congress leadership wanted to give every chance to the viceroy & the British Government Government's Response  Linlithgow, in his statement (October 17, 1939), tried to use the League & the princes against the Congress.

 The Government

 refused to define British war aims beyond stating that Britain was resisting aggression;

 said it would, as part of future arrangement, consult "representatives of several communities, parties & interests in India, & the Indian princes" as to how the 1935 Act might be modified;

 said it would immediately set up a "consultative committee" whose advice could be sought whenever required. Government's Hidden Agenda  Viceroy's statement was not an aberration, but a part of general British policy—"to take advantage of the war to regain the lost ground from the Congress" by provoking the Congress into a confrontation with the Government & then using the extraordinary situation to acquire draconian powers.

 It could also win an unusual amount of liberal & leftist sympathy all over the world by painting an aggressive Congress as being pro-Japan & pro-Germany.

 British Indian reactionary policies received full support from PM Churchill & the Secretary of State, Zetland, who branded Congress as a purely Hindu organisation.  It became clear that the British Government had no intention of loosening its hold, during or after the war, and was willing to treat the Congress as an enemy. Gandhi reacted sharply to the Government's insensitivity to Indian public opinion.

 On October 23, 1939, the CWC meeting  rejected the viceregal statement as a reiteration of the old imperialist policy,  decided not to support the war,  called upon the Congress ministries to resign in the provinces.

Resignation of Congress ministries  Viceroy Linlithgow declared India at war with Germany on 3 September 1939.

 Congress objected to the declaration of war without prior consultation with Indians.

 League promised its support to the British, with Jinnah calling on Muslims to help the Raj by "honourable co-operation" at the "critical and difficult juncture," while asking the Viceroy for increased protection for Muslims.

 The government did not come up with any satisfactory response.

 Both Linlithgow & Jinnah were pleased with the resignations.  On 2 December 1939, Jinnah put out an appeal, calling for Indian Muslims to celebrate 22 December 1939 as a "Day of Deliverance" from Congress: 2012 The congress ministries resigned in the seven provinces in 1939, because a. The Congress could not form ministries in the other four provinces b. emergence of a ‘left wing’ in the Congress made the working of the ministries impossible c. there were widespread communal disturbances in their provinces. d. None of the statements (a), (b) and (c) given below is correct.

Answer: D Debate on the 'Question of Immediate Mass Satyagraha  After Linlithgow's statement of October 1939, the debate on the question of immediate mass struggle began once again. Gandhi & his supporters were not in favor of an immediate struggle because they felt that the  Allied cause was just;  communal sensitivity & lack of Hindu-Muslim unity could result in communal riots;  Congress organisation was in shambles & the atmosphere was not conducive for a mass struggle;  masses were not ready for a struggle.

 They instead advocated toning up the Congress organisation, carrying on political work among the masses, & negotiating till all possibilities of a negotiated settlement were exhausted. Only then would the struggle be begun. The views of the dominant leadership were reflected in the Congress resolution at the Ramgarh session (1940)—"Congress would resort to civil disobedience as soon as the Congress organisation is considered fit enough or if circumstances precipitate a crisis."  A coalition of leftist groups— Bose & his Forward Bloc, Congress Socialist Party, Communist Party, —characterized the war as an imperialist war giving an opportunity to attain freedom through an all-out struggle against British imperialism.

 This group was convinced that the masses were ready for action, only waiting for a call from the leadership. They accepted hurdles, such as communalism & the shortcomings of the Congress organization, but thought that these would be automatically swept away in the course of a struggle. They urged the Congress leadership to launch an immediate mass struggle.  Bose even proposed a parallel Congress to organise an immediate mass struggle if the Congress leadership was not willing to go along with them, but the CSP & CPI differed with Bose on this  Nehru considered the Allied powers as imperialists & his philosophy & political perception leant towards the idea of an early struggle but that would have undermined the fight against Fascism. He finally went along with Gandhi & the Congress majority. Pakistan Resolution-Lahore (March 1940)  The Muslim League passed a resolution calling for "grouping of geographically contiguous areas where Muslims are in majority (North- West, East) into independent states in which constituent units shall be autonomous & sovereign & adequate safeguards to Muslims where they are in minority". HISTORY March 23, 2018 Non Cooperation Movement  Government of India Act, 1919 : Ill conceived scheme of dyarchy failed to satisfy the rising demand of the Indians for self government.

, the imposition of martial law in Punjab

 Jallianwala Bagh Massacre & the enquiry thereafter

 Reasons for which the was started was a major cause for the Muslim population

 Economic hardship post war

 inflation  Depression  Unemployment Khilafat Movement  The word Khilafat means institution or office of & the Khalifa or caliph of Turkey was regarded as the spiritual & political head of Muslim world.

 In the world war-I Turkey with Germany lost to the allied power.  The victory of allied forces imposed a humiliating treaty on Germany & a similar humiliating treaty was being planned for Turkey.

 The demands of the Khilafat Committee which was set up in India by Ali brothers, Maulana & Shaukat Ali were:-

 The Turkish Khalifa must retain control over the Muslim sacred places.  The Khilafat must be left with sufficient territory to enable him to defend the Islamic faith  The Jazirat-Ul Arale (Arabia, Syria, Iraq, & Palestine) must remain under the Muslim Sovereignty. Congress on Khilafat  Congress support was essential for Khilafat issue

 Congress wasn’t united on it

 Tilak was opposed to an alliance with Muslim leaders over a religious issue

 He had doubts over satyagraha too

 Boycott of councils was not supported by all :Leaders like C.R. Das were not willing, but bowed to Congress discipline; they leaders boycotted elections held in November 1920 and the majority of the voters too stayed away

 But Gandhi got approval of all on this issue

 Muslim league too supported the Congress on this issue  In this atmosphere emerged the Khilafat issue around which developed the historic Non-Cooperation Movement.  February 1920 : Joint Hindu Muslim delegation sent to viceroy to seek redress of grievances on the issue of Khilafat but the mission failed  In February 1920, Gandhi announced that the issues of the Punjab, wrongs & constitutional advance had been overshadowed by the Khilafat question & that he would soon lead a movement of non-cooperation if the terms of the peace treaty failed to satisfy the Indian Muslims.

 May 1920: The Treaty of Sevres completely dismembered Turkey.

 June 1920 An all-party conference at Allahabad approved a programme of boycott of schools, colleges & law courts, & asked Gandhi to lead it.

 August 31, 1920 The Khilafat Committee started a campaign of non-cooperation and the movement was formally launched. (Tilak had breathed his last on August 1st) Non Cooperation Movement  September 1920 : At a special session in Calcutta, the Congress approved a non cooperation programme till the Punjab & Khilafat wrongs were removed & swaraj was established. The programme was to include—

 boycott of government schools & colleges;  boycott of law courts and dispensation of justice through panchayats instead;  boycott of Legislative Councils;  boycott of foreign cloth & use of khadi instead; also practice of hand-spinning to be done;  renunciation of government honors and titles;

 2nd phase could include mass civil disobedience including resignation from government service, & non-payment of taxes.  During the movement, the participants were supposed to work for Hindu-Muslim unity & for removal of untouchability, all the time remaining non-violent. Nagpur session December 1920  programme of non-cooperation was endorsed;

 an important change was made in Congress creed: now, instead of having the attainment of self-government through constitutional means as its goal, the Congress decided to have the attainment of swaraj through peaceful & legitimate means, thus committing itself to an extra constitutional mass struggle;

 Organizational changes were made:  Congress Working Committee of 15 members was set up to lead the Congress from now onwards;  Provincial Congress Committees on linguistic basis were organised; ward committees was organised; & entry fee was reduced to 4 annas;  Gandhi declared that if the non-cooperation programme was implemented completely, swaraj would be ushered in within a year.  Many groups of revolutionary terrorists, especially those from Bengal, also pledged support to the Congress programme.  At this stage, some leaders like Mohammad Ali Jinnah, Annie Besant, G.S. Kharpade & B.C. Pal left the Congress as they believed in a constitutional & lawful struggle while some others like SN Banerjee founded the Indian National Liberal Federation & played a minor role in national politics hence forward.

 The adoption by the Congress of the non-cooperation movement initiated, earlier by Khilafat Committee gave it a new energy, & the years 1921 & 1922 saw an unprecedented popular upsurge. Spread of the Movement  Gandhi accompanied by the Ali brothers undertook a nationwide tour.  About 90,000 students left government schools & colleges & joined around 800 national schools & colleges which cropped up during this time.  English educational institutions were boycotted  Educational institutions were organised under the leadership of Acharya Narendra Dev, C.R. Das, Lala Lajpat Rai, Zakir Hussain,  Subhash Bose became the principal of National College at Calcutta  Kashi Vidyapeeth, Gujarat Vidyapeeth, Bihar Vidyapeeth Jamia Millia, were set up  English clothes were boycotted, foreign cloth were burnt publicly & their imports fell by 1/2.  Khadi clothes were propagated. Khadi & Charkha became a symbol of protest, national unity & self reliance.  Picketing of shops selling foreign liquor & of toddy shops was undertaken  Many lawyers gave up their practice like Motilal Nehru, J Nehru, C.R. Das, Rajaji, Saifuddin Kitchlew, Vallabhbhai, Asaf Ali, T. Prakasam & R Prasad.  Social boycott Continued  Tilak Swaraj Fund was oversubscribed & 1 crore rupees collected.  Congress volunteer corps emerged as the parallel police.  In July 1921, the Ali brothers gave a call to the Muslims to resign from the Army as that was unreligious for which they were arrested for this in September.  Gandhi echoed their call & asked local Congress committees to pass similar resolutions to that effect. Now, the Congress gave a call to local Congress bodies to start civil disobedience if it was thought that the people were ready for it.

 Already, a no-tax movement against union board taxes in Midnapore & in Guntur was going on.  In Assam, strikes in tea plantations, steamer services, Assam-Bengal Railways had been organised. J.M. Sengupta was a prominent leader in these strikes.  November1921: Visit of Prince of Wales to India invited strikes & demonstrations.  Many local struggles began such as Awadh Kisan Movement , Eka Movement, Mappila Revolt & the Sikh agitation for the removal of mahants in Punjab. The Last Phase of the Movement

 Gandhi was now under increasing pressure from the Congress rank & file to start the civil disobedience programme & the Ahmedabad session in 1921 (presided over, incidentally, by C.R. Das while still in jail; Hakim Ajmal Khan was the acting president) appointed Gandhi the sole authority on the issue.

 On February 1, 1922 Gandhi threatened to launch civil disobedience from Bardoli if

 political prisoners were not released

 press controls were, not removed Withdrawal of the Movement  A mob of peasants & Khilafat agitators burnt 22 policemen alive, after the policemen ordered lathi charge on them, at Chauri Chaura on 5th February 1922

 Congress was planning to launch a “No Tax Movement” as the next phase of this movement & a meeting was planned to be held on 12 February at Bardoli

 CWC met at Bardoli in February 1922 & resolved to stop all activity that led to breaking of law & to get down to constructive work, instead, which was to include popularization of khadi , national schools, Hindu Muslim unity, against untouchability.

Continued  Many nationalists had felt that the non-cooperation movement should not have been stopped due to isolated incidents of violence, & most nationalists, while retaining confidence in Gandhi, were discouraged.

 Impact of the withdrawal was a division of Congress into 2 groups.

 Most Congress leaders remained firmly behind Gandhi, disillusioned broke away.

 Most of the nationalist leaders including C.R. Das, Motilal Nehru, Subhash Bose, Jawaharlal Nehru, however, expressed their bewilderment at Gandhi's decision.

 Motilal Nehru & CR Das formed the Swaraj Party, rejecting Gandhi's leadership.  Ali brothers would soon become fierce critics.

 In March 1922 Gandhi was arrested & sentenced to 6 years in jail. Why Gandhi Withdrew the Movement  Felt that people had not learnt or fully understood the method of non violence.

 Incidents like Chauri Chaura could lead to excitement & fervor, turning the movement generally violent.

 The movement was also showing signs of fatigue.

 The Government seemed to be in no mood for negotiations.

 The central theme of the agitation also dissipated soon. In 1922, the people of Turkey rose under Mustafa Kamal & deprived the Sultan of political power.

 Turkey was made a secular state. A European style of legal system was established in Turkey & extensive rights granted to women. Education was nationalized & modern agriculture & industries developed. In 1924, the caliphate was abolished. Congress-Khilafat Swarajya Party  After Gandhi's arrest (March 1922), there was disintegration, disorganization & demoralization among nationalist ranks.

 A debate started among Congressmen on what to do during the transition period, i.e., the passive phase of the movement.

 One section led by C.R. Das, Motilal Nehru & Ajmal Khan wanted an end to the boycott of legislative councils so that the nationalists could enter them to expose the basic weaknesses of these assemblies & use these councils as an arena of political struggle to arouse popular enthusiasm

 They wanted, in other words, to 'end or mend‘ these councils, i.e., if the Government did not respond to the nationalists' demands, then they would obstruct the working of these councils. Continued  Those advocating entry into legislative councils came to be known as the Swarajists

 Other school of thought led by Vallabhbhai Patel, Rajendra Prasad, C. Rajagopalachari & M.A. Ansari came to be known as the 'No-changers'.

 The 'No-changers' opposed council entry, advocated ,concentration on constructive work, and continuation of boycott & noncooperation, & quiet preparation for resumption of the suspended civil disobedience programme.

 The differences over the question of council entry between the 2 schools of thought resulted in the defeat of Swarajists' proposal at the Gaya session (December 1922).

 1st January 1923: C.R Das & Motilal Nehru resigned from the president ship & secretary ship respectively of the Congress & announced the formation of new party, with C.R. as the president & Moti Lal as one of the secretaries. Swarajists' Arguments  The Swarajists argued that entering the councils would not negate the non- cooperation programme; in fact, it would be like carrying on the movement through other means—opening a new front.

 In a time of political vacuum, council work would serve to enthuse the masses & keep up their morale.

 Entry of nationalists would deter the Government from stuffing the councils with undesirable elements who may be used to provide legitimacy to government measures.

 Their only intention was to use the councils as arena of political struggle; they had no intention to use the councils as organs for gradual transformation of colonial rule. No-Changers' Arguments  No-Changers argued that parliamentary work would lead to neglect of constructive work, loss of revolutionary zeal & to political corruption.  Constructive work would prepare everyone for the next phase of civil disobedience.  Both sides wanted to avoid a SURAT & kept in touch with Gandhi who was in jail.  Both sides also realised the significance of putting up a united front to get a mass movement to force the government to introduce reforms, & both sides accepted the necessity of Gandhi's leadership of a united nationalist front.  Keeping these factors in mind, a compromise was reached at a meeting in Delhi in September 1923.  Swarajists were allowed to contest elections as a group within the Congress.  Swarajists accepted the Congress programme with only 1 difference—that they would join legislative councils.

 The elections to the newly constituted Central Legislative Assembly & to provincial assemblies were to be held in November 1923. Swarajist Activity in Councils  By 1924, the Swarajist position had weakened because of widespread communal riots, split among Swarajists themselves on communal & Responsivist-Non- responsivist lines, & the death of C.R. Das in 1925 weakened it further.

 Responsivists among Swarajists—Lalaji , Malviya & N.C. Kelkar —advocated cooperation with the Government & holding of office wherever possible to protect the so called Hindu interests.  They accused Non-responsivists like M Nehru of being anti-Hindu & beef-eater.

 Thus, the main leadership of the party reiterated faith in mass civil disobedience & withdrew from legislatures in March 1926, while another section of Swarajists went into the 1926 elections as a party in disarray & did not fare well.

 In 1930, the Swarajists finally walked out as a result of the Congress resolution on purna swaraj & the beginning of the Civil Disobedience Movement . Achievements  With coalition partners, they outvoted the Government several times, even on matters relating to budgetary grants, & passed adjournment motions.  They agitated through powerful speeches on self government, civil liberties & industrialization.  Vithalbhai Patel was elected speaker of Central Legislative Assembly in 1925.

 They defeated the Public Safety Bill in 1928 which was aimed at empowering the Government to deport undesirable & subversive foreigners (because Government was alarmed by the spread of socialist & communist ideas).

 By their activities, they filled the political vacuum at a time when the national movement was recouping its strength.

 They exposed the hollowness of the Montford scheme.  They demonstrated that the councils could be used creatively. Constructive Work by No-Changers  Ashrams sprang up where young men & women worked, among tribals & lower castes (especially in Kheda & Bardoli areas), & popularized charkha and khadi.

 National schools and colleges were set up where students were trained in a non- colonial ideological framework.

 Significant work was done for Hindu-Muslim unity, removing untouchability, boycott of foreign cloth & liquor, & for flood relief.

 The constructive workers served as the backbone of civil disobedience as active organizers. Simon Commission  In 1927, the British appointed the Indian Statutory Commission headed by Sir John Simon  At the time of introducing the Montford Reforms the British Government declared that a commission would be sent to India after 10 years to examine the effects & operations of the constitutional reforms & to suggest more reforms for India.

 The purpose of this mission was:-

 To review the Government of India Act, 1919

 To suggest legal & administrative reforms for the future

 In 1928, the members of the commission reached India to widespread protests.

 The main reason for protests was that the mission didn’t include any Indian. Continued  Commission was a group of 7 British Members of Parliament of UK.

 One of its members was Clement Attlee

 The Commission published its 2-volume report in May 1930.

 It proposed the abolition of diarchy & the establishment of representative government in the provinces.

 It also recommended that separate communal electorates be retained, but only until tensions between Hindus & Muslims had died down.

 The outcome of the Simon Commission was the Government of India Act 1935. 2013 The people of India agitated against the arrival of Simon Commission because a) Indians never wanted the review of the working of the Act of 1919 b) Simon Commission recommended the abolition of Dyarchy (Diarchy) in the Provinces c) there was no Indian member in the Simon Commission d)the Simon Commission suggested the partition of the country

The response (to formation of Simon Commission) in India was immediate and unanimous, that no Indian should be thought fit to serve on a body which claimed the right to decide the political future of India, was an insult that no Indian of even most moderate political opinion was willing to swallow. Answer C. Nehru Committee  Secretary of State, Lord Birkenhead challenged the Indians to produce a constitution that would be acceptable to all.

 Purpose of this committee was to form a draft constitution for the country

 The first meeting was held at Delhi, followed by & Lucknow

 A 8 member Sub Committer headed by Motilal Nehru was formed.

 Other prominent members were Tej Bahadur Sapru, Sir Ali Imam, M.R. Jayakar & S C Bose

 The report was placed before Congress Session in Calcutta in 1928 where it was adopted unanimously.

Recommendations  Dominion Status

 Secular State

 Universal Adult Suffrage

 To end separate electorate

 Reservations of seats for minorities

 Residuary powers to the centre Reaction to the report

 Jawaharlal Nehru, opposed dominion status.

 Young leaders like S C Bose in Congress & League demanded ‘Poorna Swaraj’ or complete independence.

 Jinnah & President of Central Sikh league, Sardar Kharak Singh rejected it.

 Later Jinnah convened an All India Conference of Muslims & drew up a list of 14 point.

Jinnah’s Fourteen Point Program 5 key demands

 Separate electorate

 Reservation of seats for Muslims

 One-third reservation in Union Parliament

 Weak Centre and strong States

 If 4/5th members of a particular community oppose a bill; it shall not be passed 2011 With reference to the period of Indian freedom struggle, which of the following was/were recommended by the Nehru report ? 1.Complete independence for India. 2.Joint electorates for reservation of seats for minorities. 3.Provision of fundamental rights for the people of India in the constitution. Select the correct answer using the codes given below : (a)1 only.

(b)2 and 3 only.

(c)1 and 3 only. (d)1,2 and 3. Nehru Report did not provide for separate electorates for any community or weightage for minorities, but yes it talks of Joint electorates. It also talks for Bills of rights so, the correct answer is B Deepawali declaration

 The suggestion of Sir John Simon(suggestion was to convene a conference of the representatives of both British India & the Indian states to take a final decision on the question of constitutional reforms for India) was accepted by the British cabinet (Ramsay MacDonald was British PM) & subsequently Irwin, made this famous declaration’(October 31,1929) according to which

 the objective of British policy was to grant Dominion status to India &

 a round table conference would be held in London after the Simon commission had reported.

 In opposition to this then Lahore Resolution from Congress was adopted Lahore Session

 In this session Congress demanded ‘Poorna Swaraj’ (complete independence) for the 1st time.

 The session was presided by Jawaharlal Nehru. Gandhi supported Jawaharlal Nehru in this session.

 On 31 December,1929 the nationalists gathered at river Ravi & hoisted the tricolour flag

 On 26 January, 1930 they celebrated . 2012 The Lahore session of the Indian National Congress, 1929 is very important in history because:

1. Congress passed a resolution demanding complete independence

2. Rift between the extremists and moderates was resolved in that session

3. A resolution was passed rejecting the Two Nation theory in that session

Which of the statements given above is/are correct? a) 1 only b) 2 and 3 c) 1 and 3 d) None of the above Answer: A Butler Committee 1927 Extent of sovereignty of the Paramount power were not yet settled properly, so, relations between Indian Princes & the crown were not well defined.

The Indian states committee was appointed to investigate & clarify the relationship between the paramount power & the Princes.

To suggest ways & means for more satisfactory adjustments of the existing relations between them & British India.

Sir Harcourt Butler was its chairman 2017 The object of the Butler Committee of 1927 was to

(a) Define the jurisdiction of the Central and Provincial Governments.

(b) Define the powers of the Secretary of State for India.

(c) Impose censorship on national press.

(d) Improve the relationship between the Government of India and the Indian States.

Answer d HISTORY March 22, 2018 Mohandas Karamchand Gandhi  Born October 2,1869 Porbander in of

 Went to South Africa for his client Dada Abdullah

 His first opposition came against the British Rule in South Africa where he was thrown out of 1st class compartment of a train as in South Africa only whites were allowed to travel in 1st class.

Moderate Phase of Struggle (1894-1906)

 During this phase, Gandhi relied on sending petitions and memorials to the authorities in South Africa & in Britain, hoping that once the authorities were informed of the plight of Indians, they would take sincere steps to redress their grievances as the Indians were, after all, British subjects.  His attempt was to unite the different sections of Indians, and to give their demands wide publicity. Phase of Passive Resistance or Satyagraha (1906-1914)  The second phase, which began in 1906, was characterized by the use of the method of passive resistance or civil disobedience, which Gandhi named satyagraha. Satyagraha against Registration Certificates (1906)

 A new legislation in South Africa made it compulsory for Indians there to carry at all times certificates of registration with their fingerprints.  Indians organised a meeting on 11 September, 1906, in the Empire Theatre in Johannesburg under Gandhi's leadership & decided not to submit to this discriminatory measure. Gandhi formed the Passive Resistance Association to conduct the campaign.  General Smuts called Gandhiji for talks, & promised to withdraw the legislation if Indians voluntarily agreed to register themselves.  Gandhiji accepted & was the first to register. But Smuts had played a trick; he ordered that the voluntary registrations be ratified under the law.  The Indians under the leadership of Gandhi retaliated by publicly burning their registration certificates. Campaign against Restrictions on Indian Migration

 The earlier campaign was widened to include protest against a new legislation imposing restrictions on Indian migration.

 The Indians defied this law by crossing over from one province to another & by refusing to produce licences.

 Many of these Indians were jailed. Setting up of Tolstoy Farm  As it became rather difficult to sustain the high pitch of the struggle, Gandhi decided to devote all his attention to the struggle.

 The funds for supporting the families of the Satyagrahis & for running Indian Opinion were fast running out.  Gandhiji’s own legal practice had virtually ceased since 1906, the year he had started devoting all his attention to the struggle.  At this point, Gandhiji set up Tolstoy Farm, made possible through the generosity of his German architect friend, Kallenbach, to house the families of the Satyagrahis & give them a way to sustain themselves.

 Tolstoy Farm was the precursor of the later Gandhian ashrams that were to play so important a role in the Indian national movement.  Funds also came from India — Sir Ratan Tata sent Rs. 25,000 & the Congress & the League, as well as the Nizam of , made their contributions. Campaign against Poll Tax and Invalidation of Indian Marriages

 A poll tax of 3 pounds was imposed on all ex indentured Indians

 The inclusion of demands for the abolition of poll tax (which was too much for the poor ex-indentured Indians who earned less than ten shillings a month) in the ongoing struggle further widened the base of the campaign.

 Fuel was added to the fire by a Supreme Court order which invalidated all marriages not conducted according to Christian rites and by the registrar of marriages.

 By implication, Hindu, Muslim & Parsi marriages were illegal & children born out of such marriages, illegitimate.

 The Indians treated this judgement as an insult to the honour of their women and many women were drawn into the movement because of this indignity.  The Indians protested by illegally migrating from Natal into Transvaal.  The campaign was launched by the illegal crossing of the border by a group of sixteen Satyagrahis, including Kasturba, who marched from Phoenix Settlement in Natal to Transvaal, & were immediately arrested.  Government held these Indians in jails. Miners & plantation workers went on a lightning strike.  In India, Gokhale toured the whole country mobilising public opinion in support of the Indians in South Africa.  Even the viceroy, Lord Hardinge, condemned the repression & called for an impartial enquiry.

 Eventually, through a series of negotiations involving Gandhi, Hardinge, C.F. Andrews and General Smuts, an agreement was reached by which the Government of South Africa conceded the major Indian demands relating to the poll tax, the registration certificates & marriages solemnised according to Indian rites, & promised to treat the issue of Indian immigration in a sympathetic manner.

2011 Mahatma Gandhi said that some of his deepest convictions were reflected in a book titled, “unto this last’’ and the book transformed his life. What was the message from the book that transformed mahatma Gandhi ? (a)Uplifting the oppressed and poor is the moral responsibility of an educated man (b)The good of individual is contained in the good of all. (c)The life of celibacy and spiritual pursuit are essential for a noble life. (d)All the statements (a), (b) and (c) are correct in this context. The correct Answer of this question is B. Gandhi in India  Returned on 9th January 1915

 He decided not to take any position on any political matter for at least 1 year.

 He was convinced about the limitations of moderate politics & was also not in favor of Home Rule agitation which was becoming popular at that time.  He thought that it was not the best time to agitate for Home Rule when Britain was in the middle of a war.  He was convinced that the only technique capable of meeting the nationalist aims was a non-violent satyagraha.

 He also said that he would join no political organisation unless it too accepted the creed of non-violent satyagraha.  During 1917-1918 involved in Champaran, Ahmedabad, Kheda before he launched Rowlatt satyagraha(his 1st all India movement) Champaran Movement  1917 (1st Civil Disobedience movement of Gandhi)

 Raj Kumar Shukla invited Gandhi to Champaran.

 The European planters had been forcing peasants to grow indigo on 3/20 of the total land (called tinkathia system).

 When towards the end of the 19th century German synthetic dyes replaced indigo, the European planters demanded high rents & illegal dues from the peasants in order to maximise their profits before the peasants could shift to other crops

 Also, the peasants were forced to sell the produce at prices fixed by the Europeans.

 Indigo Cultivation was destroying the productivity of the land which was the main reason of the peasant’s protest. Champaran Movement  Gandhi’s comrades in this movement were – Rajendra Prasad, Mazhar-ul-Haq, Narahari Parekh, JB Kriplani, Mahadev Desai Brij Kishore.  Authorities ordered him to leave the area at once but he defied & preferred to face punishment.  Gandhi was arrested but then later released by the magistrate.  This passive resistance of an unjust order was a novel method at that time.  Finally, the authorities retreated and permitted Gandhi to make an enquiry.  Government formed an Indigo commission with Gandhi as a member.  Gandhi was able to convince the authorities that tinkathia should be abolished & that the peasants should be compensated for the illegal dues extracted from them.  As a compromise with the planters, he agreed that only 25 % of the money taken should be compensated.  Also no force can be used to make them grow Indigo.  Within a decade, the planters left the area.  Gandhi had won the 1st battle of civil disobedience in India. Ahmedabad Mill strike – 1918  Unlike Champaran, in this case both the workers and employers were Indians.

 The workers demanded that bonus given to eradicate plague must be continued whereas Industrialists stopped the bonus as according to them plague had stopped.

 Gandhi advised the workers to go on strike.  This was the 1st strike called by Gandhi in India.  It was changed to fast unto death as the numbers were dwindling.

 Hunger strike proved to be very successful as it got a lot of attention.

 Industrialist referred the issue to a tribunal which awarded 35% plague bonus to the workers.

 Ambalal Sarabhai’s sister, Anasuya Behn, was one of the main lieutenants of Gandhiji in this struggle in which her brother was one of the main adversaries. Kheda Satyagraha – 1918

 The peasants in Kheda, complained that they could not pay land revenue as there was drought & hence no crop output.

 According to a 20th century Revenue Code of British Government law, land revenue should not be collected from a peasant who has lost more than 50% of the produce.

 Statements of the peasants were observed by Gandhi & he asked the peasants not to pay taxes.

 The government observed this & passed a circular stating that revenue must be collected only from those who were in a position to pay.

 Vallabhbhai Patel left his lucrative practice at the Bar at this time to help Gandhi

 Indulal Yagnik became Gandhi follower. 2011 What was the reason for Mahatma Gandhi to organize a satyagraha on behalf of the peasants off Kheda ?

1. The administration did not suspend the land revenue collection in spite of a drought.

2. The administration proposed to introduce permanent settlement in Gujarat.

Which of the statements given above is/are correct ?

(a) 1 only (b) 2 only. (c) Both 1 and 2. (d) Neither 1 nor 2. Only 1 is correct statement in this question. Rowlatt Act 1919  Government on one hand gave carrot of constitutional reforms in form of GoI Act 1919 on the other armed itself with extraordinary powers to suppress the violators  Also known as Anarchical & Revolutionary Crimes Act, 1919  The was a Sedition Committee appointed in 1918 by the British Indian Government with Mr , an English judge, as its president.  It was a legislative act passed by the Imperial Legislative Council in Delhi, indefinitely extending the emergency measures of preventive indefinite detention, incarceration without trial and judicial review enacted in the Defence of India Act 1915 during the 1st World War.  According to this act any Indian could be arrested on the charge of sedation even on suspicious ground and without any trail could be jailed for up to 2 years. Thus suspended right of “habeas corpus” which is foundation of the civil liberty  Slogan of the movement was “No appeal, no daleel, no wakeel”  Mohammad Ali Jinnah resigned from legislative council. Continued  Having seen the constitutional protest fail, Gandhi organised a Satyagraha Sabha Sarvadharma Prarthana Sabha (prayer meeting for all religions) & roped in younger members of Home Rule Leagues & the Pan Islamists.

 The hartal was accompanied by fasting & prayer, & civil disobedience against specific laws, & courting arrest & imprisonment.

 6 April 1919 was the 1st “All India Strike Day”

 Before April 6 itself mass scale resistance grew against British rule , especially in Punjab due to many reasons so Army was called which put martial law there 2015 With reference to Rowlatt Satyagraha, which of the following statements is / are correct ?

1) The Rowlatt Act was based on the recommendations of the ‘Sedition Committee’.

2) In Rowlatt Satyagraha, Gandhiji tried to utilize the Home Rule League.

3) Demonstrations against the arrival of Simon Commission coincided with Rowlatt Satyagraha. Select the correct answer using the code given below. (a) 1 only (b) 1 and 2 only (c) 2 and 3 only (d) 1, 2 and 3 Answer – (B) 2012 The Rowlatt act aimed at: a) compulsory economic support to war efforts b) imprisonment without trial and summary procedures for trial c) suppression of the Khilafat movement d) imposition of restrictions on freedom of the press

Answer: B Jallianwala Bagh Massacre  Jallianwala was a park in Amritsar

 There was a curfew imposed in Amritsar on 13 April but people from rural area had no knowledge about it.

 13 April was celebrated as Baisakhi festival & peasants had come to celebrate at Jallianwala Bagh.

 Incidentally a group of protestors protesting the arrest of 2 leaders Dr. Saifuddin Kitchlew & Dr. Satypal Malik had also come to the same place. Jallianwala Bagh Massacre – 13 April 1919  General Dyer entered the place and ordered his men to fire at the crowd killing 379 people (according to government records).

 Media & local people said that number of people killed were more than 1000.

 Hunter Commission was setup to look into the event.

returned his knighthood when House of Lords called this act by Gen Dyer as act of bravery.

 Gandhi withdrew the movement on April 18, 1919. Assassination of Michael O'Dwyer  On 13 March 1940, at Caxton Hall in London, Udham Singh, who had witnessed the events in Amritsar & was himself wounded, shot & killed Michael O'Dwyer, the British Lieutenant- Governor of Punjab at the time of the massacre, who had approved Dyer's action & was believed to be the main planner.

 Dyer himself had died in 1927.

 Udham Singh was hanged for the murder on 31 July 1940.

 At that time, many, including Nehru & Gandhi, condemned the action as senseless but courageous.

 In 1952, Nehru honored Udham Singh & soon after this recognition by the PM, Singh received the title of Shaheed Montagu's Statement-August 1917

"The government policy is of an increasing participation of Indians in every branch of administration and gradual development institutions with a view to the progressive realization of responsible government in India as an integral part of the British empire

Importance of Montagu's Statement From now onwards, the demand by nationalists for self-government or Home Rule could not be termed as seditious since attainment of self government for Indians now became a government policy, unlike Morley's statement in 1909 that the reforms were not intended to give self-government to India. Indian Objections  The objections of the Indian leaders to Montagu's statement were 2 fold

 No specific time frame was given.

 The Government alone was to decide the nature & the timing of advance towards a responsible government, & the Indians were resentful that the British would decide what was good & what was bad for Indians. Government of India Act 1919  Based on Montagu – Chelmsford Reforms in line with Montague’s August 1917 statement

 As the British Government needed India’s help in terms of both resources & manpower for WW-1, they promised the Indian leaders that more number of members will be included in the Executive council.

Provisions  relaxation of central control over provinces by demarcating & separating the central & provincial subjects. The central & provincial legislatures were authorised to make laws on their respective list of subjects. However, the structure of government continued to be centralised and unitary.

 Bicameral Legislature at the Centre : Thus, the Indian Legislative Council was replaced by a bicameral legislature consisting of an Upper House (Council of State) & a Lower House (Legislative Assembly). The majority of members of both the Houses were chosen by direct election.  Legislative assembly & Council of state to consist of 140 & 60 members

 Separate Electorates also given to Sikhs Anglo Indians,Indian Christians, & Europeans.

 Legislators could ask questions & supplementaries, pass adjournment motions & vote a part of the budget, but 75% of the budget was still not votable. Dyarchy  Dyarchy (rule of 2) in the province .‘dyarchy’—a term derived from the Greek word di-arche which means double rule. This experiment was largely unsuccessful.  Popular ministers & governors to be executive head

 Reserved & transferred (by ministers) subjects.  The reserved subjects, were to be administered by the governor & his executive council without being responsible to the legislative Council.

 Reserved subjects such as finance, Law & order , land revenue, irrigation

 Transferred subjects such as health, education, industry, local government.

 The transferred subjects were to be administered by the governor with the aid of ministers responsible to the legislative Council.

 Governor could veto the bill & issue ordinances  Women were given right to vote

 Indian legislature made more representative

 It required that the three of the 6 members of the Viceroy’s executive Council (other than the commander-in-chief) were to be Indian.

 It created a new office of the High Commissioner for India in London & transferred to him some of the functions hitherto performed by the Secretary of State for India.  It provided for the establishment of a public service commission. Hence, a Central Public Service Commission was set up in 1926 for recruiting civil servants

 It separated, for the first time, provincial budgets from the Central budget & authorised the provincial legislatures to enact their budgets.

 It provided for the appointment of a statutory commission to inquire into & report on its working after 10 years of its coming into force. 2012 Which of the following is/ are the principal feature(s) of the Government of India Act, 1919?

1. Introduction of dyarchy in the executive government of the provinces.

2. Introduction of separate communal electorates for Muslims

3. Devolution of legislative authority by the centre to the provinces

Select the correct answer using the codes given below: a) 1 only b) 2 and 3 only c) 1 and 3 only d) 1, 2 and 3 Answer: C 2015 The Government of India Act of 1919 clearly defined a) the separation of power between the judiciary and the legislature. b) the jurisdiction of the central and provincial governments. c) the powers of the Secretary of State for India and the Viceroy. d) None of the above

Answer – (B) 1919 Act introduced Diarchy, taking small steps towards a decentralized unitary form of government. Diarchy involves some sharing of power between the union government and the provinces. The act provided for – Matters of administration were to be first divided between the centre and the provinces and then the provincial subjects to be further bifurcated into transferred and reserved subjects. 2016 The Montague-Chelmsford Proposals were related to

(a) social reforms

(b) educational reforms

(c) reforms in police administration

(d) constitutional reforms

Answer d 2017 In the context of Indian History, the principle of ‘Dyarchy (diarchy) refers to

(a) Division of the central legislature into two houses

(b) Introduction of double government i.e., Central and State governments.

(c) Having two sets of rulers; one in London and another in Delhi.

(d) Division of the subjects delegated to the provinces into two categories.

Answer d HISTORY March 22, 2018 Growth Of Militant Nationalism 1) Recognition of the True Nature of British Rule:  Having seen that, the Government was not conceding any of their important demands, the more militant among those politically conscious got disillusioned and started looking for a more effective mode of political action.

 Also, the feeling that only an Indian Government could bring India on a path of progress started attracting more and more people.

 The economic miseries of the 1890s further exposed the exploitative character of colonial rule.  Severe famines killed 90 lakh persons between 1896 & 1900.  Bubonic plague affected large areas of the Deccan leading to large-scale riots there.

 The nationalists were wide awake to the fact that instead of giving more rights to the Indians, the Government was taking away even the existing ones. Continued  1892 Indian Councils Act was criticized by nationalists as it failed to satisfy them.

 1897 The Natu brothers were deported without trial & Tilak & others, imprisoned on charges of sedition.

 1898 Repressive laws under IPC Section 124 A were further amplified with new provisions under IPC Section 156 A  1899 Number of Indian members in Calcutta Corporation were reduced.

 1904 Official Secrets Act curbed freedom of press.

 Indian Universities Act ensured greater government control over universities, which it described as factories producing political revolutionaries.  Also British rule was no longer progressive—socially & culturally. It was suppressing the spread of education, especially mass & technical education. 2) Growth of Confidence and Self-Respect: A feeling started gaining currency that only the masses were capable of making the immense sacrifices needed to win freedom. 3) Growth of Education: Led to an increased awareness among the masses & the rise in unemployment & underemployment among the educated drew attention to poverty & the underdeveloped state of the country's economy under colonial rule. 4) International Influences:  Progress made by Japan & its emergence as an industrial power  The defeat of the Italian army by Ethiopians (1896), the Boer wars (1899-1902) where British faced reverses & Japan's victory over Russia (1905) demolished myths of European invincibility.  Also, the nationalists were inspired by the nationalist movements worldwide—in Ireland, Russia, Egypt, Turkey, Persia & China.

5) Reaction to Increasing Westernisation: The new leadership felt the stranglehold of excessive westernisation & sensed colonial designs to submerge the Indian national identity in the British Empire Continued 6) Dissatisfaction with Moderates: The younger elements within the Congress were dissatisfied with the achievements of the Moderates 1st 15-20 years. They were strongly critical of the methods of peaceful & constitutional agitation, popularly known as the “3 'P's"— prayer, petition & protest—& described these methods as 'political mendicancy'.

7. Reactionary Policies of Curzon: He spoke derogatorily of Indian character in general.

Administrative measures adopted during his rule— the Official Secrets Act, the Indian Universities Act & above all, the partition of Bengal left no doubts in Indian minds about the basically reactionary nature of British rule in India. Partition of Bengal

(16 Oct 1905)

Lord Curzon was the brain behind this partition but by the time actual partition took place, Lord Minto II had replaced Lord Curzon as the Viceroy. Reasons for Partition  British Government Explanation

 Bengal was a very big state. Very difficult to manage the administrative work

 Nationalist View

 Divide & Rule Policy as Calcutta was the hub of Nationalistic activities  Division of people in terms of religious and linguistic demarcations  If ease of administration was the only reason why wasn’t Punjab, Bombay or Madras partitioned (i) on the basis of language

(thus reducing the Bengalis to a minority in Bengal itself as in the new proposal Bengal proper was to have 17 million Bengalis & 37 million Hindi & Oriya speakers)

(ii) on the basis of religion as the western half was to be a Hindu majority area (42 million out of a total 54 million) & the eastern half was to be a Muslim majority area (18 million out of a total of 31 million) Effect of this demarcation  Created divide amongst residents in terms of Hindu & Muslim religion

 Created a divide in within Hindi, Oriya & Bengali speaking population

 Changed the Fusion to Fission Culture i.e. from collective thinking to individualism or community ideology Period of 1903-05  Government's decision to partition Bengal had been made public in December 1903

 During this period, the leadership was provided by Surendranath Banerjee, K.K. Mitra & Prithwishchandra Ray.

 The methods adopted were petitions to the Government, public meetings, memoranda, & propaganda through pamphlets & newspapers such as Hitabadi, Sanjibani & Bengalee.

Swadeshi Movement (After-effects of Partition of Bengal)  People came out on street to protest against decision of partition.

 They assembled at Town Hall & formed Swadeshi Bandhav Samiti which propagated swadeshi goods and services

 Local industries like textile, paper, glass flourished during this period

 Lawyers resigned from the bar councils & Panchayats started hearing the grievances of people

 Social boycott

 Women, who were traditionally home-centred, especially those of the urban middle classes, took active part in processions and picketing. Students  Teachers & students boycotted English schools

 Students came out in large numbers to propagate and practise swadeshi, & to take a lead in organising picketing of shops selling foreign goods.

 Police adopted a repressive attitude towards the students.

 Schools and colleges whose students participated in the agitation were to be penalised by disaffiliating them or stopping of grants & privileges to them.

Swadeshi Movement  Soon, the movement spread to other parts of the country—

 In Poona & Bombay under Tilak

 In Punjab under Lala Lajpat Rai & Ajit Singh

 In Delhi under Syed Haider Raza

 Madras under Chidambaram Pillai.  It was also galvanized by Bipin Chandra Pal’s extensive lecture tour.

 In response to the British India Steam Navigation Company's trade monopoly, Chidambaram started an Indian-owned shipping company. He registered the Swadeshi Shipping Company in October 1906.  Rawalpindi, Kangra, Jammu, Multan & Haridwar witnessed active participation in the Swadeshi Movement. Extent of mass participation  Some of the Muslims participated—Barrister Abdul Rasul, Liaqat Hussain, Guznavi, Azad (who joined one of the revolutionary terrorist groups)—but most of the upper & middle class Muslims stayed away or, led by Nawab Salimullah of Dacca, supported partition as it would give them a Muslim-majority East Bengal.

 To further government interests, League was propped up as an anti-Congress front & reactionary elements like Nawab were encouraged.

 Thus, the social base of the movement expanded to include certain sections of the zamindars, the students, the women, & the lower middle classes in cities & towns. Features of Swadeshi movements  It was the First Movement under the Indian National Congress.

 Class composition of the movement was everyone from the high class to low class due to which it was not only an elitist movement.

 Mobilization of masses was done by the leaders using religious symbols & organizing festivals which led to alienation of religious minority. Programme of swadeshi or national education  One of the major planks of the programme of self reliance  Bengal National College, inspired by Tagore's Shanti Niketan was set up with Aurobindo Ghosh as its principal.

 On August 15, 1906, the National Council of Education was set up to organize a system of education—literary, scientific & technical—on national lines & under national control from the primary to the university level.

 Education was to be imparted through the medium of vernaculars.

 A Bengal Institute of Technology was set up for technical education & funds were raised to send students to Japan for advanced learning.

 Scores of national schools sprang up all over the country within a short period. Swadeshi or indigenous enterprises  Self-reliance also meant an effort to set up Swadeshi or indigenous enterprises.

 The period saw a mushrooming of Swadeshi textile mills, soap & match factories; - tanneries, banks, insurance companies, shops, etc.

 While many of these enterprises, whose promoters were more endowed with patriotic zeal than with business acumen were unable to survive for long, some others such as Acharya P.C. Ray’s Bengal Chemicals Factory, became successful & famous. Impact in the cultural sphere  It was, perhaps, in the cultural sphere that the impact of the Swadeshi Movement was most marked.  The nationalists of all hues took inspiration from songs written by Tagore, Rajnikanta Sen, Dwijendralal Ray, Mukunda Das, Syed Abu Mohammad etc.

 Tagore's' Amar Sonar Bangla written on this occasion.

 In painting, Abanindranath Tagore broke the domination of Victorian naturalism over Indian art and took inspiration from Mughal, Ajanta & Rajput paintings.

 In science, Jagdish Chandra Bose, Prafulla Chandra Roy & others pioneered original research which was praised the world over. Nandlal Bose  Nandlal Bose, who left a major imprint on Indian art, was the first recipient of a scholarship offered by the Indian Society of Oriental Art, founded in 1907  To mark the 1930 occasion of Mahatma Gandhi's arrest for protesting the British tax on salt, Bose created a black on white linocut print of Gandhi walking with a staff. It became the iconic image for the non-violence movement.

 He was also famously asked by Jawaharlal Nehru to sketch the emblems for the Government of India's awards, including the Bharat Ratna & the Padma Shri.

 Along with his students, Nandalal Bose took up the historic task of beautifying/decorating the original manuscript of the . Moderates  Thus, with the coming of Swadeshi & Boycott Movement, it became clear that the Moderates had outlived their utility and their politics of petitions & speeches had become obsolete.

 They had not succeeded in keeping pace with time, & this was highlighted by their failure to get the support of the younger generation for their style of politics.

 No all- India campaigns of the scale of Swadeshi & Boycott Movement had been organized earlier by the Moderates & in this campaign, they discovered that they were not its leaders, which was rather natural. Extremist  The Extremist ideology and its functioning also lacked consistency.

 Its advocates ranged from open members & secret sympathisers to those opposed to any kind of political violence.

 Aurobindo, Tilak, B.C. Pal & Lalaji had different perceptions of their goal.

 For Tilak, swaraj meant some sort of self-government, while for Aurobindo, it meant complete independence from foreign rule.

 But at the politico-ideological level, their emphasis was on mass participation & on the need to broaden the social base of the movement.

 They raised patriotism from a level of 'academic pastime' to one of 'service & sacrifice for the country'. Continued  But the politically progressive Extremists proved to be social reactionaries.

 They had revivalist & obscurantist undertones attached to their thoughts.

 Tilak's opposition to the Age of Consent Bill (which would have raised the marriageable age for girls from 10 years to 12 years, though his objection was mainly that such reforms must come from people governing themselves & not under an alien rule), his organising of Ganapati & Shivaji festivals as national festivals, his support to anti-cow killing campaigns etc. portrayed him as a Hindu nationalist.

 Similarly B.C. Pal & Aurobindo spoke of a Hindu nation & Hindu interests.

2016 The `Swadeshi’ and ‘Boycott’ were adopted as methods of struggle for the first time during the

(a) agitation against the Partition of Bengal

(b) Home Rule Movement

(c) Non-Cooperation Movement

(d) visit of the Simon Commission to India

Answer a Moderates Extremists Believed & practiced through legal means Practiced boycott & mass movement Wanted changes in system like constitutional Demanded Swaraj & mass reforms & share for Indians in services but not mobilization Swaraj Social base—zamindars & upper middle classes Social base educated middle classes in towns. in towns & lower middle class. Believed in England's providential mission in Rejected 'providential mission India. theory' as an illusion. Focus of the movement was only in Bengal Wanted an all India Movement Ideological inspiration— western liberal thought Indian history, cultural heritage & & European history. Hindu traditional symbols. Professed loyalty to the British Crown. Believed British Crown was unworthy of Indian loyalty. Believed that the movement should be limited to Had immense faith in the capacity middle class intelligentsia; masses not yet ready of masses to participate & to make for participation in political work. sacrifices. Benaras session 1905  Presided by Gokhale, the Moderate-Extremist differences came to the fore.

 Extremists wanted to extend the Boycott & Swadeshi to outside Bengal & also to include all forms of associations (such as government service, law courts, legislative councils, etc. within the boycott & thus start a nationwide mass movement.

 The extremists wanted a strong resolution supporting the programme at the session.

 Moderates were not in favor of extending the movement beyond Bengal & were totally opposed to boycott of councils & similar associations.  They advocated strictly constitutional methods to protest against the partition.

 As a compromise, a relatively mild resolution condemning the partition & the reactionary policies of Curzon & supporting the swadeshi & boycott in Bengal was passed. Calcutta session 1906  Moderate enthusiasm had cooled a bit because of the popularity of the Extremists and the revolutionary terrorists & because of communal riots.

 Extremists wanted either Tilak or Lajpat Rai as the president, while the Moderates proposed the name of Dadabhai.

 Finally, Dadabhai was elected as the president and as a concession to the militants, the goal of the Congress was defined as Swarajya.

 Also 4 compromise resolutions supporting the programme of swadeshi, boycott , Self-Government & national education was passed.

 The word swaraj was mentioned for the 1st time, but its connotation was not spelt out, which left field open for differing interpretations by Moderates & Extremists. The Surat Split  Extremists: Emboldened by the Calcutta session, called for wide passive resistance & boycott of schools, colleges, legislative councils, municipalities, courts, etc.

 Thought that the people had been aroused and the battle for freedom had begun. Felt the time had come for the big push to drive the British out & considered the Moderates to be a drag on the movement.

 Moderates: Encouraged by the news that council reforms were on the anvil, decided to tone down the Calcutta programme.

 Thought that it would be dangerous at that stage to associate with the Extremists whose anti-imperialist agitation, it was felt, would be ruthlessly suppressed by the mighty colonial rule.  They saw in the council reforms an opportunity to realise their dream of Indian participation in the administration. They felt any hasty action under extremist pressure was bound to annoy the Liberals in power in England then. Continued

 The Moderates did not realise that the council reforms were meant by the Government more to isolate the extremists than to reward the moderates.

 Extremists did not realise that the Moderates could act as their outer line of defence in face of state repression.

 Both sides did not realize that in a vast country like India ruled by a powerful imperialist country, only a broad-based nationalist movement could succeed.

 Extremists wanted 1907 session to be held in Nagpur with Tilak or Lajpat Rai as the president and reiteration of the swadeshi, boycott and national education resolutions.

 The moderates wanted the session at Surat in order to exclude Tilak. They wanted Rashbehari Ghosh as the president & sought to drop the 4 resolutions. Continued  The split became inevitable, & the Congress now dominated by the Moderates lost no time in reiterating Congress commitment to the goal of self government within the British Empire & to constitutional methods only to achieve this goal.

 Between 1907 and 1911,new laws were enforced to check anti-government activity.

 These legislations included the

 Seditious Meetings Act, 1907;  Indian Newspapers (Incitement to Offences) Act, 1908;  Criminal Law Amendment Act, 1908;  Indian Press Act, 1910. 2015 Which one of the following movements has contributed to a split in the Indian National Congress resulting in the emergence of `moderates’ and ‘extremists’?

(a) Swadeshi Movement

(b)

(c) Non-Cooperation Movement

(d) Civil Disobedience Movement

Answer – (A) All other movements mentioned in the question had occurred after 1919. 2016 What was the main reason for the split in the Indian National Congress at Surat in 1907?

(a) Introduction of communalism into Indian politics by Lord Minto

(b) Extremists’ lack of faith in the capacity of the moderates to negotiate with the British Government

(c) Foundation of Muslim League

(d) Aurobindo Ghosh’s inability to be elected as the President of the Indian National Congress

Answer b The Government Strategy

 The Government in India had been hostile to the Congress from the beginning.

 Even after the Moderates, who dominated the Congress from the beginning, began distancing themselves from the militant nationalist trend which had become visible during the last decade of the 19th century itself, government hostility did not stop.

 This was because, in the Government's view, the Moderates still represented an anti- imperialist force consisting of basically patriotic and liberal intellectuals.

 With the coming of Swadeshi & Boycott Movement & the emergence of militant nationalist trend in a big way, the Government modified its strategy towards the nationalists.

 A three pronged approach of repression, conciliation, suppression. Carrot and Stick  In the 1st stage extremists were to be repressed mildly, mainly to frighten moderate

 In the 2nd stage, the moderates were to be placated through some concessions, & hints were to be dropped that more reforms would be forthcoming if the distance from the extremists was maintained. This was aimed at isolating the extremists.

 Now, with the moderates on its side, the Government could suppress the extremists with its full might. The moderates could then be ignored.

 Unfortunately, neither moderates nor the extremists understood the implications of the strategy.

 Surat split suggested that the policy of carrot & stick had brought rich dividends to the Government. Afterwards  Government immediately launched a massive attack on the Extremists.  Extremist newspapers were suppressed.  Between 1907 & 1908, 9 major leaders in Bengal including Ashwini Kumar Dutt & Krishna Kumar Mitra were deported, Ajit Singh was deported & Chidambaram Pillai & Harisarvottam Rao from Madras & Andhra were arrested  Tilak, was sent to Mandalay (Burma) jail for 6 years.  Aurobindo & B.C. Pal retired from active politics.  Lajpat Rai, who had been a helpless onlooker at Surat, left for Britain in 1908 to come back in 1909 & then to go off to the US for an extended stay.  Extremists were not able to organize an effective alternative party to sustain the movement.  Moderates were left with no popular base or support, especially as the youth rallied behind, the extremists.  The Government had won, at least for the moment.’

 In 1914, Tilak was released & he picked up the threads of the movement. Why Swadeshi Movement fizzled out by 1908?  Severe government repression.  With arrest or deportation of all leaders, the movement left leaderless

 Narrow social base: largely remained confined to the upper & middle classes and zamindars , & failed to reach the masses—especially the peasantry.  Surat split

 The movement failed to create an effective organisation or a party structure. It threw up an entire gamut of techniques that came to be associated with Gandhian politics—non cooperation, passive resistance, filling of British jails, social reform & constructive work—but failed to give these techniques a disciplined focus.

 The movement aroused the people but did not know how to tap the newly released energy or how to find new forms to give expression to popular resentment.  It is difficult to sustain a mass-based movement at a high pitch for too long. Muslim League  Formed at Dhaka in 1906  Sir Sultan Muhammad Shah (Aga Khan III) was appointed the 1st Honorary President of the League.

 Aga Khan(Not a name but a spiritual post within Shia muslim community)

 Nawab Waqar ul Mulk & Nawab Mohsin ul Mulk(political heir) were the other notable names in its formation.  Simla deputation

 58 delegates from all over the Subcontinent were the founding members of League.

 The headquarters were established at Lucknow.  To preach loyalty to the British Government & to keep the Muslim intelligentsia away from the Congress. Important Leaders 

 Muhammad Ali Jauhar

 Shaukat Ali

 Hakim Ajmal Ansari : Sole person elected to the Presidency of the Congress, the Muslim League & the All India Khilafat Committee.

 Elected 1st chancellor of the Jamia Milia Islamia University

 Raja of Mehmoodabad Indian Council Act (1909) - Morley Minto Reforms  Act increased the number of elected members in the imperial & provincial legislative councils. Most of the elected members were still elected indirectly.  Central Legislative council will be expanded to 68 members.  The number of members in the provincial legislative councils was not uniform.  1st attempt at introducing a representative & popular element:  At least 1 Indian in the Viceroy’s Executive Council (Satyendra Sinha was the 1st .He was appointed as the law member).  2 Indians will be made members of Indian Council office in London  Councils, for the 1st time, referred to as 'Legislative Councils'.  Separate Electorate for Hindu Zamindars, presidency corporations, universities Chamber of Commerce-Calcutta & Bombay & for Indian Muslims.  Act ‘legalised communalism’ & Minto came to be known as the Father of Communal Electorate. Continued

 Besides separate electorates for the Muslims, representation in excess of the strength of their population was accorded to the Muslims.

 Income qualification for Muslim voters was kept lower than that for Hindus.

 Powers of legislatures both at the center & in provinces were enlarged

 Legislatures could now pass resolutions (which may not be accepted)

 Ask questions & supplementaries

 Vote separate items in the budget but the budget as a whole could not be voted upon. Delhi Darbar, 1911  Was set up to welcome King George V.

 Decisions taken during this were

 Annulment of Partition of Bengal but Separation of Bihar & Orissa from Bengal

 Transfer of capital from Calcutta to Delhi in 1912 The Ghadr  The Ghadr Party was a revolutionary group organized around a weekly newspaper.  The Ghadr had its headquarters at San Francisco.  These revolutionaries included mainly ex-soldiers & peasants who had migrated from the Punjab in search of better employment opportunities.  They were based in the US & Canadian cities along the western (Pacific) coast.

 Pre-Ghadr revolutionary activity had been carried on by Ramdas Puri, G.D. Kumar, Taraknath Das, and Lala Hardayal who reached there in 1911.

 To carry out revolutionary activities, the earlier activists had set up a 'Swadesh Sevak Home' at Vancouver & 'United India House' in Seattle. Continued  Finally in 1913, the Ghadr was established.

 Ghadr programme was to organize assassinations of officials, publish revolutionary & anti-imperialist literature, work among Indian troops stationed abroad, procure arms & bring about a simultaneous revolt in all British colonies.

 The moving spirits behind the Ghadr Party were Lala Hardayal, Ramchandra, Bhagwan Singh, Kartar Singh Saraba, Bhai Parmanand.

 The ghadrites intended to bring about a revolt in India.

 Their plans were encouraged by 3 events in 1914— arrest & escape of Hardayal , the Maru incident & the outbreak of the First World War.  Created an explosive situation in the Punjab.

 Komagata Maru was the name of a ship which was carrying 376 passengers, mainly Sikh and Punjabi Muslim would-be immigrants, from Hongkong to Vancouver.

 They were turned back by Canadian authorities after 2 months of privation & uncertainty.

 Generally believed that the Canadians were influenced by the British Government.

 The ship finally anchored at Calcutta in September 1914.

 The inmates refused to board the Punjab-bound train.

 In the ensuing with the police at Budge Budge near Calcutta, 22 persons died. War  Inflamed by this and with the outbreak of the War, the Ghadr leaders decided to launch a violent attack on British rule in India.

 They urged fighters to go to India.

 Kartar Singh Saraba & Raghubar Dayal Gupta left for India.  Disappointment in Punjab  Rashbehari Bose & Sachin Sanyal were asked to lead the movement.

 The Ghadrites fixed February 21 ,1915 as the date for an armed revolt in Ferozepur, Lahore & the plan was foiled at the last moment due to treachery.  The authorities took immediate action, aided by the Defence of India Rules, 1915.

 Rebellion regiments were disbanded, leaders arrested ,deported & hanged.

 Rashbehari Bose fled to Japan from where he & Abani Mukherji made many efforts to send arms while Sachin Sanyal was transported for life.

 Apart from the Bengal terrorists and the Punjab Ghadrites, radical pan- Islamists— Ali brothers, Maulana Azad, Hasrat Mohani—were interned for years. 2014 The Ghadr (Ghadar) was a:

(a) revolutionary association of Indians with headquarters at San Francisco

(b) nationalist organization operating from Singapore

(c) militant organization with headquarters at Berlin

(d) communist movement for India’s freedom with head-quarters at Tashkent

Answer a Home Rule League Movement  The Home Rule Movement was the Indian response to the 1st World War in a less charged but a more effective way than the response of Indians living abroad which took the form of the romantic Ghadr adventure.

 The Indian Home Rule Leagues were organized on the lines of the Irish Home Rule Leagues & they represented the emergence of a new trend of aggressive politics.

 Tilak & Besant realised that the sanction of a Moderate-dominated Congress as well as full cooperation of the extremists was essential for the movement to succeed.

 Having failed at the 1914 session of the Congress to reach a Moderate-Extremist rapprochement, Tilak & Besant decided to revive political activity on their own while maintaining their pressure on the Congress to re-admit the Extremists.  Pherozeshah Mehta & his Bombay Moderate group succeeded, by winning over Gokhale & the Bengal Moderates, in keeping out the Extremists.  By early 1915, Annie Besant had launched a campaign to demand self government for India after the war on the lines of white colonies. She campaigned through her newspapers, New India & Commonweal, & through public meetings & conferences. At the annual session of the Congress in 1915 the efforts of Tilak & Besant met with some success.

 It was decided that the extremists be admitted to the Congress(Mehta was dead).  Not willing to wait for too long, Besant laid the condition that if the Congress did not implement its commitments, she would be free to set up her own League— which she finally had to, as there was no response from the Congress.  It had 200 branches, was loosely organized as compared to Tilak's League & had George Arundale as the organising secretary. Besides Arundale, the main work was done by B.W. Wadia & C.P. Ramaswamy Aiyar. Home Rule League, 1916  Tilak & Annie Besant set up their separate leagues to avoid any friction.

 Tilak's League was set up in April 1916 & was restricted to Maharashtra (excluding Bombay city), , Central Provinces & Berar. The demands included swarajya, formation of linguistic states & education in the vernacular Languages.

 Annie Besant set up The National Home Rule League in September, 1916 in Madras & covered the rest of India (including Bombay city).

 Jamnadas Dwarkadas, Shankerlal Banker & Indulal Yagnik set up a Bombay paper Young India & launched an All India Propaganda Fund to publish pamphlets in regional languages & in English.

 The Home Rule agitation was later joined by Motilal Nehru, Jawaharlal Nehru, Bhulabhai Desai, , Madan Mohan Malaviya, Mohammad Ali Jinnah, Tej Bahadur Sapru & Lala Lajpat Rai.

 Many of the moderate Congressmen who were disillusioned with Congress inactivity, & some members of Servants of India Society also joined the agitation.

 However, Anglo-Indians, most of the Muslims & non Brahmins from South didn’t join as they felt Home Rule would mean rule of Hindu majority, mainly high caste.

 In 1920, Gandhiji was elected the president of the All India Home Rule League. Government Attitude  The Government came down with severe repression, especially in Madras where the students were prohibited from attending political meetings.

 A case was instituted against Tilak which was withdrawn by the High Court.

 Tilak was defended by a team of lawyers led by Mohammed Ali Jinnah.

 Tilak was barred from entering the Punjab and Delhi.

 In 1917 Besant & her associates, B.P. Wadia & George Arundale, were arrested.

 Sir S. Subramaniya Aiyar renounced his knighthood while Tilak advocated a programme of passive resistance. Lucknow session 1916  Presided over by moderate Ambika Charan Majumdar

 Moderates & Extremists rejoined

 Various factors facilitated this reunion:

 Old controversies had become meaningless now.  Both Moderates & the Extremists realized that the split had led to political inactivity.

 To allay Moderate suspicions, Tilak declared that he supported a reform of administration & not overthrow of Government. He also denounced acts of violence.

 Annie Besant & Tilak made effort for reunion into INC

 Gokhale & Pheroj shah Mehta both moderate died who were opposed to the reunion Lucknow Pact, 1916  Congress and Muslim League sessions were held at the same venue

 Congress and Muslim League agreed to work for Hindu Muslim Unity  Madan Mohan Malaviya opposed this

 This relationship helped in various national movements like Non Cooperation, Khilafat

 This happened at a time when the League, now dominated by the younger militant nationalists, was coming closer to Congress objectives & turning increasingly anti- imperialist.

Shift in the League's position (i) Britain's refusal to help Turkey in its wars in the Balkans (1912-13) & with Italy (during 1911) had infuriated the Muslims.

(ii) Announcement of cancelation partition of Bengal in 1911 had annoyed those sections of Muslims who had supported the partition. (iii) The refusal of the British Government in India to set up a university at Aligarh with powers to affiliate colleges all over India also alienated some Muslims. (iv) The younger League members were turning to bolder nationalist politics & were trying to outgrow the limited political outlook of the Aligarh school. Calcutta session of the League (1912) had committed League to "working with other groups for a system of Self government suited to India, provided it didn’t come in conflict with its basic objective of protection of interests of the Indian Muslims". Thus, the goal of self-government similar to the Congress brought both sides closer.

(v) Younger Muslims were infuriated by the government repression during the War.  Maulana Azad's Al Hilal & Mohammad Ali's Comrade faced suppression while the Ali brothers, Maulana Azad & Hasrat Mohani faced internment.

 This generated anti-imperialist sentiments among the "Young Party".

 While the League agreed to present joint constitutional demands with the Congress to the Government, the Congress accepted League's position on separate electorates.

 The joint demands were—

 Government should declare that it would confer self government on Indians at an early date.  The legislative councils should be further expanded with an elected majority & more powers be given to them.  Half the members of the viceroy's executive council should be Indians. HISTORY April 2, 2018 Introduction  Modern education began in India under the British rule. Before the British, India had its own educational system like the Gurukulas & the Madrassas.

 The East India Company, during their first 60 years of rule didn’t care much for the education of those they ruled in India. (Even in England, universal education came about at a much later stage.)

 There were 3 agents of modern education in India. They were:

 The British Government (East India Company)

 Christian missionaries

 Indian intellectuals & reformers First Phase (1758 – 1812)

 The company wanted some educated Indians who could assist them in the administration of the land.  Also, they wanted to understand the local customs & laws well.  The British East India Company showed very little interest in the education of its subjects during this period, the minor exceptions being :

1. Calcutta Madarsa set up by Warren Hastings in 1781 for the study & teaching of Muslim law & subjects. 2. Sanskrit college at Varanasi by Jonathan Duncan in 1792 for the study of Hindu Law & Philosophy.

3. Both were designed to provide a regular supply of qualified Indians to help the administration of law in the courts of Company.

4. of Bengal was founded by William Jones in Calcutta in 1784. Fort William College

 Set up in : 1800

 Calcutta

 Set up by : Lord Wellesley

 Purpose : For the training of the civil servants of the company in languages & customs of Indians. Missionaries  The missionaries supported the spread of Western education in India primarily for their proselytising activities.

 The Baptist missionary William Carey had come to India in 1793 & by 1800 there was a Baptist Mission in Serampore & also a number of primary schools there & in nearby areas.

 Charles Grant is considered as the father of modern education in India  He is known so because of his efforts that the Charter Act of 1813 came into existence. Second Phase (1813 – 1853)  Due to the strong pressure exerted on the Company by the Christian missionaries & many humanitarians, including some Indians, to encourage & promote modern education in India. The Charter Act of 1813 required the Company to spend rupees 1 lakh annually for encouraging learned Indians & promoting the knowledge of modern sciences in India.

 It also gave an impetus to the missionaries who were given official permission to come to India.

The direction of education remained uncertain during the next 22 years on the following issues:

1. The Medium of Instruction: It could not be decided whether the medium of instruction should be English or Indian Languages. 2. Aim of Education of the British Policy: Whether education should be available to all or should be given to only a selected few. 3. Type of Knowledge: Whether to preserve and promote Oriental learning or to introduce Western knowledge, culture & science. 4. Agency of Education: Whether the Government should assume direct responsibility of educating the Indians or the Indigenous system of education of the country to continue. 5. Role of Missionaries: Whether the missionaries should be given a free hand in their educational practices or should the Company itself shoulder the total responsibility. Orientalists and Anglicists Debate  It was a debate between Orientalists & Anglicists due to the issue of language for the medium of instructions in India

 Orientalists : People who wanted to promote education in India through the medium of classical languages such as Sanskrit, Persian & Arabic.  Anglicists : People who wanted to promote Western Education in India which supported English as a medium of instruction

Macaulay minutes Bentinck had appointed Thomas Babington Macaulay as the Chairman of the General Committee of Public Instruction. Macaulay was an ardent anglicist who had absolute contempt for Indian learning of any kind. He was supported by Reverend Alexander Duff, JR Colvin, etc. On the side of the orientalists were James Princep, Henry Thomas Colebrooke, etc. Macaulay minutes refer to his proposal of education for the Indians.

According to him: English education should be imparted in place of traditional Indian learning because oriental culture was ‘defective’ & ‘unholy’. He believed in education of a few upper & middle class students. In course of time, education would trickle down to the masses. This was called filtration theory. He wished to create a class of Indians who were Indian in colour & blood but English in taste & affiliation. In 1835, the Elphinstone College & the Calcutta Medical College were established. Other developments were :

 The Government’s policy of opening a few English schools and colleges instead of a large number of elementary schools led to the neglect of education of masses.  To cover up this defect, the British took recourse to the so – called ‘Downward Filtration Theory’ which meant that education & modern ideas were supposed to filter or radiate downward from the upper classes.  It made good progress in the 3 presidencies of Bengal, Bombay & Madras where the number of schools & colleges increased.  This policy continued till the very end of British rule, although it was officially abandoned in 1854. Third Phase (1854 – 1900)  The Educational Dispatch of 1854 was also called Wood’s Dispatch (after Sir Charles Wood, the then President of Board of Control, who became the 1st Secretary of State for India). He sent a despatch to Lord Dalhousie.

 It was considered as the Magna Carta of English Education in India (formed a landmark in the history of modern education in India).  Recommendations of the Wood’s Despatch:  It rejected the ‘filtration theory’ & laid stress on mass education, female education & improvement of vernaculars, favored secularism in Education.

 Establishment of universities of Calcutta (Jan 1857) Bombay (Jul 1857), Madras (Sep 1857), Punjab (1882) & Allahabad (1887).  Lord Ripon appointed Hunter Commission (under Sir WW Hunter) .

 It recommended that local bodies (district boards & municipalities) should be entrusted with the management of primary schools.

 Also said that government, should maintain only a few schools & colleges; others to be left to private hands. Fourth Phase (1901 – 1920) :  Lord Curzon appointed a Universities Commission under Thomas Raleigh (Law member of the Viceroy’s Executive Council) in 1902 & based on his recommendations, Indian Universities Act of 1904 was passed.

 It enabled the universities to assume teaching functions (hitherto they were mainly examining bodies), periodic inspection of institutions, speedier transaction of business, strict conditions for affiliation, etc.

 Criticized by nationalists for its tightening government control over universities.

 In 1910, a separate department of Education was established at the Centre.

Fourth Phase (1901 – 1920) :  The Saddler Commission was appointed by Lord Chelmsford to review the working of Calcutta University. It included 2 Indians: Sir Ashutosh Mukherji & Dr. Ziauddin Ahmed.

 Main recommendations were: Secondary Education by a Board of Secondary education & duration of degree course will be 3 yrs.

 7 new universities were opened (Total 12 now) at Banaras, Mysore, Patna, Aligarh, Dhaka, Lucknow & Osmania.

 Kashi Vidyapeeth & Jamia Milia Islamia were established.

 University course divided into pass course & Honours. Fifth Phase (1921 – 1947):  Came under Indian control officially, as it became a provincial subject administered by provincial legislature. Thus, expansions started everywhere.

 Increase in number of universities (20 in 1947); improvement in the quality of higher education (on recommendations of Saddler Commission); establishment of an inter – University Board (1924) & beginning of inter collegiate and inter – university activities.

Hartog Committee 1929  Recommended the policy of consolidation and improvement of Primary education.

 Recommended a selective system of admission to universities and diversified courses leading to industrial and commercial careers.

 Recommended improvements in Universities.

 The Wardha Scheme of Education is also known as Nai Talim/Basic Education/Buniyadi Talim (Shiksha)/Basic Shiksha.

 The scheme was the outcome of sound thinking of Gandhiji who initiated & strengthened several constructive programmes for the economic, educational & social development of the people.

 It was based on Gandhian ideas published in a series of articles in the weekly magazine Harijan.

 He considered education as an effective instrument of national reconstruction. Wardha Education Conference  For the purpose of discussing different aspects of the proposed new scheme of education, an All India Education Conference was held in Wardha on 22nd and 23rd October, 1937.  The eminent educationists, congress leaders and workers along with the Education Ministers of the 7 states had attended the conference.  Gandhiji himself presided over it.  After serious discussions the following 4 resolutions were passed.

1) That in the opinion of this Conference, free & compulsory education be provided for 7 years on a nation-wide scale;

2) That the medium of instruction be the mother-tongue; 3) That the conference endorses the proposal, made by Mahatma Gandhi, that the process of education throughout this period should centre around some productive form of manual work, & that all other abilities to be developed or training should be given, as far as possible, be integrally related to the central handicraft chosen with due regard to the environment of the child. 4) That the conference expects that the system of education will be gradually able to cover the remuneration of the teachers. Appointment of a Committee The conference appointed a committee of distinguished educationists under the chairmanship of Dr. Zakir Hussain, the Committee consisted of 9 members.  Prof. K. G. Saiidain  Arya Nayakam,  Vinova Bhave,  Kaka Kalelkar,  J. C. Kumarappa,  Kishori Lal,  Prof. K. T. Shah etc.

The report of the committee published in March 1938, has come to be known as the Wardha Scheme of Education.

It was approved by Gandhi & was placed before Congress at its Haripura Session

Congress accepted the scheme. Sargeant Plan of Education 1944:

It envisaged:

 Establishment of elementary schools & high school.

 Universal and compulsory education for all children between the ages of 6 – 11.

 High schools of 2 types :

 Academic.  Technical & Vocational.

 Intermediate courses were to be abolished. HISTORY March 23, 2018 Economic History Of Modern India

 It helps in understanding the present day economic problems in India

 Three stages of Capitalism in India

 Drain of wealth

 Deindustrialisation

 Commercialisation of Agriculture Capitalism-System in which the means of production is in the hands of an individual

Three stages of Capitalism in India

Mercantile phase of Capitalism  Started with the arrival of Europeans and existed till the end of 18th century

 India used to receive gold and silver from the Europeans in exchange for the goods which earned India the title of Golden Bird.

 India occupied 24% of the world trade while England’s role was only 2% in 1790

 In 1947, the percentage were reversed

 Before the advent of the foreigners the trade & commerce was in Indian favour.

 The economy was well balanced between agriculture and industry. Industrial phase of Capitalism  It actually started ever since the Industrial Revolution in Europe in 1750’s to 1850’s

 Charter act hastened the process of the second phase

 The finished goods from England were brought to India

 Raw material from India were exported to England

 Economy primarily became agricultural

 Led to deindustrialization

 Policies adopted favoured only the British industries

 The export of raw materials from India to England consumed both time and money Financial phase of Capitalism  Post 1850  Some industries were setup in India due to the presence of both raw material & markets  The cost of journey and the time taken in the 2nd phase were considerably reduced  Example would be Railways, establishment of cotton & jute industries  But the problem was the kind of revolution which occurred in England never took off in India & hence it is called partial industrialization  A large number of cottage industries were ruined by the colonial system  These industries were not replaced because the capitalists feared that if the industry in India developed, the parallel industry in Britain would be forced to shut down  As a result, only few select capitalist were allowed to establish the industries

 Indian capitalists were also not encouraged by the government  These are some of the reason why the third phase was not as successful as the 1st & 2nd phase of capitalism Drain of Wealth  Questions expected from this area: People who wrote about the drain of wealth

 1st man to write about the drain of wealth was Dadabhai Naoroji  He wrote about it in a paper called ‘The English debt to India’  He also wrote a book ‘Poverty and un-British rule in India’

 Other people who wrote were

 Govind Ranade  R.C. Dutt made drain of wealth the subject of his The Economic  R.P. Dutt  Dinshah Wacha  S.N. Banerjee  Prithwishchandra Ray  G Subhramaniya Iyer supported the cause of India's freedom & used his newspaper The Hindu to protest British Imperialism. 2015 Who of the following was/were economic critic/critics of colonialism in India ?

1) Dadabhai Naoroji

2) G. Subramania Iyer

3) R.C. Dutt

Select the correct answer using the code given below: (a) 1 only (b) 1 and 2 only (c) 2 and 3 only (d) 1, 2 and 3 Answer – (D) Channels through which drain of wealth took place  Bribes taken by from the Nawabs of Bengal

 Interest paid to English companies who invested in India

 The English government in India borrowed money from English capitalists and they paid the interest from India’s exchequer

Home charges  The salaries of Secretary of State  Maintenance of the army and funds for waging wars even outside India  Other expenses related to administration  Profits earned by British capitalists through illegal channels

 They used to pay less than the market value while purchasing Indian goods  Support the India office in London.  Salaries & pensions of British personnel engaged in India. 2011 With reference to the period of colonial rule in India, “Home Charges “ formed an important part of drain of wealth from India. Which of the following funds constituted “Home Charges’’ ? 1. Funds used to support the India office in London. 2. Funds used to pay salaries and pensions of British personnel engaged in India. 3. Funds used for waging wars outside India by the British. (a) 1 only. (b) 1 and 2 only. (c) 2 and 3 only. (d) 1,2,and 3. reproducing a paragraph from the Page 149 of the Economic History of India by RC Dutt: " The Indian Tribute whether weighted in the scales of justice or viewed in the light of our interest, will be found to be at variance with humanity, with the commonsense and with the received maxims of economical science. It would be true wisdom then to provide for the future payment of such of the Home Charges of the Indian Government as really from the tribute out of Indian Exchequer. These charges would be probably found to be the dividends on East India Stock, interest in Home debt, the salaries of the officers, establishments of the and building connected with the Home Department of Indian Government, furlough and retired pay to members of the Indian Military and Civil Services when at Home, Charges of all descriptions paid in this country connected with the British troops serving in India and portion of the cost of transporting the British troops to and from India". Correct answer is Option : D Impact of Drain of Wealth  Dadabhai Naoroji wrote that it was the main reason behind the poverty of the nation

 The drain of wealth became a big issue in the Congress session that followed 1896.

 The British claimed that there was no drain of wealth but they were providing services like roads, railways, health, education etc. Deindustrialization Factors that led to deindustrialization

 Industrial revolution in England

 Requirement of raw materials from India

 Markets in India

 The English goods were much cheaper than the Indian goods & hence penetrated easily into the Indian markets

 The English traders started buying most of the raw material at a relative higher price

 Hence the Indian traders were facing shortage of raw material & since they had to pay a higher price of raw material, their cost of production increased Implementation of discriminatory laws by the government

 Import duty on Indian goods in England was 200% to 400% while for the English to sell their goods in India, it was just 2% to 10%

 Extinction of ruling class, & Indian ruling families by the British

 These Indian families used to buy a lot of Indian products.But now that they had been destroyed, there were no consumers for the Indian products

 Now British were the ruling class, they had different preference & requirements which led to the downfall of Indian industries Lack of financial and technical support  There were no banks or co-operative societies in India

 On the other hand in London alone, there were more than 600 banks at the turn of 17th & 18th century

 This shows the financial aid which the companies in England were getting while the Indian traders were not given these facilities by both Indian rulers & English rulers

 No technical progress made by Indians

 There was no sense of entrepreneurship

 Indian traders did not search for new markets after losing a big market in the west Discrimination in freight rates

 Indian goods were charged higher than the English goods for transportation through railways

 It was prevalent everywhere, not only in railways Impact of deindustrialization  Indian economy, which was a balanced economy earlier, became a purely agricultural based economy

 Large scale unemployment

 The small cottage industries which were destroyed were not replaced which led to severe unemployment

 Industries were established only in few places like Bombay, Ahmedabad etc. due to pressure from certain Indian traders

 Most of the Indian traders went back to agriculture, leading to fragmentation of land holding size.

 This resulted in stagnation in agricultural production & disguised unemployment Loss of craft industries

 India was very famous for crafts like brass work, Malmal of Dhaka etc, but on establishment of industrial products from England, these craft industries lost their markets which ultimately resulted in loss of culture and history. Commercialisation of agriculture

Agriculture I………………………………………………………………………I Direct Consumption Selling in market

Commercialisation  Food Crops E.g.-Rice, Wheat Cash crops E.g.-Sugarcane

 Commercialisation existed in ancient, medieval and modern times

 The difference between the commercialization in 1890’s and in ancient times was its magnitude

Factors leading to commercialization  Industrial revolution  They required raw material like cotton, indigo, sugarcane which were agricultural products

 Later, tea was also in great demand as it acted as a stimulant for workers who worked in cold conditions

 Improvement in communications through rail and roads  Emergence of a unified national market

 Linkage of markets : If a particular market suffers due to some reason, the supply from the other markets are used to compensate for the loss of supply from this market. Hence this increased the demand for cash crops

 Boost to international trade given by entry of British finance, capital, etc Process of commercialization  Gave advance payment to the peasants: 1st time advance payment was made

 The British traders paid part-payments, around 10% to 20% in advance for growing cash crops  This was a big allure as advance money was in great demand since there was no credit market  The British took land on lease  They signed some contracts with the peasants and the local authorities  They needed legal security for crops like tea which has a life of 40-45 years  For growing these cash crops, intensive care and technical know-how was required  They hired landless labour to work on the leased lands

 The wages paid to them was very low

 The situation in the tea gardens were very inhuman Impact of commercialization Positive

 Indian markets linked with international markets

 More profits due to export of cash crops by rich peasants

 Improved agricultural infrastructure

Negative  Shortage of food crops  Exploitation of peasants  Rural indebtedness  Got loans with heavy interests

 Negative impact due to linking with international markets: If the harvest is good in some other markets, the Indian peasants were ignored  For instance, the cotton of the 1860s pushed up prices but this mostly benefited the intermediaries, & when the slump in prices came in 1866, it hit the cultivators the most, bringing in its turn heavy indebtedness, famine & agrarian riots in the Deccan in the 1870s.

 Impact on land

 For the Indian peasant, commercialization was a forced process. Development Of Modern Industry  Only in the 2nd half of the 19th century modern machine-based industries came up in India.  The first cotton textile mill was set up in 1853 in Bombay by Cowasjee Nanabhoy  The first jute mill came up in 1855 in Rishra.

 But most of modern industries were foreign-owned.

 There was a rush of foreign capital in India at this time due to prospects of high profits, availability of cheap labor, cheap & readily available raw material, ready market in India & the neighbours, diminishing avenues for investments at home, willingness of the administration to provide all help, and ready markets abroad for some Indian exports such as tea, jute & manganese.  Indian-owned industries came up in cotton textiles & jute in the 19th century & in sugar, cement etc in the 20th century.

 Indian-owned industries suffered from many handicaps: credit problems, no tariff protection by Government, unequal competition from foreign companies, & stiff opposition from British capitalist interests who were backed by financial & technical infrastructure at home. Continued

 The colonial factor also caused certain structural & institutional changes.

 The industrial development was lopsided —core and heavy industries & power generation were neglected & some regions were favored more than the others causing regional disparities.

 These regional disparities hampered the process of nation building.

 Absence of technical education, the industry lacked sufficient technical manpower.

 Socially, the rise of an industrial capitalist class & the working class was an important feature of this phase. Famines  “An Acute shortage of food in a given area due to natural or manmade reasons is called famine” Causes  Poverty causes famine or famine cause poverty?  British scholars:- Famines cause Poverty  Indian Scholars:-Poverty cause Famine  Colonial economic policy, not using rail networks etc Between 1850 and 1900, about 2.8 crore people died in famines.  Eg:- 1876-1877 – period of Lytton. Vast past of central India.  Great famine in Calcutta of 1943 was directly linked with 2nd world war.  Famine commission headed by Richard Strachey . Famine code was drafted in 1883  It’s recommendations  Food would be supplied to people would be given in famine affected area  No tax  Fodder for animals Development of Means of Transport and Communication  They were confined to bullock-cart, camel and packhorse.  They introduced steamships on the rivers and set about improving the roads, Work on the Grand Trunk Road began in 1839 and completed by the 1850's.  The earliest suggestion to build a railway in India was made in Madras in 1831. But the wagons of this railway were to be drawn by horses.

 Construction of steam-driven railways in India was 1st proposed in 1834.  It was given strong political support by England's railway promoters, financiers, mercantile houses trading with India, and textile manufacturers.

 It was decided that Indian railways were to be constructed and operated by private companies who were guaranteed a minimum of 5% return on their capital by GoI.  1st railway line running from Bombay to Thana was opened to traffic in 1853.  Dalhousie proposed a network of 4 main trunk lines which would link the interior of country with big ports and inter-connect the different parts of the country.  By the end of 1869 more than 4,000 miles of railways had been built by the guaranteed companies; but this system proved very costly & slow, and so Government of India decided to build new railways as state enterprises

 But the speed of railway extension still did not satisfy officials in India & businessmen in Britain.

 After 1880, railways were built through private enterprise as well as state agency.

 By 1905, nearly 28,000 miles of railways had been built.  3 important aspects of development of Indian railway's should be kept in mind .  Nearly entire amount of over 350 crores of rupees invested was provided by British investors, Indian capital contributing only a negligible share of it.  They were for the first 50 years financially losing which were not able to pay interest on the capital invested in them  In their planning, construction & management, economic & political development of India & her people was not the chief concern

 On the contrary, the primary consideration was to serve the economic, political, & military interests of British imperialism.  The railway lines were laid primarily with a view to link India's raw material producing areas in the interior with the ports of export.  Needs of Indian industries regarding their markets & their sources of raw materials were neglected.  Moreover, the railway rates were fixed in a manner so as to favor imports & exports & to discriminate against internal movement of goods.  Many lines in Burma & NW India were built at high cost to serve British interests. Continued  The 1st telegraph line from Calcutta to Agra was opened in 1853.

 Dalhousie introduced postage stamps. Previously cash payment had to be made when a letter was posted.  The Post & Telegraph Department was also established in the same year.

 He also cut down postal rates & charged a uniform rate of 1/2 an anna for a letter all over the land.

 Before his reforms, the postage on a letter depended on the distance it was to travel

 In some cases the postage on a letter was the equivalent of as much as 4 days wages of a skilled Indian worker!  Trams HISTORY March 23, 2018 CIVIL SERVICES  Cornwallis was the 1st to bring into existence and organise the civil services. He tried to check corruption through

 raising salary,

 strict enforcement of rules against private trade,

 debarring civil servants from taking presents, bribes etc,

 enforcing promotions through seniority.

Continued  The Indians, however, were barred from high posts from the 'very beginning. Cornwallis thought, "Every native of Hindustan is corrupt.“

 The Charter Act of 1793 had reserved all posts worth 500 pounds per annum for the covenanted servants of the Company. The reasons for exclusion of Indians were—

 the belief that only the English could establish administrative services serving British interests.

 the belief that the Indians were incapable, untrustworthy & insensitive to the British interests.

 the fact there was high competition among the Europeans themselves for lucrative posts, so why offer them to the Indians. Continued  Although the Charter Act of 1833 theoretically threw open the services to the Indians, the relevant provisions were never really implemented.

 It attempted to introduce a system of open competitions for the selection of civil servants. However this provision was negated after opposition from the Court of Directors who continued to hold the privilege of appointing Company officials.

 Fort William College

 1806: Wellesley's college was disapproved by the Court of Directors & instead the East India College was set up at Haileybury to impart 2 years' training to the recruits. Charter Act of 1853  Macaulay Committee of 1854 gave India her first civil services.

 This act removed the right of patronage to appointments in civil service held by the Court of Directors.

 Appointment was to be done only by open competition based on merit and was open to all.

 The report recommended that only the ‘fittest’ be selected to the ICS.

 After 1857, when the Indians claimed a share in higher services, the Proclamation of 1858 declared the British intention of including the Indians, freely & impartially, in offices under the civil service.

 It gave birth to the Indian civil services & was open to all including Indians. Indian Civil Service Act, 1861  This Act reserved certain offices for covenanted civil servants but the examination was held in England in English language, based on classical learning of Greek & Latin.

 The maximum permissible age was gradually reduced from 23 (in 1859) to 22 (in 1860) to 21 (in 1866) & to 19 (1878).

 In 1863, S N Tagore became the first Indian to qualify for the Indian Civil Service.

 In 1878-79, Lytton introduced the Statutory Civil Service consisting of one-sixth of covenanted posts to be filled by Indians of high families through nominations by local governments subject to approval by the secretary & the viceroy. But, the system failed & was abolished. Aitchison Committee on Public Services (1886)  Set up by Dufferin

 Recommendations of the committee —

 dropping of the terms 'covenanted' & 'uncovenanted';

 classification of the civil service into Imperial Indian Civil Service (examination in England), Provincial Civil Service (examination in India) & Subordinate Civil Service (examination in India); &,

 raising the age limit to 23. Continued

 Montford Reforms (1919) These reforms— stated a realistic policy—"If a responsible government is to be established in India, the more Indians we can employ in public service, the better.

 recommended holding of simultaneous examination in India & England.

 recommended that one-third of recruitments be made in India itself—to be raised annually by 1.5 %. Lee Commission (1924) The commission recommended that—

 the secretary of state should continue to recruit the ICS, the Irrigation branch of the Service of Engineers, Indian Forest Service, etc.;

 the recruitments for the transferred fields like education & civil medical service be made by provincial governments;

 direct recruitment to ICS on basis of 50:50 parity between the Europeans & the Indians be reached in 15 years;

 a Public Service Commission be immediately established (as laid down in the Government of India Act, 1919). Lee Commission (1924)  Government of India Act, 1935, The Act recommended the establishment of a Federal Public Service Commission & Provincial Public Service Commission under their spheres.

 But the positions of control and authority remained in British hands & the process of Indianisation of the civil service did not put effective political power in Indian hands since the Indian bureaucrats acted as the agents of colonial rule. HISTORY April 2, 2018 Indian Factory Act, 1881 

 Act, dealt primarily with the problem of child labor (between 7 & 12 years of age).

 Its significant provisions were:

 employment of children under 7 years of age prohibited,

 working hours restricted to 9 hours per day for children,

 children to get 4 holidays in a month,

 hazardous machinery to be properly fenced off. 2017 Consider the following statements :

1. The Factories Act, 1881 was passed with a view to fix the wages of industrial workers and to allow the workers to form trade unions.

2. N.M. Lokhande was a pioneer in organizing the labour movement in British India.

Which of the above statements is/are correct ?

(a) 1 only (b) 2 only (c) Both 1 and 2 (d) Neither 1 nor 2

Answer b The Indian Factory Act, 1891  Following the previous act , a Factory Commission was appointed in 1885.  Another Factories Act in 1891  Increased the minimum age (from 7 to 9 years) & the maximum (from 12 to 14 years) for children  reduced maximum working hours for children to 7 hours a day,

 fixed maximum working hours for women at 11 hours per day with an one &-a-half hour interval (working hours for men were left unregulated),

 Provided weekly holiday for all.  Act made the requirements for fencing machinery more stringent.  Prohibition on employers to employ women within 4 weeks after confinement; Trade Disputes Act of 1929

 The principal object of the Act was to provide a conciliation machinery to bring about peaceful settlement of industrial disputes.

 The Act authorized the central & state governments to establish a board of conciliation or the court of inquiry to investigate into & settle industrial disputes. These tribunals were set up on ad hoc basis.

 The Act prohibited strikes & lock outs in public utility services without prior notice. Such strikes were illegal & punishable under the Act.

2017 The Trade Disputes Act of 1929 provided for :

(a) The participation of workers in the management of industries.

(b) Arbitrary powers to the management to quell industrial disputes.

(c) An intervention by the British Court in the event of a trade dispute.

(d) A system of tribunals and a ban on strikes.

Answer d Development Of Constitution  After the (1764), the East India Company got the Diwani (right to collect revenue) of Bengal, Bihar and Orissa. An annual subsidy was to be paid to the Shah Alam II, and an annual pension to the Nawab of Awadh, Shuja-ud-Daula.  The Company appointed 2 Indians as the deputy .

 1765-72 The dual system of government where the Company had the authority but no responsibility and its Indian representative had all the responsibility but no authority continued for 7 years. This period was characterised by—  rampant corruption among servants of the Company who made full use of private trading to enrich themselves;  excessive revenue collection and oppression of peasantry;  the Company's bankruptcy, while the servants were flourishing.

Regulating Act of 1773  Act of the Parliament intended to overhaul the management of the East India Company's rule in India.  The Act did not prove to be a long-term solution to concerns over the Company's affairs; Pitt's India Act was therefore subsequently enacted as a more radical reform. Regulating Act of 1773

 This was the 1st step to the eventual government control of India.  The Act set up a system whereby it supervised (regulated) the work of the EIC.  The Company had taken over large areas of India for trading purposes & had an army to protect its interests.  Act limited Company dividends to 6% until it repaid a GB£1.5M loan (passed by an accompanying act & restricted the Court of Directors to 4 year terms.

 It prohibited the servants of company from engaging in any private trade or accepting presents or bribes from the natives. Continued  The Act elevated Governor of Bengal, Warren Hastings to Governor-General of Bengal & subsumed the presidencies of Madras & Bombay under Bengal's control—again, a vague provision which created many problems. The whole scheme was based on checks & balances.  It laid the foundations for a centralized administration (unitary type of Government) in India.  Governor General of Bengal had an executive council of 4 to assist him. Decisions would be taken by majority & Governor General could only vote in case of tie. Hastings & 4 others were named in the Act, later ones were to be appointed by the Company.  A supreme court was established at Fort William at Calcutta. Amendments (1781)  The jurisdiction of the Supreme Court was defined—within Calcutta, it was to administer the personal law of the defendant.

 The servants of the Government were immune if they did anything while discharging their duties.

 Social & religious usages of the subjects were to be honoured. Pitt's India Act, 1784/ The East India Company Act 1784  The Act set up a system whereby it supervised (regulated) the work of the Company but did not take power for itself.  Reduced the strength of the council from four to three.  Company's territories in India were called "the British possessions in India".  Governor's council(s) established in Madras & Bombay.  Act provided for the appointment of a Board of Control, & provided for a joint government of British India by the Company & the Crown with the government holding the ultimate authority. Continued  All dispatches were to be approved by the board.  Thus a dual system of control was set up.

 The Board was supported by a Chief Secretary.

 Governors of Bombay & Madras were also deprived of their independence.

 Governor general was given greater powers in matters of war, revenue & diplomacy.  A general prohibition was placed on aggressive wars and treaties (breached often).

 The constitution set up by this act did not undergo any major changes until 1858. The Act of 1786  Cornwallis was allowed to override, the council's decision in if he owned the responsibility for the decision.

 Later, this provision was extended to all the governor general. East India Company Act 1793/ Charter Act of 1793  Act of the Parliament of Great Britain which renewed the charter issued to the EIC, & continued the Company's rule in India.

 Most importantly, the Company's trade monopoly was continued for a further 20 years.

 Salaries for the staff & paid members of the Board of Control were also now charged to the Company.  The Home Government members were to be paid out of Indian revenues. Continued  The Governor-General was granted extensive powers over the subordinate presidencies.

 The Governor-General's power of over-ruling his council was affirmed.

 Senior officials were forbidden from leaving India without permission.

 Royal approval was mandated for the appointment of the Governor-General, the governors, & the Commander-in-Chief.

 EIC was empowered to grant licences to both individuals & Company employees to trade in India (known as the "privilege" or "country" trade), which paved the way for shipments of opium to China. Charter Act, 1813/ East India Company Act 1813  In England, the business interests were pressing for an end to the Company's monopoly over trade in India because of a spirit of laissez faire & the continental system by Napoleon by which the European ports were closed to Britain.  The 1813 Act sought to redress these grievances  Act of the Parliament which renewed the charter issued to the Company & continued the Company's rule in India.  Company's monopoly over Indian trade terminated except for the tea trade & the trade with China; Indian trade thrown open to all the British subjects.  The Act expressly asserted the Crown's sovereignty over British India.  The power of the provincial governments & courts in India over European British subjects was also strengthened by the Act.  The Company's shareholders were given a 10.5 % dividend on the revenue of India.  The Company was to retain the possession of territories & the revenue for 20 years more, without prejudice to the sovereignty of the Crown. (Thus, the constitutional position of the British territories in India was defined explicitly for the first time.)  Powers of the Board of Control were further enlarged. Charter Act, 1833 /The Act 1833/ The Government of India Act 1833  Governor-general of Bengal designated as governor-general of India. Under this provision became the first Governor-General of India.  It deprived the Governors of Bombay & Madras of their legislative powers. For the first time, the Governor-General's Government was known as the 'Government of India' & his council as the 'India Council'. The Governor-General & his executive council were given exclusive legislative powers for the whole of British India.  1st faint beginnings of a Central Legislature for India.  It ended the activities of the EIC as a commercial body & it became a purely administrative body. In particular, the Company lost its monopoly on trade with China & other parts of the Far East. Charter Act, 1853  Charter Act 1853 was passed in the British Parliament to renew the EIC’s charter.  last of the Charter Acts.  Governor-General’s office  The Law member (4th member) became a full member with the right to vote.  The Legislative Council which had 6 members now had 12 members.  The 12 members were: 1 Governor-General, 1 Commander-in-Chief, 4 members of the Governor-General’s Council, 1 Chief Justice of the Supreme Court, 1 regular judge of the Supreme Court, & 4 representative members drawn from among the company’s servants with at least 10 years tenure, appointed by the local governments of Bengal, Bombay, Madras & North Western Provinces.

 The Governor-General could nominate a vice president to the council.  The Governor-General’s assent was required for all legislative proposals. Continued  The Court of Directors could create a new presidency. This was because of the difficulties that were faced in administering the large Indian territories of Britain.  It could also appoint a Lieutenant Governor for the provinces. In 1859, a Lt. Governor was appointed for Punjab.

 This Act also led to the creation of Assam, Burma & the Central Provinces.  The Act provided for the appointment of a separate governor for the . It maintained that the governor of Bengal should be different from the Governor-General who was to head administration of the whole of India.  Number of Board of Directors was reduced from 24 to18 out of which 6 were to be nominated by the British Crown which reduced Company’s influence further.  For the 1st time, the legislative & executive functions of the Governor General’s council were separated. Indian Councils Act, 1861  The Indian Councils Act 1861 was an Act of the that transformed the Viceroy of India's executive council into a cabinet run on the portfolio system.  Cabinet had 6 "ordinary members", who each took charge of a separate department in Calcutta's government: home, revenue, military, law, finance & (after 1874) public works.  Military Commander-in-Chief sat in with the council as an extraordinary member.

 The Executive Council was enlarged by addition of fifth member. The Viceroy was allowed, under the provisions of the Act, to overrule the council on affairs if he deemed it necessary, as was the case in 1879, during the tenure of Lord Lytton.  Legislative Councils established at the center and in the presidencies & provinces.  Councils to include non-official members.

 The Viceroy was allowed to issue ordinances lasting 6 months if the Legislative Council is not in session in an emergency. ARRIVAL OF THE EUROPEANS March 23, 2018 First Europeans

Alexander’s Invasion of India in 326 BCE Why couldn’t Alexander establish an empire in India? Before the Age of Exploration

The world according to Ptolemy

 1492: Columbus in America  Why did the Europeans explore? “The two greatest and most important events recorded in the history of mankind." Adam Smith on the discovery of America and the Cape route to India What was the world like in the 15th c.?  India: Decline of Tughluk Sultanate - Regional kingdoms

 China: Ming Dynasty

 Central Asia: Mongols, Timurids

 Arab World: Ottomans

 Europe  England-France: Hundred years wars  Defeat of Moors: Spain and  Italian city-states  Byzantine Empire

 1453: Fall of Constantinople Introduction  is a part of the Indian subcontinent which was under the control of European colonial powers, through trade and conquest.

 The search for the wealth and prosperity of India led to the accidental "discovery" of the Americas by Christopher Columbus in 1492.

became the 1st European to re-establish direct trade links with India since Roman times by being the 1st to arrive by circumnavigating Africa (1497–99).

 Trading rivalries brought other European powers to India.

 The , England, France, and Denmark established trading posts in India in the early 17th century. Portuguese Expeditions

Henry the Navigator Bartolomeu Dias Pedro Alvares Cabral

 Why did Portugal have a lead in exploration?

 Why were the Portuguese successful in Africa and Asia? Portuguese  The closing of the traditional trade routes in western Asia by the Ottoman Empire and rivalry with the Italian states sent Portugal in search of an alternate sea route to India.

 The first successful voyage to India was by Vasco da Gama in 1498, when he arrived in Calicut. Having arrived in Calicut he obtained permission to trade in the city.

 The colonial era in India began in 1502, when the established the first European trading center at , Kerala. In 1510 Portuguese conquered the city of , which had been controlled by Muslims.

 Policy of marrying Portuguese soldiers and sailors with local Indian girls started consequence of which was a great in Goa and other Portuguese territories .

 Another feature of the Portuguese presence in India was their will to evangelize and promote Catholicism. In this, the Jesuits played a fundamental role, and to this day the Jesuit missionary Saint is revered among the Catholics of India. Continued  The Portuguese established a chain of outposts along India's west coast and on the island of Ceylon in the early .  Goa was their prized possession and the seat of Portugal's viceroy. Portugal's northern province included settlements at Daman, Diu, Baçaim, Salsette, and Mumbai.

 The rest of the northern province, with the exception of , was lost to the in the early 18th century.

 In 1661 Portugal was at war with Spain and needed support from England. This led to the marriage of Princess Catherine of Portugal to Charles II of England, who imposed a dowry that included the insular and less inhabited areas of southern Bombay while the Portuguese managed to retain all the mainland territory north of Bandra up to Thana and Bassein.  This was the beginning of the English presence in India. Portuguese Conquests  From the beginning the Portuguese combined the use of force with trade.  In this they were helped by the superiority of their armed ships which enabled them to dominate the seas.  They didn’t shy away from piracy and plunder

Fransisco de Almeida Afonso de Albequerque

 Portuguese were the first to issue cartage.

 Through cartage they got huge income. Decline

 The Portuguese were intolerant and fanatical in religious matters. They indulged in forcible conversion 'offering people the alternative of Christianity or .'

 In spite of their barbaric behavior their possessions in India survived for a century because they enjoyed control over the high seas, their soldiers and administrators maintained strict discipline, and they did not have to face the might of the as was outside Mughal influence.

 They clashed with the Mughal power in Bengal in 1631 and were driven out of their settlements at Hugli.

 Their hold over the Arabian sea had already been weakened by the English and their influence in Gujarat had become negligible by this time. Why did Portuguese power reduce over time?

 Portugal was incapable of maintaining for long its trade monopoly or its dominions in the East

 Its population was less than a million

 It’s Court was autocratic and decadent

 It’s merchants enjoyed much less power and prestige than its landed aristocrats

 It lagged behind in the development of shipping

 It followed a policy of religious intolerance.

 Portugal had become a Spanish dependency in 1580.  The Portuguese and the Spanish had left the English and the Dutch far behind during the 15th century and the first half of the 16th century.

 But, in the latter half of the 16th century, England and Holland, and later France, all growing commercial and naval ,powers, waged a fierce struggle against the Spanish and Portuguese monopoly of world trade.

 In 1588 the English defeated the Spanish fleet called the Armada and shattered Spanish naval supremacy for ever.

 This enabled the English and the Dutch merchants to use the Cape of Good Hope route to India and so to join in the race for empire in the East. Dutch  The was formed in 1602.  The Company established trading posts on different parts along the Indian coast.  For some while, they controlled the Malabar southwest coast (Cochin) and the Coromandel southeastern coast (Golconda , Kakinada, , Negapatnam) and Surat (1616–1795).  The Dutch also established trading stations in and Tamil Nadu as well as at Rajshahi and Murshidabad, Balasore , and Ava, , and Syriam in Myanmar.  They exported indigo, raw silk, cotton textiles, saltpetre, and opium from India.  Like the Portuguese they treated the people of India cruelly.  They conquered Ceylon from the Portuguese in 1658  It was lost at the Congress of Vienna in the aftermath of the Napoleonic Wars, where the Dutch having fallen subject to France, saw their colonies raided by Britain.  The Dutch later became less involved in India, as they had the Dutch East Indies as their prized possession.  In 1795 policy of exchange between them. Danish  The Danish formed an east India company and arrived in India in 1616

 Denmark was a minor colonial power to set foot in India.

 It established trading outposts in Tranquebar (1620), Serampore(1755), Calicut(1752) and the Nicobar Islands (1750s).

 At one time, the main Danish and Swedish East Asia companies together imported more tea to Europe than the British did.

 Their outposts lost economic and strategic importance, and Tranquebar, the last Danish outpost, was sold to the British in 1845.

 They were more concerned with missionary works. French  Following others, the French also established trading bases in India.

 The French company was created ,financed and controlled by the state and it differed from the English company which was a private commercial venture.

 First establishment was in Pondicherry in 1674.

 Subsequent French settlements were Chandernagore , Yanam , Mahe and Karaikal .

 They were constantly in conflict with the Dutch and later on mainly with the British in India. Continued  Between 1744 and 1761, the British and the French repeatedly attacked and conquered each other's forts and towns in southeastern India and in Bengal in the northeast.

 After some initial French successes, the British decisively defeated the French in 1761 in the .

 By the treaty of Paris Pondicherry and some other French settlements were returned to the French.

 The enclaves of Pondicherry, Karaikal, Yanam, Mahe and Chandernagore were integrated with the India in 1954. East India Companies

 How and why were the companies different from each other?

 Why did the Dutch not become a power in India?

 Where were the other European countries? Factories Factories European Settlements HISTORY March 23, 2018 AUGUST OFFER (1940)  Hitler's astounding success & the fall of Belgium, Holland & France put England in a conciliatory mood. To get Indian cooperation in the war effort, viceroy announced the August Offer which proposed:  dominion status as the objective for India.  expansion of viceroy's executive council.  setting up of a constituent assembly after the war. Indians would decide the constitution according to their social, economic & political conceptions, subject to fulfillment of the obligation of the Government regarding defence, minority rights, treaties with states, all India services.  no future constitution to be adopted without the consent of minorities.  The Congress rejected the August Offer. Nehru said, "Dominion status concept is dead as a door nail." Gandhi said that the declaration had widened the gulf between the nationalists & the British rulers.  The Muslim League welcomed the veto assurance given to the League, & reiterated its position that partition was the only solution to the deadlock. Evaluation  For the 1st time, the inherent right of Indians to frame their constitution was recognised & the Congress demand for a constituent assembly was, conceded.

 Dominion status was explicitly offered.

 In July 1941, the viceroy's executive council was enlarged to give the Indians a majority of 8 out of 12 for the 1st time, but the whites remained in charge of defence, finance & home.

 Also, a National Defence Council was set up with purely advisory functions. Individual  Government had taken the adamant position that no constitutional advance could be made till the Congress came to an agreement with the Muslim leaders. It issued ordinance after ordinance taking away the freedom of speech & that of the press & the right to organise associations.

 Towards the end of 1940, the Congress once again asked Gandhi to take command. Gandhi now began taking steps which would lead to a mass struggle within his broad strategic perspective.

 Congress was in a confused state again after the August Offer. The radicals & leftists wanted to launch a mass Civil Disobedience Movement, but here Gandhi insisted on Individual Satyagraha.  This view was conveyed to Linlithgow by Gandhi when he met him on September 27, 1940. The non-violence was set as the center piece of Individual Satyagraha. This was done by carefully selecting the Satyagrahis.  The aims of launching individual satyagraha were—  to show that nationalist patience was not due to weakness;

 to express people's feeling that they were not interested in the war they made no distinction between Nazism & the double autocracy that ruled India;

 to give another opportunity to the Government to accept Congress' demands peacefully.  The demand of the satyagrahi would be the freedom of speech against the war through an anti-war declaration.  The Individual Satyagraha was not to seek independence but to affirm the right of speech. The other reason of this Satyagraha was that a mass movement may turn violent & he would not like to see the Great Britain embarrassed by such a situation.  If the Government did not arrest the satyagrahi, he or she would not only repeat it but move into villages and start a march towards Delhi, thus precipitating a movement which came to be known as the "Delhi Chalo Movement". Continued  was the 1st to offer the satyagraha & Nehru, the 2nd .  3rd was Brahma Datt, one of the inmates of the Gandhi’s Ashram.  They all were sent to jails for violating the Defense of India Act.  This was followed by a lot of other people. But since it was not a mass movement, it attracted little enthusiasm & in December 1940, Gandhi suspended the movement.  The campaign started again in January 1941, this time, thousands of people joined & around 20 thousand people were arrested.

 The CWC overrode Gandhi's & Nehru's objections and passed a resolution offering to cooperate with the Government in the defence of India, if  full independence was given after the war, &  substance of power was transferred immediately.  It was at this time that Gandhi designated Nehru as his chosen successor. (March 1942)  A mission headed by was sent to India with constitutional proposals to seek Indian support for the war.

 Cripps was a left-wing Laborite, the leader of the House of Commons & a member of the British War Cabinet who had actively supported Indian national movement.

Why Cripps Mission was Sent?  Because of the reverses suffered by Britain in South-East Asia, the Japanese threat to invade India seemed real now & Indian support became crucial

 There was pressure on Britain from the Allies (USA, USSR) to seek Indian cooperation.

 Indian nationalists had agreed to support the Allied cause if substantial power was transferred immediately & complete independence given after the war. Main Proposals 1. An Indian Union with a dominion status would be set up; it would be free to decide its relations with the Commonwealth & free to participate in international bodies.

2. After the end of the war, a constituent assembly would be convened to frame a new constitution. Members of this assembly would be partly elected by the provincial assemblies through proportional representation & partly nominated by the princes.

3. The British Government would accept the new constitution subject to 2 conditions: (i) any province not willing to join the Union could have a separate constitution & form a separate Union, (ii) the new constitution making body & the British Government would negotiate a treaty to effect the transfer of power & to safeguard racial & religious minorities.

4. In the meantime, defence of India would remain in British hands & the governor- general's powers would remain intact. Departures from the Past and Implications  The proposals differed from those offered in the past in many respects—

 The making of the constitution was to be solely in Indian hands now (& not "mainly" in Indian hands—as contained in the August Offer).

 A concrete plan was provided for the constituent assembly.

 Option was available to any province to have a separate constitution—a blueprint for India's partition.

 Free India could withdraw from the Commonwealth.

 Indians were allowed a large share in the administration in the interim period. Why Cripps Mission Failed?  Proposals failed to satisfy Indian nationalists & turned out to be merely a propaganda device for US & Chinese consumption.  The incapacity of Cripps to go beyond the Draft Declaration & the adoption of a rigid "take it or leave it" attitude added to the deadlock.  Cripps had earlier talked of "cabinet" & "national government" but later he said that he had only meant an expansion of the executive council.  It was not clear as to who would implement & interpret the treaty effecting the transfer of power.  Churchill, Amery (secretary of state), Linlithgow & Ward (commander-in-chief) consistently torpedoed Cripps' efforts.  Talks broke down on the question of the viceroy's veto. Congress’s Objections  the offer of dominion status instead of a provision for complete independence.

 representation of the states by nominees & not by elected representatives.

 right to provinces to secede as this went against the principle of national unity.

 absence of any plan for immediate transfer of power & absence of any real share in defence; the governor general's supremacy had been retained, & the demand for governor general being only the constitutional head had not been accepted.

 Nehru & Azad were the official negotiators for the Congress.  Gandhi described the scheme as "a post-dated cheque";  Nehru pointed out that the "existing structure & autocratic powers would remain & a few of us will become the viceroy's liveried camp followers & look after canteens & the like". League’s Objections  Criticised the idea of a single Indian Union.

 Didn’t like the machinery for the creation of a constituent assembly & the procedure to decide on the accession of provinces to the Union.

 thought that the proposals denied the Muslims the right to self-determination & the creation of Pakistan.

Others’ Objections  Other groups also objected to the provinces' right to secede.

 Liberals considered the secession proposals to be against the unity & security of India.

 Hindu Mahasabha criticised the basis of the right to secede.

 Depressed classes thought that partition would leave them at the mercy of the caste Hindus.

 The Sikhs objected that partition would take away Punjab from them.

 The explanation that the proposals were meant not to supersede the August Offer but to clothe general provisions with precision put British intentions in doubt. 2016 The plan of Sir Stafford Cripps envisaged that after the Second World War

(a) India should be granted complete independence

(b) India should be partitioned into two before granting independence

(c) India should be made a republic with the condition that she will join the Commonwealth

(d) India should be given Dominion status

Answer d Reasons for Start of a Struggle Now  Failure of Cripps Mission  There was popular discontent because of rising prices and shortage of rice, salt, etc. and because of factors such as commandeering of boats in Bengal & Orissa. There were fears of Britain following a scorched earth policy in Assam, Bengal & Orissa against possible Japanese advance.  News of reverses' suffered by the British in SE Asia & an imminent British collapse enhanced popular willingness to give expression to discontent. Faith in stability of British rule was so low that people were withdrawing deposits from banks.  The manner in which the British evacuated from SE Asia leaving the subjects to their fate (2 roads were provided— Black Road for Indian refugees & White Road exclusively for European refugees).  The leadership wanted to condition the masses for a possible Japanese invasion. QUIT INDIA MOVEMENT  Failure of the Cripps Mission embittered the people of India.

 While they still fully sympathized with the anti-Fascist forces, they felt that the existing political situation in the country had become intolerable.

 Congress now decided to take active steps to compel the British to accept the Indian demand for independence

 All India Congress Committee met at —Gowalia Tank, Bombay (August 8, 1942)

Quit India Resolution  It passed the famous Quit India Resolution & proposed the starting of a non- violent mass struggle.

The meeting resolved to

 demand an immediate end to British rule in India.

 declare commitment of free India to defend itself against all types of Fascism & imperialism.

 form a provisional Government of India after British withdrawal.

 sanction a civil disobedience movement against British rule.

 Gandhi was named the leader of the struggle. Gandhi's General Instructions to Different Sections These were spelt out at the meeting but not actually issued. They were directed at various sections of society.

 Government servants: Do not resign but declare your allegiance to the Congress.  Soldiers: Do not leave the Army but do not fire on compatriots.  Students: If confident, leave studies.  Peasants: If zamindars are anti-government, pay mutually agreed rent, & if zamindars are pro-government, do not pay rent.  Princes: Support the masses and accept sovereignty of your people.  Princely states' people: Support the ruler only if he is anti-government & declare yourselves to be a part of the Indian nation.

 Gandhi followed up with the now-famous exhortation: "Here is a mantra, a short one, that I give you You may imprint it on your hearts and let every breath of yours give expression to it The mantra is 'Do or Die'. We shall either free India or, die in the attempt; we shall not live to see the perpetuation of our ." Spread of the Movement  Gandhi had carefully built the tempo through individual civil disobedience movements, organisational revamping & a consistent propaganda campaign.

 Government, however, was in no mood to either negotiate with the Congress or wait for the movement to be formally launched.

 Before the Congress could start a movement, the Government struck hard.

 Early in the morning of 9 August, Gandhi & other Congress leaders were arrested & taken to unknown destinations

 Congress was once again declared illegal.

Public on Rampage  The news of these arrests left the country aghast, & a spontaneous movement of protest arose everywhere, giving expression to the pent up anger of the people.  Left leaderless & without any organisation, people reacted in any manner they could.

 The general public attacked symbols of authority, hoisting national flags forcibly on public buildings.

 Satyagrahis offered themselves up to arrest, bridges were blown up.

 Students responded by going on strikes in schools & colleges, participating in processions, writing & distributing illegal news sheets (patrikas) & acting as couriers for underground networks.

 Workers went on strike in Ahmedabad, Bombay, Jamshedpur, Ahmednagar & Poona. Underground Activity  This was undertaken by the Socialists, Forward Bloc members, Gandhi ashramites, revolutionary terrorists & local organisations in Bombay, Poona, Satara, Baroda & other parts of Gujarat, Karnataka, Kerala, Andhra, UP, Bihar & Delhi.

 Usha Mehta started an underground radio in Bombay.

 Jaiprakash Narayan formed ‘Azad Dasta’ in the terrains of Nepal & trained them in guerrilla warfare.

 Underground activity was carried out by Rammanohar Lohia, Aruna Asaf Ali, Biju Patnaik, Chhotubhai Puranik, , Sucheta Kripalani & R.P. Goenka.

 This phase of underground activity was meant to keep up popular morale by continuing to provide a line of command & guidance to distribute arms & ammunition. Parallel Governments  Ballia (in August 1942 for a week)—under Chittu Pandey. He got many Congress leaders released.

 Tamluk (Midnapore, from December 1942 to September 1944)—which undertook cyclone relief work, sanctioned grants to schools, supplied paddy from the rich to the poor, organised Bidyut Bahinis etc.

 Satara (mid-1943 to 1945)—named "Prati Sarkar", was organised under leaders like Y.B. Chavan, Nana Patil, etc.

 Village libraries and Nyayaclan Mandals were organised, prohibition campaigns were carried on & 'Gandhi marriages' were organised. Extent of Mass Participation  Youth, especially the students of schools and colleges, remained in the forefront.

 Women, especially school and college girls, actively participated.

 Workers went on strikes & faced repression.

 Peasants concentrated their offensive on symbols of authority and there was complete absence of anti-zamindar violence. Even some zamindars participated .  Government officials, especially those belonging to lower levels in police & administration, participated resulting in erosion of government loyalty.  Muslims helped by giving shelter to underground Activists. There were no communal clashes during the movement.

 Communists, despite their anti-war line, felt the irresistible pull of the movement.

 Princely states showed a low-key response. Government Repression  Although martial law was not applied, the repression was severe.

 Agitating crowds were lathi-charged, tear-gassed & fired Upon.

 The 'number of those killed is estimated at 10,000.

 The press was muzzled.

 The military took over, many cities; police & secret service reigned supreme.

 Rebellious villages were fined heavily & in many villages, mass flogging was done.

 Government used maximum force to supress this movement which included aircrafts & machine guns.

Estimate  February 1943 Gandhi, started a fast as an answer to an exhortation to the Government to condemn violence.  The popular response to the news of the fast was immediate & overwhelming.  Protests were organised at home & abroad through hartals, demonstrations & strikes.  3 members of the viceroy's executive council resigned.

 The fast achieved the following purposes—

 public morale was raised.  anti-British feeling was heightened.  an opportunity was provided for political activity.

 Government's high-handedness was exposed.  Gandhi got the better of his opponents and refused to oblige by dying.  March 23, 1943 Pakistan Day was observed. QUIT INDIA MOVEMENT  It was an all India Movement.

 The movement was strongest in Bihar followed by U.P. & Maharashtra  Students’ agitation & peasant movements in Bihar made it a strong movement in Bihar.

 Although this movement theoretically was led by Gandhi but in practice it was in the hands of revolutionaries & the local leaders.

 There was a change in stance of Gandhi during this movement.

 He did not advocate violence but he did not condemn it also.  Gandhi didn’t stop the movement due to violent activities as he did in Non- Cooperation Movement.  Hindu Mahasabha leaders did not participate in Quit India as their agenda was Religion specific whereas Congress was a Secular Party.

 Government lifted ban on Communist party in 1942 which it had enforced in 1934.

 Communists did not join Quit India as the movement was against English which was partner of Russia but Communists didn’t help British in suppressing the movement.

Importance of this Movement  The Revolt of 1942 was in fact short lived.

 Its importance lay in the fact that it demonstrated the depth that nationalist feeling had reached in the country and the great capacity for struggle & sacrifice that the people had developed.

 There was an institutional breakdown in this movement.

 This movement is considered to be the turning point in Indian History

 The British hopes of ruling India for 100 years more was crushed due to this movement. 2013 Quit India Movement was launched in response to a) Cabinet Mission Plan b) Cripps Proposals c) Simon Commission Report d) Wavell Plan After the failure of Cripps mission, Gandhi decided to launch his third movement against the British Rule. This was the Quit India campaign, which began in August 1942 Why had it become necessary to launch a (Quit India) movement in these difficult conditions, when the possibility of a brutal repression was a certainty?…..For one, the failure of the Cripps mission in April 1942 made it clear that Britain was unwilling to offer an honorable settlement…. Answer B. 2011 With reference to Indian freedom struggle, Usha Mehta is well-known for ?

(a)Running the secret congress radio in the wake of quit India movement.

(b)Participating in the second round table conference.

(c)Leading a contingent of .

(d)Assisting in the formation of Interim government under Pandit Jawaharlal Nehru.

Secret Radio was run by Usha Mehta . Correct Option is A 2011 Which one of the following observation is not true about the quit India movement of 1942 ? (a) It was a non-violent movement. (b) It was led by mahatma Gandhi.

(c) It was a spontaneous movements.

(d) It did not attract the labor class in general. On August 8, 1942 the Quit India Resolution was passed at the Bombay session of the All India Congress Committee and here Gandhi made a call to participate people in non-violent way. It's worth note that the communists had opposed this movement and it virtually damaged the labour movement. Labor Unions under Communist influence had apparently decided against participation in the movement, there were large-scale strikes in mills at Kanpur, Jamshedpur and Ahmedabad. There was an indifference of the Labor Class, so the statement D is correct. The First statement in this question needs to be looked into. The Quit India Movement was basically promoted as a nonviolent and non cooperative movement but it was not a non-violent. There were various events in which Police station, Railway Stations, Railway Lines and Post-Offices were burnt and destroyed. So correct option is A. C. Rajagopalachari's formula (or C. R. formula or Rajaji formula)  TheMeanwhile, efforts were on to solve the on going constitutional crisis, & some individuals also tried to come up with constitutional proposals. The main points in CR Plan were:  The League was to endorse the Indian demand for independence and to co-operate with the Congress in formation of Provisional Interim Government for a transitional period.  At the end of the War, a commission would be appointed to demarcate the districts having a Muslim population in absolute majority and in those areas plebiscite to be conducted on all inhabitants (including the non-Muslims) on basis of adult suffrage.

 All parties would be allowed to express their stance on the partition & views before the plebiscite.

 In the event of separation, a mutual agreement would be entered into for safeguarding essential matters such as defense, communication commerce & for other essential services.

 The transfer of population, if any would be absolutely on a voluntary basis.

 The terms of the binding will be applicable only in case of full transfer of power by Britain to GoI. Continued  It was a tacit acceptance of the League's demand for Pakistan.  Although the formula was opposed even within the Congress, Gandhi supported it.

 Gandhi used it as his proposal in his talks with Jinnah in 1944.

 However, Jinnah rejected the proposal & the talks failed.

Jinnah's Objections  Jinnah wanted the Congress to accept the two-nation theory.  He wanted only the Muslims of North-West & North-East to vote in the plebiscite & not the entire population.  He also opposed the idea of a common centre.  While the Congress was ready to cooperate with the League for the independence of the Indian Union, the League did not care for independence of the Union. It was only interested in a separate nation.  Hindu leaders led by Vir Savarkar condemned the CR Plan. Desai-liaqat Pact  Efforts continued to end the deadlock.

 Bhulabhai Desai, leader of the Congress Party in the Central Legislative Assembly, met Liaqat Ali Khan, deputy leader of the Muslim League in that Assembly, & both of them came up with the draft proposal for the formation of an interim government at the centre, consisting of—

 an equal number of persons nominated by the Congress & the League in the central legislature.

 20% reserved seats for minorities.

 No settlement could be reached between the Congress & the League on these lines, but the fact that a sort of parity between the Congress & the League was decided upon, which had far-reaching consequences. The Indian National Army  The idea of the Indian National Army (INA) was first conceived in Malaya by Mohan Singh, an Indian officer of the British Indian Army, when he decided not to join the retreating British Army and instead turned to the Japanese for help.  The 1st Phase  Japanese handed over the Indian POWs to Singh who tried to recruit them into INA. After fall of Singapore, Mohan Singh further got 45,000 POWs into his sphere of influence. By the end of 1942, 40,000 men were ready to join the INA.  The outbreak of the Quit India gave a fillip to the INA as well. In September 1942, 1st division of INA was formed with 16,300 men. The Second Phase  The next phase began with the arrival of Subhash Bose in Singapore in July 1943.  Earlier, he had left the Congress after having developed differences with Gandhi & had formed the Forward Bloc in 1940  In March 1941, he escaped from India, where he had been under house arrest, & approached the Russian leaders for help against Britain.

 When in June 1941 the Soviet Union joined the war on behalf of the Allies, Bose went to Germany & from there he reached Japan in February 1943.

 He asked for Japanese help for an armed struggle against the British. He came to Singapore in July 1943 where he was assisted by Rashbehari Bose & others, such as the Indian residents of S- E Asia & Indian POWs from Burma, Malaya & Singapore.

 October 1943:He set up a Provisional Indian Government known as Arzi Hukumat- e- with HQ at Rangoon & Singapore. This Government was recognised by the Axis powers. INA Movement  Indian Independence League was setup up by in Tokyo.  Major Fujiwara helped Captain Singh in setting it up.  The battalions raised were called Gandhi battalion, Nehru battalion, Subhash battalion & Rani Lakshmi battalion.

 July 1944: Bose asked for Gandhi's blessings for "India's last war of independence".  One INA battalion commanded by Shah Nawaz was allowed to accompany the Japanese Army to the Indo-Burma front & participate in the Imphal campaign.  But the discriminatory treatment by the Japanese, which included being denied rations & arms & being made to do menial work for the Japanese units, completely demoralised the INA units.

 The failure of the Imphal campaign & the steady Japanese retreat thereafter quashed any hopes of the INA liberating the nation.  The retreat continued till mid-1945 & ended only with the final surrender to the British in S-E Asia. Trial/INA Trial  Trial was held in 1945-46 in which Captain Shah Nawaaz Khan, Captain Prem Kumar Sehgal, Captain Gurubaksh Singh Dhillon were trialled.  The trial, was held between November & December 1945 against the backdrop of general elections in India with the Attorney General of India, Noshirwan P. Engineer as the chief prosecutor.

 All three of the accused were charged with waging war against the king contrary to section 121 of the Indian Penal Code'.  Defendants came from 3 different religions - one Hindu, one Sikh, & one Muslim - but all 3 elected to be defended by the defense committee set up by the Congress.

 They were sentenced to death, but under pressure from the political parties from India, Army Chief Claude Auchinleck was forced to commute the sentences of the three defendants in the first trial.  The defence of the INA prisoners was taken up by the Congress & Bhulabhaj Desai, K.N. Katju, Nehru & Asaf Ali appeared in court.  2 dozen counsel for the defense were led by Sir Tej Bahadur Sapru.

 The Congress organised an INA Relief & Enquiry Committee, which provided small sums of money & food to the men on their release, & attempted, though with marginal success, to secure employment for these men.

Netaji  3 commissions were set up to probe the death of Subhash Chandra Bose.

 Captain Shah Nawaaz Khan commission,  Justice Khosla Commission  Justice Mukherjee Commission. Wavell Plan (1945)  Although war in Europe came to an end in May 1945, Japanese threat still remained.

 Conservative Government in Britain was keen to reach a solution on the constitutional question in India.

 Lord Archibald Wavell was permitted to start negotiations with Indian leaders & so they were released from jails in June 1945.

 Why the Government was keen on a Solution Now

 The general election in England was scheduled for mid-1945. The Conservatives wanted to be seen as sincere on reaching a solution.

 There was pressure from the Allies to seek further Indian cooperation in the war.  Government wanted to divert Indian energies into channels more profitable for the British. The Plan  Idea was to reconstruct the governor general's executive council pending the preparation of a new constitution.  For this purpose, a conference was convened Lord Wavell, at Shimla in June 1945.  The main proposals of the Wavell Plan were as follows:  Dominion Status to India  With the exception of the governor-general & the commander-in-chief, all members of the executive council were to be Indians.  Caste Hindus & Muslims were to have equal representation. Head of Government – Indian, 5 ministers – Hindus, 5 ministers – Muslims, others – Dalits, Sikhs etc.  The reconstructed council was to function as an interim government within the framework of the 1935 Act (i.e. not responsible to the Central Assembly).  Governor- general was to exercise his veto on the advice of ministers.  Representatives of different parties were, to submit a joint list to the viceroy for nominations to the executive council. If a joint list was not possible, then separate lists were to be submitted.

Muslim League's Stand  League wanted all Muslim members to be League nominees, because it feared that since the aims of other minorities—depressed classes, Sikhs, Christians, etc. were the same as those of the Congress, this arrangement would reduce the League to a 1/3rd minority. (Wavell wanted Khizr Hyatt Khan as Muslim representative from Western Punjab.)  The League claimed some kind of veto in the council with decisions opposed' to Muslims needing a 2/3rd majority for approval.  Congress Stand  The Congress objected to the plan as "an attempt to reduce the Congress to the status of a purely caste Hindu party & insisted on its right to include members of all communities among its nominees".  Wavell's Mistake: Wavell announced a breakdown of talks thus giving the League a virtual veto.  This strengthened League's position, as was evident from the elections in 1945-46  boosted Jinnah's position  exposed the real character of the Conservative Government of Churchill. Elections in the Provinces (1945-1946)  Muslim League used many slogans

 Pakistan was one of the big agendas through which Muslim League campaigned.

 League used very strong slogans in their campaign like “If you do not vote for Muslim League then you will not be buried in Muslim graveyard”.

 It is to be noted that the top leaders of Muslim League were not practicing Muslims like Jinnah whereas orthodox Muslims like Azad was whole heartedly supporting Congress, Gandhi and Nationalist movement.

ELECTION RESULTS Congress' Performance  It got 91% of non-Muslim votes.  It captured 57 out of 102 seats in the Central Assembly.  In the provincial elections, it got a majority in most provinces except in Bengal, Sindh & Punjab. The Congress majority provinces included NWFP & Assam which were being claimed for Pakistan. Muslim League's Performance  It got 86.6% of the Muslim votes.  It captured all the 30 reserved seats in the Central Assembly.  In the provincial elections, it got a majority in Bengal & Sindh.  Unlike in 1937, now the League clearly established itself as the dominant party among Muslims.

 Almost 507 seats were reserved for Muslims from all over India in 11 provinces.  Amongst these 492 seats were of Muslim League.  Punjab: Unionist-Congress-Akali coalition under Khizr Hyatt Khan assumed power. Royal mutiny  (also called the Royal Indian Navy Revolt or Bombay Mutiny) encompasses a total strike & subsequent revolt by Indian sailors of the Royal Indian Navy on board ship & shore establishments at Bombay harbor on 18 February 1946.

 From the initial flashpoint in Bombay, the revolt spread & found support throughout British India, from Karachi to Calcutta & ultimately came to involve 78 ships, 20 shore establishments & 20,000 sailors.

 Only the Communist Party supported the strikers

 Congress & the Muslim League condemned it. Cabinet Mission Plan 1946  Pethick Lawrence, A V Alexander and Stafford Cripps.  They took a unanimous decision that India cannot be partitioned.  Cabinet Mission Plan comprised of 4 major parts:-  Grouping of Provinces in terms of:-  Hindu Majority Provinces (provinces like UP, CP, Bombay etc.)  Muslim Majority Provinces (Western India – Punjab, NWFP )  Muslim Majority Provinces (Eastern India – Assam and Bengal)  Each group would form separate constitution for their province & will collectively make a constitution for India  Interim Government would be formed  Princely states were free to join interim government or could remain with British  A Constituent assembly would be formed  Cabinet Mission plan was rejected by both Congress & League  16th Aug 1946: League declared the infamous & gave controversial statements like ‘ladke lenge Pakistan’. 2015 With reference to the Cabinet Mission which of the following statements is/ are correct?

1) It is recommended a federal government.

2) It enlarged the powers of the Indian courts.

3) It provided for more Indians in the ICS.

Select the correct answer using the code given below. (a) 1 only (b) 2 and 3 (c) 1 and 3 (d) None Answer – (A) Formation of Interim Government  After the initial boycott, League joined interim government in the last week of October 1946.

 5 League members were made ministers in Interim government including Liaquat Ali Khan who was made the Finance Minister.

 Top leaders of League were kept away from the Interim Government as they were active in Pakistan movement.

 7th Feb 1947: Nehru wrote a letter to British PM Clement Richard Attlee to complain about Muslim League.  20th Feb 1947: Attlee declared that India would be freed by June 1948 & also announced that Lord Mountbatten would be the last Governor General of India.

 Lord Mountbatten announced Mountbatten plan on 3rd June. Mountbatten Plan  On 15th August India would be freed.  If one group of Punjab & Bengal assembly demands for partition, it would be done.  Sindh assembly would decide whether they want partition or not.  In NWFP plebiscite will be used which will be based on limited suffrage.  If partition happened, then there would be boundary commission headed by Radcliffe.  Princely states had to join either state & were not allowed to remain free.  India would get dominion status immediately & Mountbatten would remain as the Governor General (of both India & Pakistan)

 Within Congress 2 groups emerged, one which accepted the plan & the other which completely opposed the plan.

 On 18th July 1947, the British Parliament passed Indian Independence Act.  On 14th August 1947 Pakistan was created & on 15th August 1947 India was created. Indian Independence Act On July 18, 1947  The British Parliament ratified the Mountbatten Plan as the "Independence of India Act-1947". The Act was implemented on August 15, 1947.  The Act provided for the creation of 2 independent dominions of India & Pakistan with effect from August 15, 1947.  Each dominion was to have a governor-general to be responsible for effective operation of the Act. The constituent assembly of each new dominion was to exercise the powers of the legislature of that dominion, & the existing Central Legislative Assembly & the Council of States were to be automatically dissolved  For the transitional period, i.e., till a new constitution was adopted by each dominion, the governments of the two dominions were to be carried on in accordance with the Government of India Act, 1935. Plan Balkan  Between March & May of 1947, Mountbatten decided that the Cabinet Mission Plan had become untenable & formulated an alternative plan.

 The plan envisaged the transfer of power to separate provinces (or to a confederation, if formed before the transfer), with Punjab & Bengal given the option to vote for partition of their provinces.

 The various units thus formed along with the princely states (rendered independent by lapse of paramountcy) would have the option of joining India or Pakistan or remaining separate.

 The plan was quickly abandoned after Nehru reacted violently to it. HISTORY March 23, 2018 1857 - the year that changed everything… Q. Explain how the uprising of 1857 constitutes an important watershed in the evolution of British policies towards colonial India.(2016) 1856  The British paramountcy was established over India during the tenure of Dalhousie.

 After annexing Punjab, he extended his control over Peshawar and Pathan tribes in the north west of India.

 He also annexed a large part of Burma and he extended empire in the east.

 The successor of Dalhousie, Lord Canning wanted to extend his control over the region.  The revolt of 1857 prevented the grand design of Canning from materializing. Who was the Governor-General of India during the Sepoy Mutiny?

(a) Lord Canning (b) Lord Dalhousie (c) Lord Hardings (d) Lord Lytton Answer: a Causes of the Revolt

Beginning and Spread of the Revolt

Consequences of the Revolt The Military Causes

 Uses of cartridges made from cow and pig fat (rumored) in new Enfield replaced the older . This use of fat was respectively offensive to the Hindus and Muslims  Overseas deployment  Differences in salaries for equal ranks .Special provisions for European soldiers  Refusal to pay Batta (allowance) to Indian soldiers.  Soldiers were considered inferior and higher posts were exclusively reserved for Britishers .  Post Office Act of 1854, withdrew the privilege of free postage so long enjoyed by the Sepoys.  Religious identities seemed to be in crisis as the military authorities forbade sepoys to wear caste and sectarian marks beards, or turbans.  Regular humiliation at the hands of British officers

 The British suffered defeats in 1st Afghan war (1838-1842) shattering the belief of their invincibility.  British suffered major reverses in the Punjab wars (1845-49) and the Crimean War (1854-56). The threat of conversion

 The Religious Disability Act of 1850 modified Hindu customs; a change of religion did not debar a son from inheriting the property of his father.

 The rumor was that the English were conspiring to convert the Indians to Christianity.

 Were promised promotions if they accepted the 'true faith'.

 Sepoys knew that the army was. maintaining chaplains at state cost. The Political Causes  The political prestige of the British suffered

 The paramountcy of the British was established through a number of devices such as direct as in the case of Mysore and Sindh.  The method of subsidiary alliance though initially designed to help and protect the local rulers against their wars with the others, also used to create such conditions whereby it led to annexation of these territories by the British.  Doctrine of Lapse– states under this revolted

 The house of the Mughals was humbled when Dalhousie announced in 1849 that the successor to Bahadur Shah would have to abandon the historic Red Fort .

 In 1856, Canning announced that after Bahadur Shah's death the Mughals would lose the title of kings and would be known as mere princes. The ruler of which one of the following States was removed from power by the British on the pretext of misgovernance?(2007)

(a) Awadh (b) Jhansi (c) Nagpur (d) Satara

Answer: a The Economic Causes

 High rate of taxation  Discriminatory tariff policy against Indian products and destruction of traditional handicrafts resulted into deindustrialization which resulted in unemployment  The peasants were ruined through exorbitant charges made from their lands by the new class of landlords established under Zamindari system introduced by the British.  Middle and upper classes, particularly in the North, were hard hit by their exclusion from the well-paid higher posts in the administration.  The craftsmen were destroyed by the influx of the British manufactured goods  Systems of law and administration: corruption and oppression The Socio-religious Causes  Foreignness of British rule : racial superiority , no social link

 Some reforms like Anti Sati Resolution (1829), Widow Remarriage Act (1856) , opening of Western education to girls were unpopular among orthodox Hindus.  The policy to tax religious schools further anguished both Hindus and Muslims.  Racial discrimination by British against Indians, forceful conversion to Christianity

 The English described the Hindus as barbarian with hardly any trace of culture or civilization, while Muslims were dubbed as bigots, cruel and faithless.  Idolatry was renounced and Hindus were dubbed as ignorant and superstitious. The Immediate Cause

 When the atmosphere was surcharged with an anti-British feeling the episode of the greased cartridges provided the spark which turned it into a conflagration.  The new 'Enfield Rifle' introduced by the British in the army needed a special type of cartridge which had a greased paper cover.  This paper had to be bitten off before the cartridge was loaded into the rifle – it was rumoured that the grease used in the paper was made of beef and pig fat.

 This angered both the Hindu and the Muslim sepoys - both the communities felt that their religions were at stake. The Mutiny began... Revolt  One group of historians and writers has asserted that the Revolt was the result of a widespread and well organized conspiracy.  They point to the circulation of chappattis and red lotuses, propaganda by wandering sanyasis, faqirs and .  They say that many of the Indian regimen s were carefully linked in a secret organization which had fixed 31 May 1857 as the day when all of them were to revolt.  It is also said that Nana Sahib and Maulavi Ahmad Shah of Faizabad were playing leading roles in this conspiracy.

 Other writers equally forcefully deny that any careful planning went into the making of the Revolt.  They point out that not a scrap of paper was discovered before or after the Revolt indicating an organized conspiracy, nor did a single witness come forward to make such a claim. The Spread of the Revolt On 29th March, 1857, the Brahmin sepoy, of Barrackpore refused to use the greased cartridge and fired at his adjutant.

The revolt broke out at Meerut on 10th May 1857.

The Meerut soldiers marched to Delhi and proclaimed the aged and powerless Bahadur Shah the Emperor of India.

Delhi was soon to become the Centre of Great Revolt and Bahadur Shah its great symbol. Bahadur Shah in return wrote to all chiefs and rulers to organize confederacy to overthrow British regime.

The entire Bengal army soon rose in revolt which spread quickly.

Awadh, Rohilkhand, the Bundelkhand, Central India, large parts of Bihar, and East Punjab all shook off British authority.

In many princely states, rulers remained loyal to their British administrators, but the soldiers revolted. For example, in Indore, the Holkars remained loyal but many of the Indore troops rebelled and joined sepoys. The Spread of the Revolt (2) Everywhere in Northern and Central India, the mutiny of sepoys triggered popular revolts of the civilian population. After the sepoys had destroyed British authority, the common people rose up in arms often fighting with spears and axes, bows and arrows. In many places, people revolted even before the sepoys did or even when no sepoy regiments were present. It is the wide participation by the peasants, the artisans, shopkeepers and zamindars which gave it real strength as well as the character of a popular revolt, especially in the areas included in present day U P and Bihar. Here the peasants and zamindars expressed their grievances by attacking money lenders and new zamindars, British law court, revenue offices. Much of the strength of the revolt of 1857 lay in Hindu-Muslim unity. Among the soldiers and people and among leaders, there was complete cooperation between Hindus and Muslims. The Leaders Centre Indian Leader(s) Delhi Bahadur Shah II ‘Zafar’ and Bareilly Khan Bahadur Khan Kanpur 1. [adopted son of BajiRao II]

2 .Tantia Tope [accountant of Nana Saheb]

3. Azeem Ullah Khan

Lucknow , mother of Birjis Qadar Jhansi Rani LaxmiBai Faizabad Maulvi Ahmadullah Shah (Leader of Wahabi movement) Allahabad Liyaqat Ali Jagdishpur (Bihar) and Amar Singh Patna Maulvi Pir Ali (leader of Wahabi movement)

DELHI On 12th May 1857, Delhi was seized by rebels.

Bahadur Shah II was persuaded to support.

Loss of Delhi was a prestige loss for British.

On 14th September 1857, British attacked.

On 20th September 1857, British regained Delhi.

Bahadur Shah & Zeenat Mahal fled to Humayun’s tomb.

 But were followed & captured by General Hudson.  3 sons killed & the king & queen exiled to Rangoon.  The king died in 1862. Bahadur Shah II  He was angered by the assertion of authority by the leaders of the sepoys.

 He vacillated between the desire to reign as Emperor and the desire to save his skin in case the Revolt was crushed by the British.

 His position was also undermined by his favorite Queen Zeenat Mahal and his sons who carried on intrigues with the enemy.

 His weak personality

 Old age

 Lack of qualities of leadership OUDH (LUCKNOW) The Nawab of Oudh exiled to Calcutta.

But his wife Begum Hazrat Mahal and 11 year old son continued to live in Oudh but in very poor condition. The revolt broke out on 4th June 1857.

Henry Lawrence, the British Resident, with some Europeans with some hundred sepoys took refuge in a Residency. Begum seized Residency and killed Henry & some others. In November, Sir Colin Campbell (Commander-in-Chief) attacked with Gorkha regiment. In March 1858, the city was finally recaptured.

The rebels driven to Nepal border to die in bad climate or captured by Gorkhas. Kanpur Nana Sahib, the adopted son of Peshwa Baji Rao was proclaimed the Peshwa.

He led the revolt in Kanpur with Tantia Tope, his assistant. Declared himself to be Zafar’s governor.

After a fierce battle, General Sir Hugh Wheeler surrendered on 27th June 1857.

Nana Saheb assured safe passage of British to Allahabad.

Angry Indians killed the passing British citizens.

On 6th December 1857 General Campbell occupied Kanpur.

Nana fled to Nepal and Tantia joined Rani Laxmibai.

JHANSI & GWALIOR In June 1857, the troops at Jhansi revolted Rani Laxmibai (widow of Raja Gangadhar Rao) declared ruler.

On 3rd April 1858, Sir Hugh Rose recaptured Jhansi.

She escaped to Kalpi (near Gwalior) where Tantia joined her after Kanpur was lost.

Both marched to Gwalior.

Sir Hugh Rose also marched to Gwalior and captured in June 1858.

Rani died fighting bravely.

Tantia escaped southward but was betrayed by his friend Man Singh. Tantia was finally hanged. With reference to the ‘revolt of the year’ who of the following was betrayed by ‘friend’ captured and put to death by the British?

(a) Nana Sahib (b) Kunwar Singh (c) Khan Bahadur Khan (d) Answer: d  At Bareilly, Khan Bahadur, a descendant of the former ruler of Rohilkhand, was placed in command.

 Not enthusiastic about the pension being granted by the British, he organized an army of 40,000 soldiers and offered stiff resistance to the British.

 In Bihar, the revolt was led by Kunwar Singh, the zamindar of Jagdishpur.

 An old man in his seventies, he nursed a grudge against the British who had deprived him of his estates. He unhesitatingly joined the sepoys when they reached Arrah from Dinapore.

 Maulvi Ahmadullah of Faizabad was another outstanding leader of the revolt.

 He was a native of Madras and had moved to Faizabad in the north where he fought a stiff battle against the British troops. The Direct Results The Revolt of 1857 exposed the danger involved in allowing a commercial organization to rule over a country. Consequently, the British Parliament by an Act transferred the control of the Indian government from the East India Company to the British Crown. The said Act passed on 2 August, 1858, was known as the Act for the Better Government of India or the Government of India Act, 1858. Queen Victoria, by a Proclamation announced on 1 November, 1858, directly assumed the responsibility of the Indian administration in her own hands.

And it was in accordance with the Queen's Proclamation that the honorific title of Viceroy (Crown's personal representative )was added to the Governor-General of India. The supreme executive and legislative authority in India henceforth came to be called the 'Governor- General and the Viceroy'.

Lord Canning, so far known as the Governor-General of India, also became the 1st Viceroy of the country. Continued

In her Proclamation, Queen Victoria also announced certain changes in the governmental policy to be pursued by the British in India.

Secretary of State of India appointed with a council of 15 members to assist him.

The Revolt of 1857 led to an extensive reorganization of the army and the civil administration.

It must, however, be remembered that in spite of all these bold theoretical statements hardly any change occurred in the basic exploitative nature of the British rule in India. The Government of India Act 1858 Company’s territories were to be vested in the Queen and be governed by her. While authority over India had previously been wielded by the Directors of the Company and the Board of Control, now this power was to be exercised by a Secretary of State for India aided by a Council. The Secretary of State was a member of the British Cabinet and as such was responsible to Parliament. Thus ultimate power over India remained with Parliament. The Crown was empowered to appoint a governor general & governor of presidencies. Provision created for Indian Civil Services under the Secretary of State All property of East India Company were transferred to the Crown Victoria’s Proclamation

Queen Victoria Proclaimed that  No state would be annexed.  No more intervention in religious matters.  No conversion in religion by force.  No discrimination in recruitment of Army Services. Note – Victoria became the ‘Empress of India’ not in 1858 but in 1876. (2014) What was/were the object/objects of Queen Victoria’s Proclamation (1858)?

1)To disclaim any intention to annex Indian States

2)To place the Indian administration under the British Crown

3) To regulate East India Company’s trade with India

Select the correct answer using the code given below. a)1 and 2 only b) 2 only c) 1 and 3 only d) 1, 2 and 3 Ans a The Indirect Results  The Revolt further widened the difference between the ruler and the ruled.

 During the post-Revolt years the British rulers, in order to maintain their supremacy in India, deliberately followed a policy of communal disharmony (the Divide & Rule policy).

 The seed of communal disharmony planted by the English in India sprouted like a poison tree, and led to the 90 years later.

 Although the British government in India did not pursue a policy of territorial expansion in India during the post-1857 days, the period was yet marked by a new era of economic exploitation by the British in India.

 From now on the British pursued a policy of opposing the educated middle class and supporting the landlords and the native princes. Measures Adopted by the British After 1857

Appease certain sections of Indian Society – the ‘divide and rule’ policy

The British government decided not to interfere in the matter of religion of Indians.

This derailed the socio-religious reforms by government Strategic changes in the army  The proportion of Europeans to Indians in the army was raised and fixed at one to two in the Bengal army and two to five in the Madras and Bombay armies  The older policy of excluding Indians from the officer corps was strictly maintained.  All sensitive posts were reserved only for the Europeans  European troops were kept in key geographical and military positions  Caste based battalions were raised - new units recruited from castes so far under- utilised by the British & from the minority so-called "Martial Races", such as the Sikhs & the Gurkhas.

 Newspapers, journals, & nationalist publications were prevented from reaching the soldiers. Reasons for Failure  British forces were better equipped with technology and arms.

 Limited territorial spread :The revolt didn’t spread to entire country. The southern and western parts of India remained more or less unaffected.

 Lack of complete nationalism- Sindhias, Holkars, Nizam & others actively helped the British.

 Certain classes and groups did not join and, in fact, worked against the revolt.

 Big zamindars acted as "breakwaters to storm"; even Awadh tahasildars backed off once promises of land restitution were spelt out.  Moneylenders and merchants suffered the wrath of the mutineers badly and anyway saw their class interests better protected under British patronage. Continued  Lack of coordination between sepoys, peasants, zamindars and other classes.

 The rebel units did not have a common plan of action, or authoritative heads, or centralized leadership.

 Sepoys were also poorly organized. The sepoys were also ill-disciplined sometimes behaved more like a riotous mob than a disciplined army.

 No vision for the post mutiny institutions & the leaders didn’t have any forward looking program.

Modern educated Indians viewed this revolt as backward looking, and mistakenly hoped the British would usher in an era of modernization. By one estimate, not more than one-fourth of the total area and not more than one- tenth of the total population was affected. Important Books on the Revolt

Book Author The First Indian War of Independence-1857-59 Karl Marx

Rebellion, 1857 : A Symposium P.C. Joshi

The Sepoy Mutiny & the rebellion of 1857 R.C. Mazumdar

1857 S.N. Sen Causes of Indian Revolt Saiyed Ahmad Khan The First War of Indian Independence V.D. Savarkar

“On the whole, it is difficult to avoid the conclusion that so-called First National War of Independence is neither First, nor National, nor a war of independence.” - R.C. Majumadar Sepoy Mutiny or 1st War of Indian Independence?

• Not just a Sepoy Mutiny - it was not just confined to the sepoys or soldiers, but was much broader in base. The civilian population, aristocracy, peasants, religious leaders played active role in the revolt.

• Not completely the First War of Independence – India was merely a geographical term then; leaders fought for their own personal reasons to get back what they had lost; there was no broad vision of a unified India. South India remained more or less unaffected. Also, the war was won by the British through the support of Indians.

Therefore, the exact nature of the revolt was somewhat between these extreme views.

HISTORY March 23, 2018 India in 18th century Decline of Mughal Empire Bahadur Shah(1707 C.E. – 1712 C.E.)  Aurangzeb died in 1707  Bahadur Shah emerged victorious  His original name was Muazzam.

 Learned dignified tolerant

 Attempted better ties with Rajput, Marathas, Sikhs, Bundelas & Jats  He adopted a liberal policy with the Rajputs. In 1709 C.E. He recognized Ajit Singh as the Rana of Marwar.

 Sahu the son of Shambaji who was imprisoned by Aurangzeb was released in 1707 C.E. by Bhadhur Shah & granted him Sardesh Mukhi but not chauth.

 Made peace with Guru Govind Singh, the 10th Sikh Guru.

 Banda Bahadur or Banda Bairagi, the greatest Sikh leader after Guru Govind Singh was defeated by Bahadur Shah at Lohgarh & he reoccupied Sirhind in 1711 C.E. Jahandar Shah(1712 C.E – 1713 C.E)  Period of ambitious nobles

 Zulfiqar Khan supported him

 He was a very weak ruler. He was dominated by his wife Lal Kunwar & the nobles of the court.

 His nephew Farukh Siyar with the cooperation of the Syed Brothers (Syed Abdullah Khan & Syed Hussain Ali Khan) murdered Jahandar Shah & usurped the throne.

Farukh Siyar(1713 C.E. – 1719 C.E.)  The Syed Brothers were the real rulers during his reign. Syed Abdullah Khan was his PM & Syed Ali Khan was his CiC.

 The Jats under Churaman revolted against the Mughals during his reign.

 Banda Bairagi was defeated & killed in 1716 C.E.

 He issued a Farman granting the East India Company many trading privileges which is considered to be the Magna Carta of the trade for Company in India. (1717 C.E.)

 In 1719, the Syed brothers entered into an agreement with Peshwa Balaji Vishwanath which granted Swarajya of Shivaji to Sahu and the right to collect Chauth and Sardeshmuki of 6 Mughal provinces in the Deccan.

 The Syed brothers with the help of the Marathas killed Farruk Siyar.

Muhammed Shah(1719 – 1748 C.E.)  Muhammad Shah made emperor as 18 year old by Saiyyid brothers  But looked down as traitors  He with the help of few trusted nobles like Chin Qulich Khan & Sadat Khan killed the Syed Brothers.Younger killed & elder defeated  His original name was Roshan Akhtar.  Chin Quli Khan adopted the title Nizam-ul-Mulk & became his wazir in 1722 C.E.  Nizam-ul-Mulk decided to move  Bengal acquired virtual independence  Sadat Khan & Daud Khan established themselves virtually as independent rulers in Oudh & Rohilkhand respectively.  In 1739 C.E., the Maratha Peshwa Baji Rao-I invaded Delhi.  Niamat Khan (veena player), Feroz Khan were some of the prominent musicians in his court. The emperor gave them the titles Sadarang & Adarang.  1739 C.E.: Nadir Shah the Persian invader invaded India. He defeated the Mughals in the Battle of Karnal.

 In 1748 C.E.,Abdali invaded India for the 1st time but was defeated in the battle of Manpur. Ahmad Shah(1748 – 1754 C.E.)  He was a weak & incompetent ruler. He was fully dominated by the Queen Mother Udham Bai & the eunuch superintendent of the Harem Javid Khan.

 Ahmad Shah Abdali, plundered Punjab & Multan twice during his reign. Abdali then captured Punjab & Sindh provinces & installed his men in charge of these provinces.

 Initially Safdarjung was his Wazir, but was later removed & Imad-ul-Mulk was made his Wazir.

Alamgir II (1748 – 1754 C.E.)  Imad-ul-Mulk killed Ahmad Shah & made Alamgir II as the new sultan.

 He was a puppet in the hands of the Wazir Imad-ul-Mulk.

 Ahmad Shah Abdali attacked India in 1756 C.E. He plundered & routed Delhi & Mathura.

 Imad-ul-Mulk killed the emperor & made Ali Gauhar the son of Alangir II to ascend the throne. SHAH ALAM II (1758 C.E – 1806 C.E)  Ali Gauhar ascended the throne under the title Shah Alam II

 He had to remain in exile from Delhi for 12 years till 1772 C.E.  He participated in the Battle of Buxar (1764 C.E.).

 He became the 1st Mughal ruler to receive pension (Rs. 26 lacs) from the British.

 The third battle of Panipat, 1761 C.E. was fought during the period.  Shah Alam supported by the British and escorted by the Maratha leader Mahadji Scindia was assisted to annex the throne at Delhi.

 Later Ghulam Kadir Rohella, the Afghan chief blinded Shah Alam II.

 As a blind ruler, Shah Alam II signed the documents surrendering Delhi to the Britishers in 1803 C.E. Akbar II (1806 -1837 C.E.)  He was king in name only.

 He was treated as a pensioner of the East India Company. (1837 – 1857 C.E.)  Bahadur Shah was the last Mughal King.

 The British controlled almost the whole country. Bahadur Shah’s command did not go beyond the walls of the Red fort.

 He was a prolific poet and a Calligrapher. He wrote under the pen name Zafar.

 Urdu poets like Mirza Ghalib, Zauk, Momin & Daagh were his contemporaries. 2 types of states  Succession states like Hyderabad Awadh & Bengal

 Others like Marathas Jats Afghans Punjab etc. Hyderabad  Nizam ul mulk Asaf Jah

 Tolerant

 1748 death issues with succession

 Asaf Jahi dynasty Carnatic

 Came under the Deccan thus under Nizam

 Deputy governor known as Nawab of Carnatic

 Made his office hereditary

 Nawab Saadatullah Khan made his nephew dost Ali his successor without approval of Nizam

 Founder of Nawayath dynasty Bengal  1700: Murshid Quli khan appointed diwan of Bengal

 After Aurangzeb's death founded an independent kingdom

 Succeeded by his son in law Shuja ud din

 He died in 1739 & was succeeded by his son Sarfaraz Khan

 1739: Ali Vardi khan revolts ,kills Nawab & becomes subedar of Bengal.

 Tolerant appointed Hindus as revenue farmers

 Didn’t build big army

 Didn’t tackle corruption Awadh  Saadat khan appointed governor in 1722

 Bold energetic iron willed & tolerant

 1739 dead by then virtually independent

 Succeeded by his nephew Safdar Jang who became the Wazir of Mughals in 1748

 Died in 1754

 Prolonged peace resulted in growth of Awadhi culture Mysore

 Nominal part of Mughal empire

 King Chikka Krishna Raj a puppet at the hands of 2 ministers

 Modern arsenal in Dindigal

 1761 established his authority

Tipu Sultan  Complex but innovator  New calendar ,new coinage system new scales of weights & measures  French revolution: Planted a 'Tree of Liberty' at Seringapatam & he became a member of a Jacobian Club.

 Only one to understand threat of the English

 Tried to establish modern navy  Mysore flourished & comparatively was better than others  Sent emissaries to Russia France Iran Turkey  Traded with China  Promoted trade with Russia & Arabia  Described as religious fanatic  His personal library contained books on such diverse subjects as religion, history, military science, medicine, & mathematics. Kerala

 Calicut, Cochin , Chirakkal, Travancore

 Martand Varma 1729-58

 Defeated the Dutch: (1741), resulting in the complete eclipse of Dutch power in Malabar.  Padmanabhaswamy temple recreated

 By 1763 only Calicut, Cochin , Travancore

 Rama Varma 1758-98

 Malyalam literature

 Sanskrit scholarship Rajput States  Raja Sawai Jai Singh Amber(1699-1743 rule)

 Astronomer & Reformer

 Man of Science & arts

 Founded Jaipur: Made it a seat of science

 Observatories in Ujjain, Jaipur, Delhi, Varanasi , Mathura  He drew up a set of tables, entitled Zij Muhammad Shahi, to enable people to make astronomical observations.  Euclid's "Elements of Geometry", translated into Sanskrit & also several works on trigonometry, & Napier's work on the construction & use of logarithms.

 Law to reduce expenditure on daughter’s marriage Wars, Battles and Treaties Anglo-French Carnatic Wars

 Why South?

 Aurangzeb had stalled

 But Nadir Shah showed the decay

 West Marathas

 Bengal Ali vardi Khan

 South central power had disappeared & 1748 death of Nizam

 Both English and French desired it

Anglo-French Carnatic Wars

Joseph Dupleix Robert Clive Anglo-French Wars

War of Austrian Succession :1740-1748

France, Prussia, Spain, Bavaria, Britain ,Habsburg, Hanover, Dutch Sicily, Naples, Genoa, Sweden Republic, Saxony, Sardinia, Russia First Carnatic War 1746-1748

 Madras vs. Pondicherry

 Treaty of Aix-la-Chappelle

 Madras to be returned to British in exchange of Louisburg in North America to France

Second Carnatic War 1748-1754  Civil Wars in Hyderabad and Carnatic on the issue of succession.  Carnatic Chanda sahib vs Anwaruddin the Nawab

 Hyderabad Muzaffar Jang (grand son) vs Nasir Jang (son)

 Anwaruddin killed but his son Muhammed Ali fled to Trichi

 France was initially successful in both places

 While French were in Trichi English attacked Arcot so French had to raise the siege.

 Dupleix failure to capture Trichi along with his political ambitions & ruinous expenses sealed his fate.

 Treaty of Pondicherry – Dupleix returns to France( English demand) Third Carnatic War 1757-1763

 This war was an echo of seven year war in Europe & thus ended the short term peace in India between the Europeans.

 1761: Wandiwash English (Eyre Coote) defeat the French (de Lally)

 1763 Treaty of Paris

 Pondicherry & few other territories were returned to French but was never to be fortified.

 This sealed French ambitions in India. British occupation of Bengal

 Company had secured the Mughal Farman in 1717

 Freedom to export or import without paying any taxes & the right to issue dastak or passes for movement of such goods

 Farman caused troubles between Nawab & the English

Battle of Plassey  1756: Alivardi died & was succeeded by his 23 year-old grandson, Siraj-ud-daulah.

 Combination of a ferocious temper and a feeble understanding.

 He was particularly suspicious of the large profits made by the European companies in India ,asked them to trade on similar terms like Murshid’s times .

 When the British and the French started improving their fortifications in anticipation of another war between them, he immediately ordered them to stop such activities as they had been done without permission. French agreed but not the English.

 When the British refused to cease their constructions, the Nawab led a detachment of 3,000 men to surround the fort & factory of & took several British officials as prisoners, before moving to Calcutta. Continued  The city was occupied on 16 June by Siraj’s force & the fort surrendered after a brief siege on 20 June.

 The prisoners who were captured at the siege of Calcutta were transferred by Siraj to the care of the officers of his guard, who confined them to the common dungeon of Fort William known as The Black Hole.

 English took refuge in Fulta near the sea.

 Robert Clive entered into agreement with

(Mir Bakshi)

Continued The other traitors of were as follows:

 Jagat Seth: A Marwari Banker. After 9 years of the Battle of Plassey, the entire family of Jagat Seth was beheaded by .

 Omi Chand or Amir Chand : He tried to get 5% from the treasure after Mir Jafar becomes Nawab but was deceived by the British by fake treaty & this shock was enough to plunge him into mental retardation. He survived for some 10 years & died anonymously.

 Manik Chand: officer in charge of Calcutta.

 Rai Durlab: He was the treasurer of Nawab.  Ghaseti Beghum: The rich maternal aunt of Nawab.  Khadim Khan(commended large army) Battle of Plassey  The battle was merely a skirmish but in its results was one of the most decisive battles of Indian history as it paved the way for the foundation of British rule.

 Only Mir Madan & Mohan Lal fought

 Nawab was put to death

 Mir Jafar became a puppet in the hands of Clive who couldn’t satisfy the demands & was forced to resign in 1760. Battle of Buxar  Mir Kasim succeeded him but he too couldn’t satisfy the growing demands.  Was an able & competent leader  Tried to check the misuse of dastak

 Abolished all inland duties

 He shifted his capital to Monghyr.

 War broke with the company lost in a series of battle & he fled to Oudh to organize a confederacy with Shuja ud Daula Nawab of Oudh & shah Alam II.

 The combined armies were defeated Mir Kasim fled while the other 2 surrendered. Results of Battle of Buxar  Treaty of Allahabad was signed between the British and Shuja ud daula Nawab of Oudh & Shah Alam II.

 Shuja ud Daula was confirmed in his possessions on the following conditions:

 Nawab surrendered Allahabad & Kara

 Pay 50 lakh to company

 forced to maintain English forces.

 Shah Alam was taken into company’s protection

 Was to reside in Allahabad.  Granted the company the diwani of Bengal Bihar & Odessa. Continued  1763 Mir Jafar made the Nawab again

 On his death his son Nizam ud Daulah made the Nawab

 Robert Clive was appointed the 1st governor of Bengal & signed a new treaty

 Disband it’s army & administer Bengal through a deputy Subedar who was nominated by British

Dual System of administration  The dual government began wherein the administration of Bengal was carried out by 2 heads with the Nawab of Bengal being the nominal head & company as the dewan having control over revenues as well as police and judicial powers.  It was introduced by Robert Clive.  The British East India Company obtained the actual power; where as the responsibility & charge of administration was entrusted to the Nawab of Bengal.  Under this the British administration acquired both the functions of the Diwani or revenue & Nizamat or civil administration of Bengal from 2 distinct sources, Diwani from the Mughal emperor & Nizamat from the Nawab of Bengal.  Company was authorized to accumulate revenues of the Bengal province as the Diwan.  Moreover with the right to appoint the Deputy Subahdar, the British obtained the designation to control the police and judicial powers or the Nizamat.  The difficulty was created by the fact that the servants of the Company did not undertake duties as Diwan or Nizam in their own person.  The nominal head of the administration was Deputy Naib Mohammad Khan at Murshidabad & Raja Shitab Rai at Patna. Bengal  What made the East India Company prosperous and powerful?

 What happened to the spice trade?

 How did they get a monopoly?

 Why was the Army of the EIC loyal?

 What was the effect of Indian victories of EIC in England?

 What was the impact on local economy and society? Bengal

 Why did the EIC win the Battle of Plassey?

 Why did the Battle of Buxar occur? Anglo–Mysore Wars The Anglo–Mysore Wars were a series of wars fought in India over the last 3 decades of the 18th century between the & the British East India Company, represented chiefly by the Madras Presidency.

The 4th war resulted in the overthrow of the house of Hyder Ali & Tipu, in 1799, & the dismantlement of Mysore to the benefit of the British and their allies. First Anglo–Mysore War (1767–1769)  Saw Hyder Ali gain some measure of success against the British but suffer heavy defeats at the hands of the Marathas.

 British wanted Northern circars on rent for connection between madras & Bengal but Nizam rejected it so they got firman from Mughal ruler

 Hyder Ali's alliance with the Nizam of Hyderabad against the British too was a failure owing to defeats of their combined power against the British & later the spread of mutual suspicion between the 2 Islamic powers.

 Treaty of Madras in 1769  Support each other if attacked  Territories occupied were restored to each other. Second Anglo–Mysore War (1780–1784)  Witnessed bloodier battles with fortunes fluctuating between the contesting powers.  Repeating his earlier exploits, Hyder inflicted one defeat after another on the British armies in the Carnatic & forced them to surrender in larger numbers.  Warren Hastings bribed Nizam by giving Guntur thus leading him to withdraw

 In 1781 the Eyre Coote defeated Hyder Ali at Porto Novo and saved Madras.

 Hyder died in 1782

 Hyder & Tipu prevailed & this led to the last British-Indian treaty with an Indian ruler on equal footing.

 Treaty of , at which both sides agreed to restore the other's lands to the status quo ante bellum.  Thus no tangible benefits. Third Anglo–Mysore War (1789–92)  Tipu an ally of France, invaded the nearby state of Travancore, which was a British ally.

 Lord Cornwallis, had succeeded through shrewd diplomacy in isolating him by winning over the Marathas, the Nizam, & the rulers of Travancore and Coorg.

 A resounding defeat for Mysore.

 The war ended after the siege of Seringapatam and the signing of the Treaty of Seringapatam, according to which Tipu had to surrender half of his kingdom to the Company.

 Paid 330 lakhs of rupees as indemnity. Fourth Anglo–Mysore War (1799)  Saw the defeat of Tipu and further reductions in Mysorean territory.

 Mysore's alliance with the French was seen as a threat to the Company.

 Nizam & the Marathas launched an invasion from the north.

 The British won a decisive victory at the Battle of Seringapatam in 1799.

 Tipu was killed. Much of the remaining Mysorean territory was annexed by the British, the Nizam & the Marathas.

 The remaining core, around Mysore & Seringapatam, was restored to the Indian prince belonging to the Wodeyar dynasty.

 An important result was the complete elimination of the French threat to British supremacy in India.  Mysorean The Rise and Fall of the Maratha Power Marathas  Most important challenger to the decaying Mughal power.

 Number of brilliant commanders

 Lacked unity

 Lacked the outlook & programme which were necessary for founding an all-India empire.

 They did however succeed in waging continuous war against the Mughal Empire, till they destroyed it. Evolution of Peshwaship  Shahu, grandson of Shivaji, had been a prisoner of Aurangzeb since 1689.

 Was released in 1707

 Soon a civil war broke out between Shahu at Satara & his aunt Tara Bai at Kolhapur who had carried out an anti-Mughal struggle since 1700 in the name of her son Shivaji II after the death of her husband .

 Arising out of the conflict between Shahu & his rival, a new system of Maratha government was evolved under the leadership of Balaji Vishwanath, the Peshwa (1713-1818 C.E.) of King Shahu.  With this change began the period of Peshwa domination in Maratha history in which the Maratha state was transformed into an empire.

 He and his son made the Peshwa the functional head of the Marathas. Balaji Vishwanath (1713-1720 C.E.)  Balaji Vishwanath started his career as a small revenue official. He played a crucial role in the victory of Sahu over Tarabai in the battle of Khed in 1707 C.E.  After Sahu’s (1707-48 CE) coronation as Chatrapathi at Satara, Balaji was made his Sena Karte (organizer of forces)  In 1713 C.E., he was raised to the post of Peshwa, this marked the domination of Peshwa supremacy in Maratha politics.

 He was the first Maratha official to reach Delhi & involve in the internal matters of the Mughal rulers.  He secured the release of Sahu’s mother.

 In 1719 C.E., he got certain rights from Farruk Siyar.

 He initiated the northward extension of Maratha kingdom, which was taken further by his son Baji Rao. Baji Rao I(1720-1740 C.E.)  Succeeded by his 20-year old son Baji Rao I.  The Maratha power reached its zenith under him.  Bold & brilliant commander & an ambitious & clever statesman.  Described as "the greatest exponent of guerrilla tactics after Shivaji.  He preached the ideal of Hindu Padpad Shahi (All India Hindu Empire). He said that the Maratha flag will fly from Attock to Cuttack (river Krishna).  He initiated a system of confederacy among the Maratha chiefs – under this system, each Maratha chief was assigned a territory which would be administered autonomously. As a result, many Maratha families like Gaekwad of Baroda, the Bhonsles of Nagpur ,Holkars of Indore, Sindhias of Gwalior & the Peshwas of Poona became prominent.  He defeated the Nizam of Hyderabad in the battle of Bhopal.  1737-38 C.E., he attacked Delhi & defeated the Mughal forces. Muhammed Shah signed the treaty of Sironj, which gave Marathas complete sovereignty over the whole of Malwa & the entire territory between Narmada & Chambal.  1739 C.E: He defeated the Portuguese & occupied the parts of Salsette & Bassein.  The marriage of Baji Rao I with Mastani a Muslim princess created lot of trouble. Balaji Baji Rao (1740-1761 C.E.)  His18-year old son Balaji Baji Rao (Nana Saheb) was new Peshwa .

 He continued with the policy of northward expansion.  He was as able as his father though less energetic.  Shahu died in 1749 & by his will left all management of state affairs in the Peshwa's hands.  The office of the Peshwa had already become hereditary & the Peshwa was the de facto ruler of the state.  The deed also required the peshwas to protect the high-esteem for the descendants of Tara bai.  Balaji Baji Rao, imprisoned Ram Raja (1749-77 C.E.) after the death of Sahu in 1748 C.E. This imprisonment virtually seized the Maratha monarchy as a symbol of political authority & made Peshwas as the centre of political authority.  Now he became the official, head of the administration and, as a symbol of this fact, shifted the government to Poona, his headquarters. Threat : Ahmad Shah Abdali  Peshwa dispatched a powerful army under the nominal command of his minor son Vishwas Rao, the actual command being in the hands of his cousin Sadashiv Rao Bhau.

 Contingent of European style infantry & commanded by Ibrahim Khan Gardi.  Balaji Baji Rao, though conquered lot of territories, lacked his father’s diplomatic skills.  He had cultivated animosity of all neighbouring rulers.  Marathas under Balaji Baji Rao became everybody’s enemy & nobody’s friend. They had attacked everyone including Rajputs, Jats, Bengal & Orissa.  The Marathas now tried to find allies among the northern powers. They had to fight their enemies all alone, except for the weak support of Imad-ul-Mulk

 Senior Maratha commanders constantly bickered with each other. Battle of Panipat  14 January 1761

 The Peshwa's son, Sadashiv Rao Bhau & numerous other Maratha commanders perished on the battle field as did nearly 28,000 soldiers.

 Those who fled were pursued by the Afghan cavalry & robbed & plundered by the Jats, Ahirs, & Gujars of the Panipat region.

 The Peshwa, who was marching north to render help, was stunned by the tragic news. Already seriously ill, his end was hastened & he died in June 1761.

 This Battle did not decide who was to rule India but rather who was not. The third battle of Panipat was fought in 1761. Why were so many empire- shaking battles fought at Panipat?2014 India in 1761 Aftermath  The 17 year old Madhav Rao became the Peshwa in 1761.

 He was a talented soldier & statesman.

 Within short period of 11 years, he restored the lost fortunes of the Maratha Empire.

 In 1771, the Marathas brought back to Delhi Emperor Shah Alam, who now became their pensioner.

 Died 1772 First Anglo-Maratha War (1775–1782)

 First of 3 Anglo-Maratha wars fought between the Company and Maratha Empire.

 The war began with the Treaty of Surat & ended with the Treaty of Salbai.

 After the death of Madhav Rao Peshwa, his brother Narayanrao became Peshwa. However, , their uncle, had his nephew assassinated in a palace conspiracy & declared him as Peshwa, although he was not the legal heir.

 Narayanrao's widow, Gangabai, gave birth to a posthumous son, who was legal heir to the throne. The newborn infant was named 'Sawai' Madhavrao (Sawai means "One & a Quarter").

 12 Maratha chiefs, led by Nana Phadnavis directed an effort to name the infant as the new Peshwa & rule under him as regents. Continued  Raghunathrao, unwilling to give up his position of power, sought help from the British at Bombay & signed the Treaty of Surat .

 But the British Calcutta Council condemned the Treaty, sending Colonel Upton to Pune to annul it & make a new treaty with the regency. The Treaty of Purandhar (1776) annulled that treaty, Raghunathrao was pensioned and his cause abandoned, but the revenues of districts were retained by the British.

 Treaty of Salbai: After the British defeat, Warren Hastings through Mahadji Sindhia proposed a new treaty between the Peshwa and the British that would recognize the young Madhavrao as the Peshwa & grant Raghunathrao a pension.

 It also guaranteed peace between the two sides for 20 years, thus ending the war.  Moreover, the Treaty enabled the British to exert pressure on Mysore as the Marathas promised to help them in recovering their territories from Hyder Mahadji Sindhia  He organized a powerful army with the help of French officers & established control over Emperor Shah Alam in 1784.

 From the Emperor he secured the appointment of Peshwa as the Emperor's Deputy (Naib-i-Munaib) on the condition that Mahadji would act on behalf of the Peshwa.

 But he spent his energies in intriguing against Nana Phadnis & Holkars of Indore.

 Died in 1794.

 Mahadji Sindhia, Tukoji Holkar, Ahilya Bai Holkar, Peshwa Madhav Rao & Nana Phadnis, the man who had kept the Maratha confederacy together for the last 30 years, all were dead by the year 1800. Second Anglo-Maratha War  Sawai Madhav Rao died in 1795

 Succeeded by the worthless Baji Rao II, son of Raghunath Rao.

 Marathas were the only major Indian power left outside the sphere of British control.

 The British had by now decided to put an end to the Maratha challenge.

 Wellesley now turned his attention towards them & began aggressive interference in their internal affairs.

 British divided, the warring Maratha sardars through clever diplomacy & then overpowered them in separate battles during the second Maratha War. Second Anglo-Maratha War (1803–1805)  In October 1802, Peshwa Baji Rao II & Sindhias were defeated by , at the Battle of Poona.

 Peshwa fled to British protection & in December concluded the Treaty of Bassein with the Company ceding territory for the maintenance of a subsidiary force & agreeing to treaty with no other power.

 This act on the part of the Peshwa, their nominal overlord, horrified and disgusted the Maratha chieftains; in particular, the Sindhias & the Bhonsles contested the agreement.

 On December 17 1803, Raghoji II Bhonsle signed the Treaty of Deogaon with the British after the Battle of Adagaon/Argaon .

 On 30 December 1803, Daulat Scindia signed the Treaty of Surji-Anjangaon Continued  Wellesley turned to Holkar, but Yeshwant Rao Holkar proved more than a match for the British.  He began hostilities with the British by securing the alliance of the Raja of Bharatpur.  Using traditional Maratha tactics of mobile warfare & in alliance with the Jats, he fought British armies to a standstill

 Moreover, overcoming his age-old antagonism to the Holkar family, Sindhia began to think of joining hands with Holkar.

 On the other hand, shareholders of Company discovered that policy of expansion through war had increased debt from £ 17 million in 1797 to £ 31 million in 1806.

 Moreover, Britain's finances were getting exhausted at a time when Napoleon was once again becoming a major threat in Europe. Result  Bhonsle and Sindhia became subsidiary allies of the Company.

 Again the blind Emperor of India became a pensioner of the Company.

 The Peshwa became a disgruntled puppet in their hands.

 By the Treaty of Rajghat in 1805, Holkar got back most of his territories.

 Directors of the Company felt that time had come to check further expansion, to put an end to ruinous expenditure.

 Wellesley was therefore recalled from India

 Wellesley's expansionist policy had been checked near the end.

 East India Company became the paramount power in India. Third Anglo-Maratha War (1817-1818)

 Last bid by Marathas to overthrow the British

 The lead in organizing a united front of the Maratha chiefs was taken by the Peshwa who was smarting under the rigid control exercised by the British Resident.

 However, once again the Marathas failed to evolve a concerted & well-thought out plan of action.

 Peshwa attacked the British Residency at Poona in November 1817. Appa Sahib of Nagpur attacked the Residency at Nagpur & Madhav Rao Holkar made preparations for war.

Result  Kingdom of Satara was founded and given to the descendant of Shivaji who ruled it as a complete dependent of the British.

 Lord Hastings compelled Sindhia to accept British suzerainty, and defeated the armies of the Peshwa, Bhonsle & Holkar

 Holkar & Bhonsle accepted subsidiary forces.

 Peshwa was dethroned and pensioned off at Bithur near Kanpur

 Maratha confederacy dissolved and peshwaship abolished.

 All the Maratha chiefs had to cede to the Company large tracts of their territories.

 The Rajputana states had been dominated for several decades by Sindhia & Holkar.  After the downfall of the Marathas, they lacked the energy to reassert their independence & readily accepted British supremacy. Continued  By 1818, the entire Indian sub-continent except Punjab & Sindh had been brought under British control.  Part of it was ruled directly by the British & the rest by a host of Indian rulers over whom the British exercised paramount power.

 These states had virtually no armed forces of their own, nor did they have any independent foreign relations.

 They paid heavily for the British forces stationed in their territories to control them.

 They were autonomous in their internal affairs, but even in this respect they acknowledged British authority wielded through a Resident.

 They were on perpetual probation.

 On the other hand, the British were now free to 'reach out to the natural frontiers of India.' Subsidiary Alliance

 The doctrine was introduced by Lord Wellesley.

 Early in his governorship Wellesley adopted a policy of non-intervention in the princely states, but he later adopted the policy of forming subsidiary alliances.

 This policy was to play a major role in British expansion in India.

Lord Wellesley (1798-1805) Main principles :  An Indian ruler entering into a subsidiary alliance with the British had to accept British forces within his territory & also agree to pay for their maintenance.  The ruler would accept a British Resident in his state.  The Indian ruler would not enter into any further alliance with any other power, nor would he declare war against any power without the permission of the British.

 The ruler would not employ any Europeans other than the British, and if he were already doing so, he would dismiss them.

 In case of conflict with other state, he would agree to resolution decided upon by the British.

 The ruler would acknowledge the East India Company as the paramount power in India.  In return for the ruler accepting its conditions, the Company undertook to protect the state from external dangers and internal disorders.

 If the Indian rulers failed to make the payments required by the alliance, then part of their territory was to be taken away as a penalty. Subsidiary Alliance System by Lord Wellesley  Wellesley made the Nawab & Nizams subsidiary allies by signing almost 100 such treaties.

 Initially Wellesley compelled the friendly rulers to accept this alliance.

 The policy of subsidiary alliance was 1st used by Wellesley in dealing with the Nizam.

 Wellesley neutralized the Nizam by getting him to sign the Subsidiary alliance to replace his French detachments.

 He also forbade Nizam to correspond with the Marathas without British consent.

 As the Nawab was a French protégé, he had appointed many Frenchmen at his court, but after this treaty, he was forced to dismiss the French employees.

 Marathas in Deccan had not entered into any kind of treaty, but still they were neutralized by Wellesley by a promise of share in the spoils of Tipu. After that only Wellesley demanded submission of Tipu & followed an invasion. In summary, the system of Subsidiary Alliance could be any of the following:

 The company lent its army in place of the Cash

 Company kept the armies near the border of the Protectorate and collected cash.

 Company kept the army inside the border for protection and collected cash.

 Company kept its army inside the border of army and got some territories.  The last among the above given 4 types was dangerous.

 It was Nawab of Oudh that entered into this kind of arrangement in 1801 (Treaty of Lucknow) & ceded half of Awadh to the Company & also agreed to disband his troops in favor of a hugely expensive, British-run army.

 After this, the British were able to use Oudh's vast treasuries, repeatedly digging into them for loans at reduced rates. They also got revenues from running Oudh's armed forces.

 Last, but not least, the subsidiary alliance made Oudh a "buffer state", which gave strategic advantage to the British. Timeline of British-Indian subsidiary alliances

1798 – Hyderabad

1799 – Mysore

1799 – Tanjore

1801 – Awadh

1802 – Peshwa

1803 – Scindia

1803 - Gaekwad The Sikhs  Founded at the end of the 15th century by Guru Nanak, the religion spread among the Jat peasantry & other lower castes of the Punjab.

 The transformation of the Sikhs into a militant, fighting community was begun by Guru Hargobind (1606-1645).

 It was, however, under the leadership of Guru Gobind Singh (1664-1708) that they became a political and military force.

 From 1699 onwards, Guru Gobind Singh waged constant war against Aurangzeb.

 After Aurangzeb's death Guru Gobind Singh joined Bahadur Shah's camp as a noble of the rank of 5000 zat & 5000 sawar & accompanied him to the Deccan where he was treacherously murdered by one of his Pathan employees. Banda Bahadur  Rallied the Sikh peasants of the Punjab & carried on a vigorous though unequal struggle against the Mughal army.

 Was captured in 1715 & put to death.

 The invasions of Nadir Shah & Ahmad Shah Abdali

 With the withdrawal of Abdali from Punjab, they began to fill the political vacuum.

 Between 1765 and 1800 they brought the Punjab & Jammu under their control.

 12 or confederacies which operated in different parts of the province. Ranjit Singh  Sukerchakia

 A strong and courageous soldier, an efficient administrator, & a skillful diplomat.

 He soon brought all Sikh chiefs west of the Sutlej under his control and established his own kingdom in the Punjab.

 He captured Lahore in 1799 & Amritsar in 1802. Later, he conquered Kashmir, Peshawar & Multan.

 Treaty of perpetual friendship with English in 1809

 Built up an army along European line with the help of European instructors.  It is said that he possessed the second best army in Asia.

 He set up modern foundries to manufacture at Lahore & employed Muslim gunners to man them. Conquest of the Punjab  Ranjit Singh died in June 1839

 Followed by political instability & rapid changes of government.  Kingdom saw beginning of process of disintegration when Kharak Singh, his eldest son & his only son Naunihal Singh (grandson of Ranjit Singh) were killed in 1840.

 Then Sher Singh, another son of Ranjit Singh was successful with help of the Sikh army in proclaiming himself Maharaja in 1841 but he too was assassinated in 1843.  In September 1843 Duleep Singh, youngest son of Maharaja was proclaimed the king with Rani Jindan as regent & Hira Singh as Wazir (who was murdered later).

 Selfish & corrupt leaders came to the front.

 Ultimately, power fell into the hands of the brave & patriotic but utterly indiscipline army. First Anglo-Sikh War

 The corrupt chiefs & officials found that the army would sooner or later deprive them of their power, position, and possessions. They conceived the idea of saving themselves by embroiling the army in a war with the British.

 The PM Raja Lal Singh & the Commander-in-Chief Tej Singh, were secretly corresponding with the enemy.

 The danger from the foreigner united the Hindus, the Muslims & the Sikhs.

 War between the two was thus declared on 13 December 1845.

 The Punjab Army was forced to concede defeat Treaty of Lahore (1846)  Sir Henry Lawrence was appointed at the Lahore Darbar to control the policies

 Dalip Singh, was recognized as Raja

 Jullandur Doab was added to the British territory

 Limited the Sikh army to a specified number

 Later, on 16 December 1846, another treaty was signed giving the British Resident at Lahore full authority over all matters in every department of the state.

 Moreover, the British were permitted to station their troops in any part of the state, From now on the British Resident became the real ruler of the Punjab.

 The British imposed a heavy war indemnity Second Anglo Sikh war 1848-49  First major conflict during the period of Lord Dalhousie  After the treaty of Lahore, Rani Jindan realized the true intentions of the English.  The major causes for this war were :  The Sikh Sardars were discontent with the British control over Punjab,  The Sikh army wanted to avenge their humiliation in the first war  The treatment of Rani Jindan when she was removed from Lahore to Shekhupura on charges of conspiracy against the British Resident.  The immediate cause for the English Company’s invasion was the revolt of Mulraj, the Governor of Multan.  In March 1849 Dalhousie annexed Punjab under the Treaty of Lahore and pensioned off Dalip Singh to England along with his mother Rani Jindan.  Therefore, Punjab became a British province.  Although Patiala & some other small states retained their rulers after recognizing sovereignty of the British.  The Kohinoor diamond was also taken from him. Conquest of Sindh  Growing Anglo Russian rivalry in Europe and Asia

 British feared Russia might attack India through Afghanistan or Persia.  To counter it British decided to increase its influence in Afghanistan and Persia.

 It felt this policy could be success fully pursued only if Sindh was brought under British control.

 The commercial possibilities of the river Sindh were an additional attraction.

 The roads and rivers of Sindh were opened to British trade by a treaty in 1832.  The chiefs, known as Amirs, were made to sign a Subsidiary Treaty in 1839.

 In spite of previous assurances that its territorial integrity would be respected, Sindh was annexed in 1843 after a brief campaign by Sir Charles Napier Dalhousie and the Policy of Annexation (1848-1856)

 His belief that British administration was far superior to the corrupt & oppressive administration of the native rulers

 British exports to the native states of India were suffering because of the maladministration of these states by their Indian rulers

 Dalhousie also refused to recognize the titles of many ex-rulers or to pay their pensions

 Thus, the titles of the Nawabs of Carnatic & of Surat & the Raja of Tanjore were extinguished

 Similarly, after the death of the ex-Peshwa Baji Rao II, Dalhousie refused to extend his pay or pension to his adopted son, Nana Saheb. Doctrine of lapse

 Was an annexation policy purportedly devised by Lord Dalhousie.

 According to the Doctrine, any princely state or territory under the direct influence (paramountcy) of the British East India Company, as a vassal state under the British Subsidiary System, would automatically be annexed if the ruler was either "manifestly incompetent or died without a direct heir".

 The latter supplanted the long-established right of an Indian sovereign without an heir to choose a successor.

 In addition, the British decided whether potential rulers were competent enough.

 At the time of its adoption, the Company had absolute imperial administrative jurisdiction over many regions spread over the subcontinent.  The company took over the princely states of Satara (1848) , Jaitpur, & Sambalpur (1849), Nagpur and Jhansi (1854) , Tanjore & Arcot (1855), Udaipur & Awadh (1856) using this doctrine.

 The Doctrine of was a naked violation of all of national justice to deny the legitimate rights to India rulers and to grab their territories.

 The armies of the annexed states were largely disbanded.

 The Company added about 4 million pounds sterling to its annual revenue by use of this doctrine. Doctrine of Lapse before Dalhousie

 Dalhousie applied the Doctrine of Lapse vigorously for annexing Indian princely states, but the policy was not solely of his invention.

 The Court of Directors of the East India Company had articulated this early in 1834.

 As per their policy they annexed Mandavi in 1839, Kolaba & Jalaun in 1840 & Surat in 1842. Social and Economic Conditions of the People  India of the 18th century failed to make progress economically, socially, or culturally

 India of those days was also a land of contrasts.

 Even so, the life of the Indian masses was by & large better at this time than it was after over 100 years of British rule at the end of the 19th century

 Constant warfare & disruption of law & order in many areas during the 18th century harmed the country's internal trade & disrupted its foreign trade to some extent and in some directions.

 Political factors which hurt trade also adversely affected urban industries. Many prosperous cities, centres of flourishing industry, were sacked & devastated. Delhi was plundered by Nadir Shah; Lahore, Delhi & Mathura by Abdali; Agra by the Jats; Surat & other cities of Gujarat & the Deccan by Maratha chiefs; Sarhind by the Sikhs, & so on. India: Land of extensive manufactures  Indian artisans still enjoyed fame all the world over for their skill.

 India was still a large-scale manufacturer of cotton & silk fabrics, sugar, jute, dyestuffs, mineral & metallic products like arms, metal wares & saltpetre & oils.

 The important centres of textile industry were Dacca ,Murshidabad, Patna, Surat, Ahmedabad, Broach , Chanderi in , Burhanpur in Maharashtra, Jaunpur, Varanasi, Lucknow, Agra , Multan & Lahore, Masulipatam, Aurangabad, Chicacole & Vishakhapatnam in Andhra, , & Coimbatore & Madurai.

 Kashmir was a centre of woollen manufactures.

 Ship-building industry flourished in Maharashtra, Andhra & Bengal. Education  Education was defective.  Idea of education was culture not literacy

 It was traditional & out of touch with the rapid developments in the West.  The knowledge which it imparted was confined to literature, law, religion, philosophy, & logic, & excluded the study of physical & natural sciences, technology, & geography.

 Among the Hindus, higher education was based on Sanskrit learning & was mostly confined to Brahmins.  Centres of high education in Sanskrit were called Chatuspathis or tols in Bengal  Persian education being based on the official language of the time was equally popular among Hindus & Muslims.  Teachers enjoyed high prestige in the community.  Girls were seldom given education, though some women of the higher classes were an exception. Social Life  Caste was the central feature of the social life of the Hindus.  Muslims were no less divided  The family system : patriarchal  Women possessed little individuality of their own.  Ahilya Bai 1766 to 1796.  Purdah was common mostly among the higher classes in the North  Marriages were arranged  Men were permitted to have more than one wife, but except for the well-off, they normally had only one.  Early marriage

 Raja Sawai Jai Singh of Amber & the Maratha General Prashuram Bhau tried to promote widow remarriage but failed.  Patdam : Maratha tax on widow remarriage  Slavery  Traffic in Slaves abolished by a proclamation in 1789 Cultural Life  Many of the painters of the Mughal school migrated to provincial courts & flourished at Hyderabad, Lucknow, Kashmir, & Patna.

 At the same time new schools of painting were born & achieved distinction  The paintings of Kangra & Rajput schools revealed new vitality & taste.

 In the field of architecture, the Imambara of Lucknow (built by Asf-ud-Daula) reveals proficiency in technique but a decadence in architectural taste.

 On the other hand, city of Jaipur and its buildings are example of continuing vigour.  Music continued to develop & flourish in the 18th century. Significant progress was made in this field in the reign of Muhammad Shah.

 Spread of Urdu language & the growth of Urdu poetry.  Urdu gradually became the medium of social intercourse among the upper classes in north . It produced brilliant poets like Mir, Sauda. Nazir, & Mirza Ghalib. Continued  Revival of literature  Padmanabhan Palace with its remarkable architecture & mural paintings was also reconstructed

 Main weakness of Indian culture lay in the field of science

 Struggle for power and wealth, economic decline, social backwardness & cultural stagnation had harmful impact on the morals of a section of the Indian people.  Friendly relations between Hindus & Muslims were a very healthy feature

 The ways of life of the upper class Hindus & Muslims converged much more than the ways of life of upper class & lower class Hindus or of upper class & lower class Muslims

 Marathas supported Moinuddin Chisti & Raja of Tanjore supported shrine of Shahul Hamid of Nagore.